Sunteți pe pagina 1din 156

CHAPTER

Higher-Order Linear Second-Order Linear Differential Equations Differential Equations

4.1

The Harmonic Oscillator

The Undamped Oscillator 1.

x + x = 0 , x (0) = 1 , x ( 0) = 0
The general solution of the harmonic oscillator equation x + x = 0 is given by

x ( t ) = c1 cos t + c2 sin t x ( t ) = c1 sin t + c2 cos t . Substituting the initial conditions x ( 0 ) = 1 , x ( 0 ) = 0 , gives

x ( 0 ) = c1 = 1 x ( 0 ) = c2 = 0 so c1 = 1 , c2 = 0 . Hence, the IVP has the solution x ( t ) = cos t .


2.

x + x = 0 , x (0) = 1 , x (0) = 1
The general solution of the harmonic oscillator equation x + x = 0 is given by x ( t ) = c1 cos t + c2 sin t x ( t ) = c1 sin t + c2 cos t . Substituting the initial conditions x ( 0 ) = 1 , x ( 0 ) = 1 , gives x ( 0 ) = c1 = 1 x ( 0 ) = c2 = 1 or c1 = c2 = 1 . Hence, the IVP has the solution x ( t ) = cos t + sin t .

294

SECTION 4.1

The Harmonic Oscillator

295

In polar form, this would be


x ( t ) = 2 cos t . 4
3.

x + 9x = 0 , x (0) = 1 , x (0) = 1
The general solution of the harmonic oscillator equation x + 9 x = 0 is given by x ( t ) = c1 cos3t + c2 sin 3t x ( t ) = 3c1 sin 3t + 3c2 cos3t . Substituting the initial conditions x ( 0 ) = 1 , x ( 0 ) = 1 , gives x ( 0 ) = c1 = 1 x ( 0 ) = 3c2 = 1 so c1 = 1 , c2 =
1 . Hence, the IVP has the solution 3 1 x ( t ) = cos3t + sin 3t . 3 In polar form, this would be

x (t ) = where = tan 1
4.

10 cos ( 3t ) 3

1 . This would be in the first quadrant. 3

x + 4 x = 0 , x ( 0 ) = 1 , x ( 0 ) = 2
The general solution of the harmonic oscillator equation x + 4 x = 0 is given by x ( t ) = c1 cos 2t + c2 sin 2t x ( t ) = 2c1 sin 2t + 2c2 cos 2t . Substituting the initial conditions x ( 0 ) = 1 , x ( 0 ) = 2 , gives x ( 0 ) = c1 = 1 x ( 0 ) = 2c2 = 2 so c1 = 1 , c2 = 1 . Hence, the IVP has the solution x ( t ) = cos 2t sin 2t . In polar form, this would be

x ( t ) = 2 cos 2t + . 4

296

CHAPTER 4

Higher-Order Linear Differential Equations

5.

x + 16 x = 0 , x ( 0 ) = 1 , x ( 0 ) = 0 The general solution of the harmonic oscillator equation x + 16 x = 0 is given by x ( t ) = c1 cos 4t + c2 sin 4t x ( t ) = 4c1 sin 4t + 4c2 cos 4t . Substituting the initial conditions x ( 0 ) = 1 , x ( 0 ) = 0 , gives x ( 0 ) = c1 = 1 x ( 0 ) = 4c2 = 0 so c1 = 1 , c2 = 0 . Hence, the IVP has the solution x ( t ) = cos 4t .

6.

x + 16 x = 0 , x ( 0 ) = 0 , x ( 0 ) = 4
The general solution of the harmonic oscillator equation x + 16 x = 0 is given by x ( t ) = c1 cos 4t + c2 sin 4t x ( t ) = 4c1 sin 4t + 4c2 cos 4t . Substituting the initial conditions x ( 0 ) = 0 , x ( 0 ) = 4 , we get x ( 0 ) = c1 = 0 x ( 0 ) = 4c2 = 4 so c1 = 0 , c2 = 1 . The IVP has the solution x ( t ) = sin 4t .

7.

x + 16 2 x = 0 , x (0) = 0, x ( 0 ) =

0 =

16 2 = 4 1

x = c1 cos 4 t + c2 sin 4 t x = 4 c1 sin 4 t + 4 c2 cos 4 t x(0) = 0 = c1 x(0) = = 4 c2 x=


1 sin 4 t 4

c2 =

1 4

SECTION 4.1

The Harmonic Oscillator

297

8.

4 x + 2 x = 0 , x (0) = 1, x ( 0 ) =

0 =

2
4

x = c1 cos 4 t + c2 sin 4 t x = c1

sin

t + c2

cos

x(0) = 1 = c1 x(0) = = c2 x = cos

, c2 = 2

t + 2sin

Graphing by Calculator 9. y = cos t + sin t


y 1.5 T = 2

The equation tells us T = 2 and because 2 T= , 0 = 1 . We then measure the delay

angle 0.8 (1) = 0.8 . The amplitude A can be measured directly giving A 1.4 . Hence, cos t + sin t 1.4cos ( t 0.8 ) . Compare with the algebraic form in Problem 15.
10. y = 2cos t + sin t
1.5

0.8 which we can compute as the phase 0

0.8 0

t A 1.4

y 2.5

The equation tells us T = 2 and because 2 T= , 0 = 1 . We then measure the delay

0.5 0
4

T = 2

A 2.2

0.5 , which we can compute as the phase 0 angle 0.5 (1) = 0.5 . The amplitude A can be
measured directly giving A 2.2 . Hence, 2cos t + sin t 2.2cos ( t 0.5 ) .

2.5

298

CHAPTER 4

Higher-Order Linear Differential Equations

11.

y = 5cos3t + sin 3t

y 5

2 and The equation tells us that period is T = 3 2 , 0 = 3 . We then measure the because T =

0.05 T = 2 /3 A 5.1

delay

0.05 , which we can compute as the 0 phase angle 3 ( 0.05 ) = 0.15 . The amplitude A
5

can be measured directly giving A 5.1 . Hence, 5cos3t + sin 3t 5.1cos ( 3t 0.15 ) .
12. y = cos3t + 5sin 3t

The equation tells us the period is T = because T = 2

2 and 3

0.5 0

T = 2 /3

A 5.1

, 0 = 3 . We then measure the


3 t

0.5 , which we can compute as the delay 0 phase angle 0.5 ( 3) = 1.5 . The amplitude A
can be measured directly giving A 5.1 . Hence, cos3t + 5sin 3t 5.1cos ( 3t 1.5 ) .
13. y = cos5t + 2sin 5t
y 5

equation tells us that period is T = because T = delay 2

2 5

and

0.4 T = 2 /5

A 2.2

the phase angle 5 ( 0.4 ) = 2 . The amplitude A can be measured directly giving A 2.2 . Hence,
cos5t + 2sin t 2.2cos ( 5t 2 ) .

or 0.4 , which we can compute as 0 8

, 0 = 5 . We then measure the


1 2

SECTION 4.1

The Harmonic Oscillator

299

Alternate Forms for Sinusoidal Oscillations 14.

We have
A cos (0t ) = A ( cos 0t cos + cos 0t cos ) = ( A cos ) cos 0t + ( A sin ) sin 0t = c1 cos 0t + c2 sin 0t where c1 = A cos , c2 = A sin .
Single-Wave Forms of Simple Harmonic Motion

15.

cos t + sin t
By Equation (4) c1 = 1 , c2 = 1 , and 0 = 1 . By Equation (5)

A= 2 , =
yielding

cos t + sin t = 2 cos t . 4 (Compare with solution to Problem 9.)


16.

cos t sin t
By Equation (4) c1 = 1 , c2 = 1 , and 0 = 1 . By Equation (5)

A= 2 , =
yielding

cos t sin t = 2 cos t + . 4 Because c1 is positive and c2 is negative the phase angle is in the 4th quadrant.
17.

cos t + sin t By Equation (4) c1 = 1 , c2 = 1 , and 0 = 1 . By Equation (5) A= 2 , = yielding


3 cos t + sin t = 2 cos t 4 .

3 4

Because c1 is negative and c2 is positive the phase angle is in the 2nd quadrant.

300

CHAPTER 4

Higher-Order Linear Differential Equations

18.

cos t sin t By Equation (5) c1 = 1 , c2 = 1 , and 0 = 1 . By Equation (6)


5 4

A= 2 , =
yielding

5 cos t sin t = 2 cos t 4

Because c1 and c2 are negative, the phase angle is in the 3rd quadrant.
Component Form of Harmonic Motion

Using cos ( A + B ) = cos A cos B sin A sin B , we write:


19. 20.

2cos ( 2t ) = 2{cos 2t cos ( ) sin 2t sin ( )} = 2cos 2t 3 1 cos t + = cos t cos sin t sin = cos t sin t 3 2 3 3 2 2 3 2 2 3cos t = 3 cos t cos sin t sin = 3 cos t + sin t = {cos t + sin t} 4 4 4 2 2 2

21.

22.

3 1 cos 3t = cos3t cos sin 3t sin = cos3t + sin 3t 6 6 6 2 2


Interpreting Oscillator Solutions

23.

x + x = 0 , x (0) = 1 , x ( 0) = 0 Because 0 = 1 , we know the natural frequency is


1 Hz and the period is 2 seconds. Using 2

the initial conditions, we find the solution (see Problem 1) x ( t ) = cos t , which tells us the amplitude is 1 and the phase angle = 0 radians.
24.

x + x = 0 , x (0) = 1 , x (0) = 1
Because 0 = 1 radians per second, we know the natural frequency is 1 Hz (cycles per 2

second), and the period is 2 . Using the initial conditions, we find the solution (see Problem 2) x ( t ) = 2 cos t , 4 which tells us the amplitude is 2 and the phase angle is =

radians.

SECTION 4.1

The Harmonic Oscillator

301

25.

x + 9x = 0 , x (0) = 1 , x (0) = 1 Because 0 = 3 radians per second, we know the natural frequency is second), and the period is 3 Hz (cycles per 2

2 . Using the initial conditions, we find the solution (see Problem 3) 3

x (t ) = where = tan 1

10 cos ( 3t ) 3

1 , which tells us the amplitude is 3

10 3 and the phase angle is

= tan 1 0.3218 radians.


26.

1 3

x + 4 x = 0 , x ( 0 ) = 1 , x ( 0 ) = 2 Because 0 = 2 radians per second, we know the natural frequency is 1

Hz (cycles per second),

and the period is . Using the initial conditions, we find the solution (see Problem 4)

x ( t ) = 2 cos 2t + , 4
which tells us the amplitude is
27.

2 and the phase angle is =

radians.

x + 16 x = 0 , x ( 0 ) = 1 , x ( 0 ) = 0 Because 0 = 4 radians per second, we know the natural frequency is and the period is 2

Hz (cycles per second),

. Using the initial conditions, we find the solution (see Problem 5) x ( t ) = cos ( 4t ) ,

which tells us the amplitude is 1 and the phase angle is = radians.

302

CHAPTER 4

Higher-Order Linear Differential Equations

28.

x + 16 x = 0 , x ( 0 ) = 0 , x ( 0 ) = 4 Because 0 = 4 radians per second, we know the natural frequency is and the period is 2

Hz (cycles per second),

. Using the initial conditions, we find the solution (see Problem 6)

x ( t ) = cos 4t , 2
which tells us the amplitude is 1 and the phase angle is =
29.

radians.

x + 16 2 x = 0 , x (0) = 0, x ( 0 ) =
From Problem 7, x = Amplitude =
1 sin 4 t 4

1 1 2 2 = 8. , x = cos 4 t , phase angle = , and period T = = 2 0 4 4 2 4

30.

4 x + 2 x = 0 , x (0) = 1, x ( 0 ) = 4r2 + 2 = 0

r=

x = c1 cos x=

t + c2 sin

1 = c1

c1 sin

t+

2
t

c2 cos

c2

x = cos

t + 2sin

c2 = 2

Amplitude: A = 1 + 22 = 5

x=

5 cos t 1.11 2

Relating Graphs 31.

(a) (b)

See graph, next page.

x + 0.25 x = 0
From (4)

0 =

k = 0.5 m

x = c1 cos 0.5t + c2 sin 0.5t


c t t x(0) = 0 c1 = 0 so x(t) = c2 sin and x(t ) = 2 cos . 2 2 2

SECTION 4.1

The Harmonic Oscillator

303

Alternatively, you could use (5)

x = A cos (0.5t ) x(0) = 0 =


and x(t ) = (c) See graph

t t so x(t) = A cos = A sin , 2 2 2

A t cos . 2 2

Graph for b) and d) (d) Amplitudes are approximately

Graph for a)

A A 2A 5A , , , , and A 3 2 3 6

Phase Portraits

For comparison of phase portraits, the main observation is that the elliptical shape depends on 0, which is k in all of these problems because x + kx = 0 . If 0 = 1, trajectories are circular. As 0 increases above 1, ellipses become taller and thinner. As 0 decreases from 1 to 0, ellipses become shorter and wider. x 1 The aspect ratio of max = . xmax Other observations include: All these phase portraits show closed elliptical trajectories that circulate clockwise. The trajectory of Problem 33 has a greater radius than that of Problem 32 because the initial condition is further from the origin. The trajectories in Problems 36 and 37 are on the same ellipse with different starting points that give different solution equations.

304

CHAPTER 4

Higher-Order Linear Differential Equations

32.

x+x=0

1 x0 = 0

From Problem 1 x(t) = cos(t), so x(t ) = sin(t).

33.

x+x=0

1 x0 = 1
x(t) = cos(t) + sin(t), x(t ) = sin(t) + cos(t).

From Problem 2 so

34.

x + 9x = 0

1 x0 = 1
x(t) = cos 3t +
1 sin 3t , 3

From Problem 3, so

x(t ) = 3 sin 3t + cos 3t.

SECTION 4.1

The Harmonic Oscillator

305

35.

x + 4x = 0

1 x0 = 2
x(t) = cos 2t sin 2t, x(t ) = 2 sin 2t 2 cos 2t.

From Problem 4, so

36.

x + 16 x = 0

1 x0 = 0
x(t) = cos 4t, x(t ) = 4 sin 4t.

From Problem 5, so

37.

x + 16 x = 0

0 x0 = 4
x(t) = sin 4t, x(t ) = 4 cos 4t.

From Problem 6, so

306

CHAPTER 4

Higher-Order Linear Differential Equations

38.

x + 16 2 x = 0

0 x0 =
x(t) =

From Problem 7, so

1 sin 4 t , 4

x(t ) = cos 4 t.

39.

4x + 2 x = 0

1 x0 =

From Problem 8, x(t) = cos so


x(t ) =

t + 2sin

t,

sin

t + cos

t.

Matching Problems 40. 41. 42. 43. B A D C

SECTION 4.1

The Harmonic Oscillator

307

Changing Frequencies 44. (a)

0 = 0.5 gives tx curve with lowest frequency (fewest humps); 0 = 2 gives the highest frequency (most humps).

4 2

x 0 = 05 = 2 . 0

(b)

0 = 0.5 gives the innermost phaseplane trajectory; as 0 increases, the amplitude of x increases. In Figure 4.1.8 the trajectory that is not totally visible is the one for 0 = 2 .
2 4

0 = 1

Detective Work 45. (a) 8 is a sinusoidal curve with period 2 , amplitude A 1.4 , The curve y = 1.4cos t 5 8 and phase angle . 5 From this graph we estimate 0 = 1 , A 2.3 , and

(b)

. Thus, we have

x ( t ) = A cos (0t 0 ) = 2.3cos t = 2.3 cos t cos sin t sin 4 4 4 2 2 = 2.3 cos t + sin t 1.6 ( cos t + sin t ) . 2 2 Pulling a Weight 46. (a) The mass is m = 2 kg . Because a force of 8 nt stretches the spring 0.5 meters, we find 8 that k = = 16 nt m . If we then release the weight, the IVP describing the motion of 0.5 the weight is 2 x + 16 x = 0 or

x + 8 x = 0 , x ( 0 ) = 0.5 , x ( 0 ) = 0 .
The solution of the differential equation is
x ( t ) = A cos

8t .

Using the initial conditions, we get the simple oscillation


x ( t ) = 0.5cos

( 8t ) .

308

CHAPTER 4

Higher-Order Linear Differential Equations

(b)

Amplitude =

1 2 2 8 = sec , f = m; T = cycles per second 2 2 0 8

(c)

Setting cos

( 8t ) = 0 , we find that the weight will pass through equilibrium at


t=

1 of the 4

period or after

2 8

0.56 seconds.

At that time velocity is x ( 0.56 ) = 2 sin 1.414 m sec 2 moving away from original displacement.

Finding the Differential Equation 47.


(a) The mass is m = 500 gm, which means the force acting on the spring is 500 980 dynes. This stretches the spring 50 cm, so the spring constant is k= 500 980 = 9800 dynes cm . 50

The mass is then pulled down 10 cm from its initial displacement, giving x ( 0 ) = 10 (as long as we measure downward to be the positive direction, which is typical in these problems). The initial velocity of the mass is assumed to be zero, so x ( 0 ) = 0 . Thus, the IVP for the mass is

500 x + 9800 x = 0
or

5 x + 98 x = 0 , x ( 0 ) = 10 , x ( 0 ) = 0 .
(b) The solution of the differential equation found in part (a) is 98 98 x ( t ) = A cos t = 10cos t. 5 5 (c) In part (b) the amplitude is 10 cm, phase angle is 0, the period is T = 2 m 5 = 2 1.4 sec, k 98
1 = 0.71 oscillations per sec1.4

and the natural frequency is given by the reciprocal f = ond.

SECTION 4.1

The Harmonic Oscillator

309

Initial-Value Problems 48.


(a) 16 1 = slugs. (See Table 4.1.1 in text.) This 32 2 1 16 foot, hence k = 1 = 32 lb ft . This yields the equation mass stretches the spring 2 2 The weight is 16 lbs, so the mass is roughly
1 ( x ) + 32 x = 0 , or 2

x + 64 x = 0 . The initial conditions are that the mass is pulled down 4 inches ( 1 foot) from equilib3 rium and then given an upward velocity of 4 ft sec . This gives the initial conditions of 1 x (0) = ft, x ( 0 ) = 4 ft/sec, using the engineering convention that for x, down is 3

positive. (b) We have the same equation x + 64 x = 0 , but the initial conditions are x ( 0 ) = x ( 0 ) = 1 ft/sec. 1 ft, 6

One More Weight 49.


The mass is m = 12 3 1 = slugs. The spring is stretched foot, so the spring constant is 32 8 2 12 1 1 k = 1 = 24 lb ft . The initial position of the mass is 4 inches ( ft) upward so x ( 0 ) = . The 3 3 2

initial motion is 2 ft sec upward, and thus x ( 0 ) = 2 . Hence, the equation for the motion of the mass is
1 x + 64 x = 0 , x ( 0 ) = , x ( 0 ) = 2 , 3

which has the solution 1 1 x ( t ) = cos8t sin 8t . 3 4

310

CHAPTER 4

Higher-Order Linear Differential Equations

Writing this in polar form, we have 5 1 1 2 A = c12 + c2 = + = 12 3 4


2 2

x 0.4

= tan 1

1 3 = tan 4 3.78 radians ( angle in 3rd quadrant ) .


0.4

c2 c1

0.2 0.4 0.6 0.8 1 1.2 1.4

Hence, we have the solution in polar form x (t ) = See figure. 5 cos ( 8t 3.78 ) . 12

x(t ) =

5 cos(8t 378) . 12

Spring oscillation

Comparing Harmonic Motions 50.


The period of simple harmonic motion is given by T = 2

, where 0 =

k . Notice that this m

does not depend at all on our initial conditions. Period is the same so is the frequency, but the amplitude will be twice that in the first case.

Testing Your Intuition 51.

x + x + x3 = 0
Here we have a vibrating spring with no friction, but a nonlinear restoring force F = x x3 that is stronger than a purely linear force x. For small displacement x the nonlinear F will not be much different (for small x, x3 is very small), but for larger x, the force F will be much stronger than in a linear spring; as F increases, the frequency of the vibration increases. This equation is called Duffings (strong) equation, and the associated springs are called strong springs.

52.

x + x x3 = 0
Here we have a vibrating spring with no friction, and a nonlinear restoring force F = x + x3 . For small displacement x the nonlinear term x3 has little effect, but as x increases toward 1, the restoring force F diminishes (i.e., the spring weakens when it is stretched a lot, and the restoring force becomes zero when x = 1 ). The decreasing F causes decreasing frequency (and increasing period). This equation is called Duffings (weak) equation, and the associated springs are called weak springs.

SECTION 4.1

The Harmonic Oscillator

311

53.

xx=0 This equation describes a spring with no friction and a negative restoring force. You may wonder if there are such physical systems. In the next two sections we will see that this equation describes the motion of an inverted pendulum (4.3 Problems 58, 59), and it has solutions sinh t and cosh t (4.2 Example 2), in contrast to x + x = 0 , which has solutions sin t and cos t. The restoring force for the equation under discussion is always directed away from the equilibrium position; hence the solution always moves away from the equilibrium, which us unstable.

54.

1 x+ x+x=0 t This equation can be interpreted as describing the motion of a vibrating mass that has infinite 1 friction x at t = 0 , but friction immediately begins to diminish and approaches zero as t t becomes very large. You may simulate in your mind the motion of such a system. Do you think for large t that the oscillation might behave much like simple harmonic motion? (See 4.3 Problem 68.)

55.

x + x2 1 x + x = 0
This is called van der Pols equation and describes oscillations (mostly electrical) where internal friction depends on the value of the dependent variable x. Note that when x < 1 , we actually have negative friction, so for a small displacement x we would expect the system to move away from the zero solution (an unstable equilibrium) in the direction of x = 1 . But when x > 1 , we will have positive friction causing damping. We will see in 4.3 Problem 70 and in Chapter 7 that there is a periodic solution between small x and large x that attracts all these other solutions.

56.

x + tx = 0
Here we have a vibrating spring with no friction, but the restoring force tx gets stronger as time passes. Hence we expect to see no damping, but faster vibrations as t increases.
LR-Circuit

57.

(a)

Without having a capacitor to store energy, we do not expect the current in the circuit to oscillate. If there had been a constant voltage V0 on in the past, we would expect the V current to be (by Ohms law) I = 0 . If we then shut off the voltage, we would expect R the current to die off in the presence of a resistance.

(b)

If a current I passes through a resistor with resistance R, then the voltage drop is RI; the voltage drop across an inductor of inductance L is LI . We obtain the IVP:
LI + RI = 0 , I ( 0 ) = V0 R

312

CHAPTER 4

Higher-Order Linear Differential Equations

(c)

The solution of the IVP is


I (t ) = V0 ( R L )t . e R

(d)

1 If R = 40 ohms, L = 5 henries, V0 = 10 volts, then I ( t ) = e 8t ohms. 4

LC-Circuit

58.

(a)

With a nonzero initial current and no resistance, we do not expect the current to damp to zero. We would expect an oscillatory current due to the capacitor. Thus the charge on the capacitor would oscillate indefinitely. The exact behavior depends on the initial conditions and the values of the inductance and capacitance.

(b)

Kirchoffs voltage law states that the sum of the voltage drops around the circuit is equal to the impressed voltage source. Hence, we have
LI +

1 I =0 C

or, in terms of the charge across the capacitor, we have the IVP
LQ + 1 Q = 0 , Q ( 0) = 0 , Q ( 0) = 5 . C

(c)

The solution of the IVP is

Q (t ) = 5

sin

1 LC t 1 LC

).

This agrees with the oscillatory behavior predicted in part (a). (d) With values L = 10 henries, C = 103 farads, the charge on the capacitor is
Q (t ) = 5 sin

100t 100

) = 1 sin10t .
2

A Pendulum Experiment 59.


The pendulum equation is

g sin = 0 . L

For small , we can approximate sin , giving the differential equation

+ =0.

g L

SECTION 4.1

The Harmonic Oscillator

313

This is the equation of simple harmonic motion with circular frequency 0 = frequency f 0 =
1 2 g 1 L . Hence, the period of motion is T = . = 2 f0 g L

g , and natural L

Tearth = Tsun

g sun = 400,000 = 100 40 632 . g earth

Changing into Systems 60.


4 x 2 x + 3 x = 17 cos t x= y y= 1 (3x + 2 y + 17 cos t ) 4

61.

1 Lq + Rq + q = V (t ) c q=I I= 5q + 15q + 1 1 q RI + V (t ) L c

62.

1 q = 5cos3t 10 q=I I = 1 q 3I + cos3t 50

63.

t 2 x + 4tx + x = t sin 2t 4 1 sin 2t x+ x+ 2 x= t t t x= y y=

t>0

x 4 sin 2t y+ 2 t t t

64.

4 x + 16 x = 4sin t x + 4 x = sin t x= y y = 4 x + sin t

Circular Motion 65.


Writing the motion in terms of polar coordinates r and and using the fact that the angular velocity is constant, we have = 0 (a constant). We also know the particle moves along a circle of constant radius, which makes r a constant. We then have the relation x = r cos , and hence

314

CHAPTER 4

Higher-Order Linear Differential Equations

x = ( r sin ) x = ( r sin ) ( r cos ) 2 . Because = 0 , = 0 , we arrive at the differential equation


2 x + 0 x = 0 .

Another Harmonic Motion 66.


For simple harmonic motion the circular frequency 0 is

0 =
so the natural frequency f 0 is
f0 = 1 2

kR 2 , mR 2 + I
kR 2 . mR 2 + I

Motion of a Buoy 67.


The buoy moves in simple harmonic motion, so the period is

T = 2.7 =

2 m = 2 . 0 k

We have one equation in two unknowns, but the buoyancy equation yields the second equation. If we push the buoy down 1 foot, the force upwards will be F = V , where V is the submerged volume and is the density of water. In this case, V = r 2 h , r = 9 inches = 0.75 ft , h = 1 ft, and 9 = 62.5 ft sec , so the force required to push the buoy down 1 foot is (1)( 62.5 ) 110 lbs. 16 110 But k is the force divided by distance, so k = = 110 lbs ft . Finally, solving for m in the 1 kT 2 equation for T, we get m = 2 , and substituting in all of our numbers, we arrive at m 20.4 4 slugs (see Table 4.1.1. in the text.) The buoy weighs mg = ( 20.4 )( 32.2 ) = 657 lbs.

Los Angeles to Tokyo 68.


(a) Along the tunnel,

mx = kr cos = kx x(0) = d if x is measured positive to the left of the center of the tunnel. x(0) = 0 means that the train starts from rest (as soon as a brake is released).

SECTION 4.1

The Harmonic Oscillator

315

(b)

The solution to the IVP in part (a) is

x(t) = c1 cos 0t + c2 sin 0 t,


where 0 =

k . m

At the surface of the earth mg = kr where r = R, so 0 =


k = m q . R

Letting x(0) = d yields c1 = d, while letting x(0) = 0 yields c2 = 0. Hence we have

x(t) = d cos

q t. R

For the train to go from L.A. to Tokyo, x(t) goes from d to d. and
q t goes from 0 to . R

Hence,
tf = = R q

4000 mi 5280 ft/mi 32 ft/sec 2 = 2552 sec 42.5 minutes (c) The solution tf = R from part (b) does not depend on the location of the points on the q

earths surface; , R, and q are all constant.

Factoring Out Friction 69.


(a) Letting x ( t ) = e(
b 2 m )t

X ( t ) , we have

b ( b 2 m )t b 2 m )t e X ( t ) + e( X (t ) 2m b 2 ( b 2 m )t b ( b 2 m )t b 2m t x (t ) = e X (t ) + e X (t ) + e ( ) X (t ). 2 m 4m x (t ) =

316

CHAPTER 4

Higher-Order Linear Differential Equations

b 2m t Substituting this into the original equation (1) and dividing through by e ( ) , we arrive

at b b2 b mX X + X + b X + X + k[X ] = 0 . 2 m 2m 4m Rearranging terms gives b2 b2 + k X = 0 mX + [ b + b ] X + 4 m 2m or b2 mX + k X =0. 4m (b) If we assume k b2 > 0 , then divide by m and let 4m

0 =
we find the solution of this DE in X is

1 4mk b 2 ) 2m

X ( t ) = c1 cos 0t + c2 sin 0t = A cos (0t ) . Thus, we have


b 2m t b 2m t x ( t ) = e ( ) X ( t ) = Ae ( ) cos (0t ) .

Suggested Journal Entry 70.


Student Project

SECTION 4.2

Real Characteristic Roots

317

4.2

Real Characteristic Roots

Real Characteristic Roots 1.


y = 0 The characteristic equation is r 2 = 0 , so there is double root at r = 0 . Thus, the general solution is

y ( t ) = c1e0t + c2te0t = c1 + c2t .


2.

y y = 0
The characteristic equation is r 2 r = 0 , which has roots 0, 1. Thus, the general solution is

y ( t ) = c1 + c2 et .

3.

y 9 y = 0
The characteristic equation is r 2 9 = 0 , which has roots 3, 3. Thus, the general solution is

y ( t ) = c1e3t + c2 e 3t .

4.

y y = 0
The characteristic equation is r 2 1 = 0 , which has roots 1, 1. Thus, the general solution is

y ( t ) = c1et + c2 et .
5.

y 3 y + 2 y = 0
The characteristic equation is r 2 3r + 2 = 0 , which factors into ( r 2 )( r 1) = 0 , and hence has roots 1, 2. Thus, the general solution is

y ( t ) = c1et + c2 e 2t .
6.

y y 2 y = 0
The characteristic equation is r 2 r 2 = 0 , which factors into ( r 2 )( r + 1) = 0 , and hence has roots 2, 1. Thus, the general solution is

y ( t ) = c1e 2t + c2 e t .

7.

y + 2 y + y = 0
The characteristic equation is r 2 + 2r + 1 = 0 , which factors into ( r + 1)( r + 1) = 0 , and hence has the double root 1, 1. Thus, the general solution is

y ( t ) = c1e t + c2te t .

318

CHAPTER 4

Higher-Order Linear Differential Equations

8.

4 y 4 y + y = 0 The characteristic equation is 4r 2 4r + 1 = 0 , which factors into ( 2r 1)( 2r 1) = 0 , and hence has the double root 1 1 , . Thus, the general solution is 2 2

y ( t ) = c1et 2 + c2tet 2 .

9.

2 y 3 y + y = 0 The characteristic equation is 2r 2 3r + 1 = 0 , which factors into ( 2r 1)( r 1) = 0 , and hence has roots
1 , 1. Thus, the general solution is 2

y ( t ) = c1et 2 + c2 et .
10.

y 6 y + 9 y = 0
The characteristic equation is r 2 6r + 9 = 0 , which factors into ( r 3)( r 3) = 0 , and hence has the double root 3, 3. Thus, the general solution is

y ( t ) = c1e3t + c2te3t .
11.

y 8 y + 16 y = 0
The characteristic equation is r 2 8r + 16 = 0 , which factors into ( r 4 )( r 4 ) = 0 , and hence has the double root 4, 4. Thus, the general solution is

y ( t ) = c1e 4t + c2te 4t .

12.

y y 6 y = 0
The characteristic equation is r 2 r 6 = 0 , which factors into ( r + 2 )( r 3) = 0 , and hence has roots 2, 3. Thus, the general solution is

y ( t ) = c1e 2t + c2 e3t .

13.

y + 2 y y = 0
The characteristic equation is r 2 + 2r 1 = 0 , which factors into r + 1 2 r + 1 + 2 = 0 , and hence has roots 1 + 2 , 1 2 . Thus, the general solution is
y ( t ) = e t c1e

)(

2t

+ c2 e

2t

).
2

14.

9 y + 6 y + y = 0 The characteristic equation is 9r 2 + 6r + 1 = 0 , which factors into ( 3r + 1) = 0 , and hence has the
1 1 double root , . Thus, the general solution is 3 3

y ( t ) = c1et 3 + c2te t 3 .

SECTION 4.2

Real Characteristic Roots

319

Initial Values Specified 15.

y 25 y = 0 , y ( 0 ) = 1 , y ( 0 ) = 0
The characteristic equation of the differential equation is r 2 25 = 0 , which factors into ( r 5)( r + 5) = 0 , and thus has roots 5, 5. Hence,
y ( t ) = c1e5t + c2 e 5t .

Substituting in the initial conditions y ( 0 ) = 1 gives c1 + c2 = 1 . Substituting in y ( 0 ) = 0 gives 5c1 5c2 = 0 . Solving for c1 , c2 gives c1 = c2 = 1 . Thus the general solution is 2

1 1 y ( t ) = e 5 t + e 5 t . 2 2

16.

y + y 2 y = 0 , y ( 0 ) = 1 , y ( 0 ) = 0

The characteristic equation of the differential equation is r 2 + r 2 = 0 , which factors into ( r + 2 )( r 1) = 0 , and thus has roots 1, 2. Thus, the general solution is
y ( t ) = c1e 2t + c2 et .

Substituting into y ( 0 ) = 1 , y ( 0 ) = 0 yields c1 =

1 2 , c2 = , so 3 3

1 2 y ( t ) = e 2 t + e t . 3 3

17.

y + 2 y + y = 0 , y ( 0 ) = 0 , y ( 0 ) = 1
The characteristic equation is r 2 + 2r + 1 = 0 , which factors into ( r + 1)( r + 1) = 0 , and hence has the double root 1, 1. Thus, the general solution is
y ( t ) = c1e t + c2te t .

Substituting into y ( 0 ) = 0 , y ( 0 ) = 1 yields c1 = 0 , c2 = 1 , so


y ( t ) = te t .

18.

y 9 y = 0 , y ( 0 ) = 1 , y ( 0 ) = 0

The characteristic equation is r 2 9 = 0 , which factors into ( r 3)( r + 3) = 0 , and hence has roots are 3, 3. Thus, the general solution is
y ( t ) = c1e3t + c2 e 3t .

Substituting into y ( 0 ) = 1 , y ( 0 ) = 0 yields c1 = c2 =

1 , so 2

1 1 y ( t ) = e3t e 3t . 2 2

320

CHAPTER 4

Higher-Order Linear Differential Equations

19.

y 6 y + 9 y = 0 , y ( 0 ) = 0 , y ( 0 ) = 1
The characteristic equation is r 2 6r + 9 = 0 , which factors into ( r 3)( r 3) = 0 , and hence has the double root 3, 3. Thus, the general solution is

y ( t ) = c1e3t + c2te3t .
Substituting into y ( 0 ) = 0 , y ( 0 ) = 1 yields c1 = 0 , c2 = 1 , so
y ( t ) = te3t .

20.

y + y 6 y = 0 , y ( 0 ) = 1 , y ( 0 ) = 1
The characteristic equation is r 2 + r 6 = 0 , which factors into ( r + 3)( r 2 ) = 0 , and hence has roots 3, 2. Thus, the general solution is
y ( t ) = c1e 3t + c2 e 2t .

Substituting into y ( 0 ) = 1 , y ( 0 ) = 1 yields c1 =

1 4 , c2 = , so 5 5

1 4 y ( t ) = e 3t + e 2t . 5 5

21.

y y = 0 r r=0
2

y(0) = 2,

y(0) = 1

(Characteristic equation)
r = 0, 1

r(r 1) = 0

y = c1 + c2et 2 = c1 + c2 y = c2et 1 = c2, y = 3 et c1 = 3

22.

y 4 y 12 y = 0 r2 4r 12 = 0

y(0) = 1, y (0) = 1

(Characteristic equation)
r = 2, 6
y (0) = 1 c1 + c2 = 1 7 1 c1 = , c2 = y (0) = 1 2c1 + 6c2 = 1 8 8

(r + 2)(r 6) = 0
y = c1e 2t + c2 e6t y = 2c1e y=
2 t

+ 6c2 e

6t

1 2t 3 6t e + e 4 4

SECTION 4.2

Real Characteristic Roots

321

Bases and Solution Spaces 23.


y 4 y = 0 r2 4r = 0

(Characteristic equation)

r(r 4) = 0 r = 0, 4

Basis: {1, e4t} Solution Space: {y y = c1 + c2e4t; c1, c2 }

24.

y 10 y + 25 y = 0 r2 10r + 25 = 0

(Characteristic equation)

(r 5)2 = 0 r = 5, 5 Basis: {e5t, te5t} Solution Space: {yy = c1e5t + c2te5t; c1, c2 }

25.

5 y 10 y 15 y = 0 5r2 10r 15 = 0 Basis: {e3t, et} Solution Space: {y y = c1e3t + c2et; c1, c2 } (Characteristic equation)

r2 2r 3 = 0 (r 3)(r + 1) = 0 r = 3, 1

26.

y + 2 2 y + 2 = 0 r 2 + 2 2r + 2 = 0

(Characteristic equation)
r = 2, 2

(r + 2)(r + 2) = 0 Basis: e

2t

, te

2t

}
y = c1e
2t

Solution Space:

{y

+ c2te

2t

; c1 , c2

Other Bases 27.


y 4 y = 0 r2 4 = 0 (Characteristic equation)

r = 2 {e2t, e2t} is a basis To show {cosh 2t, sinh 2t} is a basis, we need only show that cosh 2t and sinh 2t are linearly independent solutions: W= cosh 2t 2sinh 2t sinh 2t = 4 cosh2 2t 4 sinh2 2t 2cosh 2t

322

CHAPTER 4

Higher-Order Linear Differential Equations


2

e 2t + e 2t e 4t + 1 + e 4t cosh 2t = = 2 2
2

e 2t e 2t e 4t 1 e 4t sinh 2t = = 2 2
2

so cosh2 2t sinh2 2t = 1 and W = 4 0. cosh 2t, sinh 2t are linearly independent. Substitute y = cosh 2t, y = 2 sinh 2t, y = 4 cosh 2t Then y 4y = 4 cosh 2t 4 cosh 2t = 0 y = cosh 2t is a solution.

In similar fashion, we can show that y = sinh 2t is also a solution. To show that e 2t ,cosh 2t is a basis, we use the facts that e2t and cosh 2t are solutions. Then: W=
e 2t
2e 2 t cosh 2t 2sinh 2t

e 2t 2e 2 t

e 2 t + e 2 t 2 2t e e 2t

= (e4t 1) (e4t + 1) = 2 0 e2t and cosh 2t are linearly independent

28.

y = 0 r2 = 0 (Characteristic equation) so that r = 0, 0. Basis: {1, t} To show {t + 1, t 1} is also a basis: Note that for both y = t + 1 and y = t 1, y = 0, so both are solutions. W= t +1 t 1 1 1 = (t + 1) (t 1) = 2

t + 1, t 1 are linearly independent To show {2t, 3t 1} is another basis: Note that for both y = 2t and y = 3t 1, y = 0, so both are solutions. W= 2t 3t 1 2 3 - 6t 2(3t 1) = 2

2t, 3t 1 are linearly independent

SECTION 4.2

Real Characteristic Roots

323

The Wronskian Test


t +1 t 1 t2 + t 1 2t + t 3t 2 t 1 t2 + t t3 2t + 1 3t 2 = (t + 1) 0 2 6t 1 0 2 6t 2 6t 0 0 6 0 0 6 0 6 t3

29.

W=

1 0 0

1 0 0

= (t + 1)12 12(t 1) = 24 0 Yes, {t + 1, t 1, t2 + t, t3} is a basis for the solution space for y(4) = 0.
te5t e 5t 5e5t 25e5t 2e 5 t 1 10e5t 50e5t t 1 2 e 5t = e5t 5t + 1 5 10 25t + 10 25 50

30.

W=

(5t + 1)e5t (25t + 10)e5t

5 10 5t + 1 10 5t + 1 5 1 + (2 e 5t ) = e5 t t 25t + 10 50 25t + 10 25 25 50

= 25e5t 0
Yes, {te5t, e5t, 2e5t 1} is a basis for the solution space for y 10 y + 25 y = 0.

31.

The given set has only three solutions, so it cannot be a basis. A basis for the solution space of y(4) = 0 must have 4 linearly independent solutions.

Sorting Graphs 32.

324

CHAPTER 4

Higher-Order Linear Differential Equations

Relating Graphs
For Problems 3335, x + 5 x + 6 x = 0 has (from Example 1) solutions x(t ) = c1e 2t + c2 e 3t (1) x(t ) = 2c1e2t 3c2 e 3t (2)

33.

(a), (b)

x(0) 10 c1 + c2 = 10

x(0) 0 2c1 3c2 = 0

c1 = 30, c2 = 20 (c) From (1) in box, x(t) = 30e2t + 20e3t. For t > 0, each term diminishes as t increases; the result remains negative, below the t-axis. For t < 0, each exponential increases as t decreases; the negative term cancels the positive term when 30e2t = 20e3t or et = 1.5, that is, when t = ln 1.5 .405 which looks about right on the tx-graph. (d) From (2), x(t ) = 60e2t 60e3t = 60e2t(1 et) which is always positive for t > 0, decreasing as t increases. x(t ) reaches a maximum when x(t ) = 120e 2t + 180e3t = 0
2 + 3e t = 0 2 e t = , 3 so t = ln
2 0.406, which looks about right on the tx -graph. 3

SECTION 4.2

Real Characteristic Roots

325

34.

(a)

(b) (b) x(0) 5 c1 + c2 = 5 x(0) 0 2c1 3c2 = 0

Because all problems for finding ci are of type Ac = b , we solve for c = A 1b 3 1 5 1 1 1 We have A = so A = 2 1 , and here b = 0 2 3 so 3 1 5 +15 c = = 2 1 0 10 (c) From (1) in box on previous page, x(t)= 15e2t 10e3t. As t increases from zero, both exponentials decrease with their sum remaining positive, which agrees with the tx graph. (d) From (2), x(t ) = 30e2t 30e3t = 30e2t (1 + et) For t > 0, this quantity is always negative, and as long as t increases, each term gets closer to zero, in agreement with tx -graph.

326

CHAPTER 4

Higher-Order Linear Differential Equations

35.

(a) (b)

x(0) 0 and x(0) 8 From the method of 34(b), c1 3 1 0 8 1 0 c = A 8 = 2 1 8 = +8 2 so from (1), x(t) = 8e2t + 8e3t.

(c)

For t > 0, e 2t > e3t so the sum is always negative and approaches zero as t increases, in agreement with the tx graph.

(d)

From (2) x(t ) = +16e2t 24e3t = 0, so 2e2t 3e3t = 0, which yields et = 2 or t .406, 3

which looks about right on the tx -graph. For t > 0.406, x(t ) > 0 and decreases toward zero as t increases.

SECTION 4.2

Real Characteristic Roots

327

For Problems 3639, x x 6 x = 0 has (from Example 1) solutions

x(t ) = c1e2t + c2 e3t x(t ) = 2c1e 2t + 3c2 e3t


36.
(a)

(1) (2)

(b)

From (1) From (2)

c1 + c2 = 0 2c1 + 3c2 = 2

2 2 c1 = ; c2 = 5 5

2 2 x(t) = e 2t + e3t 5 5 4 2t 6 3t x(t ) = e + e 5 5

(c)

For t > 0, e2t < e3t, so x(t) is always positive, and as t increases, so does x(t). This result agrees with the tx-graph. For t < 0, e3t < e2t so x(t) is always negative, and as t becomes more negative, x(t) becomes more negative.

(d)

x(t ) is always positive.


For t > 0, e2t < e3t, so the second term dominates as t increases, and x(t ) increases as well. These facts are in agreement with the tx -graph.

37.

(a)

328

CHAPTER 4

Higher-Order Linear Differential Equations

(b)

From (1), From (2),

c1 + c2 = 2 2c1 + 3c2 = 0 c1 =

6 4 ; c2 = 5 5

x(t) =

6 2t 4 3t e + e 5 5 12 12 x(t ) = e 2t + e3t 5 5

(c)

x(t) is always positive.


For t > 0, as t increases, the first term decreases toward 0 and the second term increases ever more rapidly, in agreement with the tx-graph. For t > 0, e3t > e2t, so x(t ) is positive, and x(t ) increases as t increases, as shown on the

(d)

tx graph.
For t < 0, the first term will dominate and x(t ) will be negative, ever more so as t becomes more negative, in agreement with the tx -graph.

38.

(a)

(b)

From (1), From (2),

c1 + c2 = 3 2c1 + 3c2 = 0

9 6 c1 = ; c2 = 5 5

9 6 x(t) = e 2t e3t 5 5 18 2t 18 3t x(t ) = + e e 5 5 (c)

x(t) is always negative, with a maximum at t = 0. (See part (d) and set x(t ) = 0.)
These facts agree with the tx graph. For t > 0, e3t > e2t so the negative term dominates in x(t ) and x(t ) is negative, ever more so as t increases. For t < 0, e2t > e3t so the positive term dominates in x(t ) and x(t ) is positive, ever more so as t becomes more negative. These facts agree with the tx graph.

(d)

SECTION 4.2

Real Characteristic Roots

329

39.

(a)

(b)

From (1), From (2),

c1 + c2 = 0 2c1 + 3c2 = 1 c1 =

1 1 ; c2 = 5 5

x(t) =

1 2t 1 3t e e 5 5 2 3 x(t ) = e 2t e3t 5 5

(c)

For t > 0 the second term dominates, so x(t) is negative, ever more so as t increases. For t < 0 the first term dominates, so x(t) is positive, ever more so as t becomes more negative. These facts agree with the tx graph.

(d)

x(t ) is always negative. The maximum value will occur when t = 0, as shown on the tx graph.

Phase Portraits
Careful inspection shows:

40.

(B)

41.

(D)

42.

(A)

43.

(C)

330

CHAPTER 4

Higher-Order Linear Differential Equations

Independent Solutions 44.


Letting

c1e r1t + c2 e r2t = 0


for all t, then by setting t = 0 and t = 1 we have, respectively
c1 c1e
r1

+ c2 + c2 e
r2

= 0 = 0.

When r1 r2 then these equations have the unique solution c1 = c2 = 0 , which shows the given functions e r1t , e r2t are linearly independent for r1 r2 .

Second Solution 45.


Substituting y = v ( t ) ebt 2 a into

ay + by + cy = 0
gives

y = vebt 2 a y = ve bt 2 a

b bt 2 a ve 2a b b2 vebt 2 a + 2 ve bt 2 a . a 4a

Substituting v, v, v into the differential equation gives the new equation (after dividing by

e bt 2 a ) b b2 b a v v + 2 v + b v v + cv = 0 . a 2a 4a
Simplifying gives

b2 av cv = 0 . 4a
Because we have assumed b 2 = 4ac , we have the equation v = 0 , which was the condition to be

proven.

Independence Again 46.


Setting

c1e bt 2 a + c2te bt 2 a = 0
for all t, we set in particular t = 0 and then t = 1 . These yield, respectively, the equations

c1 = 0 c1e
b 2 a

+ c2 e

b 2 a

=0

which have the unique solution c1 = c2 = 0 . Hence, the given functions are linearly independent.

SECTION 4.2

Real Characteristic Roots

331

Repeated Roots, Long-Term Behavior 47.


Because e bt 2 a approaches 0 as t (for a, b > 0 ), we know the first term tends toward zero. For the second term we need only verify that

tebt 2 a =

t e
bt 2 a

does as well. To use lHpitals rule, we compute the derivatives of both the numerator and denominator of the previous expression, getting
b 2a

1 , ebt 2 a

which clearly approaches 0 as t . Then lHpitals rule assures us that the given expression
b 2a t te ( ) approaches 0 as well.

Negative Roots 48.


We have r = b b 2 4mk , so in the overdamped case where b 2 4mk > 0 , these

characteristic roots are real. Because m and k are both nonnegative, b 2 4mk < b 2 causing
r1 = b + b 2 4mk to be a negative sum of negative and positive terms

and
r2 = b b 2 4mk to be a negative sum of two negative terms.
Circuits and Springs 49.

(a)

The LRC equation is LQ + RQ + hold:

1 Q = 0 , hence the following discriminant conditions C

L <0 C L = R2 4 = 0 C L = R2 4 > 0 C
= R2 4

( underdamped ) ( critically damped ) ( overdamped ) .

332

CHAPTER 4

Higher-Order Linear Differential Equations

(b)

The conditions in part (a) can be written


R<2 R=2 R>2 L C L C L C

( underdamped ) ( critically damped ) ( overdamped ) .


1 . C

These correspond to the analogy that m, b, and k correspond respectively to L, R, and (see Table 4.1.3 in the textbook.)
A Test of Your Intuition 50.

Intuitively, a curve whose rate of increase is proportional to its height will increase very rapidly as the height increases. On the other hand, upward curvature doesnt necessarily imply that the function is increasing! (The curve e t has upward curvature, yet decreases to 0 as t .) In this case, the restriction that y ( 0 ) = 0 will cause the second curve to increase, but probably not nearly as rapidly as the first curve. Solving the equations, the IVP y = y , y ( 0 ) = 1 has the solution y = et , whereas the second curve described by y = y , y ( 0 ) = 1 , y ( 0 ) = 0 has the solution
y (t ) = 1 t 1 t e + e . 2 2

The first curve is indeed above the second curve.


An Overdamped Spring 51.

(a)

The solution of an overdamped equation has the form


x ( t ) = c1e r1t + c2 er2t .

Suppose that
c1er1t1 + c2 er2t1 = 0
r r t for some t1 . Because e r2t1 is never zero, we can divide by e r2t1 to get c1e( 1 2 ) 1 + c2 = 0 .

Solving for t1 gives


t1 =
c 1 ln 2 . r1 r2 c1

This unique number is the only value for which the curve may pass through 0. If the argument of the logarithm is negative or if the value of t1 is negative, then the solution does not cross the equilibrium point. (b) By a similar argument, we can show that the derivative x ( t ) also has one zero.

SECTION 4.2

Real Characteristic Roots

333

A Critically Damped Spring 52.

(a)

Suppose

( c1 + c2t1 ) er t

11

=0.

We can divide by the nonnegative quantity er1t1 getting the equation c1 + c2t1 = 0 , which c has the unique solution t1 = 1 . Hence, the solution of a critically damped equation can c2 pass through the equilibrium at most once. If the value of t1 is negative, then the solution does not cross the equilibrium point. (b) By a similar argument, we can show that the derivative x ( t ) has one zero.

Linking Graphs

After inspection, we have labeled the yt and y t graphs as follows.


53.
y 2 3 t 2 3 t y 1 5 y' 3 1 1 2 5 y' 3

5 4

t=0 5

54.

334

CHAPTER 4

Higher-Order Linear Differential Equations

55.

Damped Vibration 56.

The IVP problem is

x + 2 x + x = 0 , x ( 0 ) = 3 in =
The solution is

1 ft , x ( 0 ) = 0 ft sec . 4

1 1 x ( t ) = e t + tet . 4 4 This is zero only for t1 = 1 , whereas the physical system does not start before t = 0 .
Surge Functions 57.

For mx + bx + kx = 0 , let m = 1, find b, k and initial conditions for the solution x = Atert
x + bx + kx = 0 r 2 + br + k = 0 (characteristic equation) r=
b b 2 4 1 k 2

b b b2 4k = 0 to obtain repeated roots, r = , 2 2

x = c1ert + c2tert x = rc1e rt + c2 (t ( r )e rt + e rt )


c1 = 0 = x(0)

c2 = A = x(0) x(0) = c2
b = 2r, 4k = b2

and from above we know 4k = 4r2 so that k = r, for k > 0

SECTION 4.2

Real Characteristic Roots

335

Results: r and A are given, and

b = 2r k=r x(0) = 0 x(0) = A


LRC-Circuit I

58.

(a) (b) (d)

LQ + RQ +

1 Q = 2Q + 101Q + 50Q = 0 , Q ( 0 ) = 99 , Q ( 0 ) = 0 C (c)


I ( t ) = Q ( t ) = 50e 50t 50e t 2

Q ( t ) = e 50t + 100e t 2

As t , Q ( t ) 0 and I ( t ) 0

LRC-Circuit II

59.

(a) (b) (d)

LQ + RQ +

1 Q = Q + 15Q + 50Q = 0 , Q ( 0 ) = 5 , Q ( 0 ) = 0 C (c)


I ( t ) = Q ( t ) = 50e 5t + 50e 10t

Q ( t ) = 10e 5t 5e10t

As t , Q ( t ) 0 and I ( t ) 0

The Euler-Cauchy Equation at 2 y + bty + cy = 0 60.

Let y ( t ) = t r , so
y = rt r 1 y = r ( r 1) t r 2 .

Hence
at 2 y + bty + cy = ar ( r 1) t r + brt r + ct r = 0 .

Dividing by t r yields the characteristic equation


ar ( r 1) + br + c = 0 ,

which can be written as

ar 2 + ( b a ) r + c = 0 .
If r1 and r2 are two distinct roots of this equation, we have solutions
y1 ( t ) = t r1 y2 ( t ) = t r2 .

336

CHAPTER 4

Higher-Order Linear Differential Equations

Because these two functions are clearly linearly independent (one not a constant multiple of the other) for r1 r2 , we have
y ( t ) = c1t r1 + c2t r2

for t > 0 .
The Euler-Cauchy Equation with Distinct Roots

For Problems 6165, see Problem 60 for the form of the characteristic equation for the Euler-Cauchy DE.
61.

t 2 y + 2ty 12 y = 0

In this case a = 1 , b = 2 , c = 12 , so the characteristic equation is

r ( r 1) + 2r 12 = r 2 + r 12 = ( r + 4 )( r 3) = 0 .
Hence, we have roots r1 = 4 , r2 = 3 , and thus
y ( t ) = c1t 3 + c2t 4 .
62.

4t 2 y + 8ty 3 y = 0 In this case a = 4 , b = 8 , c = 3 , so the characteristic equation is

4r ( r 1) + 8r 3 = 4r 2 + 4r 3 = ( 2r 1)( 2r + 3) = 0 .
Hence, we have roots r1 =
1 3 , r2 = , and thus 2 2

y ( t ) = c1t1 2 + c2t 3 2 .
63.

t 2 y + 4ty + 2 y = 0

In this case a = 1 , b = 4 , c = 2 , so the characteristic equation is


r ( r 1) + 4r + 2 = r 2 + 3r + 2 = ( r + 1)( r + 2 ) = 0 .

Hence, we have roots r1 = 1 , r2 = 2 , and thus


y ( t ) = c1t 1 + c2t 2 .

SECTION 4.2

Real Characteristic Roots

337

64.

2t 2 y + 3ty y = 0 In this case a = 2 , b = 3 , c = 1 , so the characteristic equation is 2r ( r 1) + 3r 1 = 2r 2 + r 1 = ( 2r 1)( r + 1) = 0 . Hence, we have roots r1 = 1 , r2 = 1 , and thus 2
y ( t ) = c1t1 2 + c2t 1 .
Repeated Euler-Cauchy Roots

65.

We are given that the characteristic equation

ar 2 + ( b a ) r + c = 0
of Eulers equation
at 2 y + bty + cy = 0

has a double root of r. Hence, we have one solution y1 = t r . To verify that t r ln t is also a solution, we differentiate y = rt r 1 ln t + t r 1 ,

y = r ( r 1) t r 2 ln t + rt r 2 + ( r 1) t r 2 = r ( r 1) t r 2 ln t + ( 2r 1) t r 2 .
By direct substitution we have
at 2 y + bty + cy = at 2 r ( r 1) t r 2 ln t + ( 2r 1) t r 2 + bt rt r 1 ln t + t r 1 + ct r ln t
= ar ( r 1) + br + c t r ln t + a ( 2r 1) + b t r .

We know that ar ( r 1) + br + c = 0 , so this last expression becomes simply


at 2 y + bty + cy = a ( 2r 1) + b t r .

Thus the root of the characteristic equation is r =

ba , which makes this expression zero. 2a ba is the double 2a

To verify that t r and t r ln t are linearly independent (where r = root of the characteristic equation), we set
c1t r + c2t r ln t = 0

for specific values t = 1 and 2, which give, respectively, the equations


c1 = 0 c1 2r + c2 2r ln 2 = 0

and yields the unique solution c1 = c2 = 0 . Hence, t r and t r ln t are linearly independent solutions.

338

CHAPTER 4

Higher-Order Linear Differential Equations

Solutions for Repeated Euler-Cauchy Roots

For Problems 66 and 67 use the result of Problem 60, y ( t ) = c1t r + c2t r ln t .
66.
t 2 y + 5ty + 4 y = 0

In this case, a = 1 , b = 5 , and c = 4 , so our characteristic equation for r is r 2 + 4r + 4 = 0 , with a double root at 2. The general solution is

y ( t ) = c1t 2 + c2t 2 ln t
for t > 0 .
67.
t 2 y 3ty + 4 y = 0

In this case, a = 1 , b = 3 , and c = 4 , so our characteristic equation for r is r 2 4r + 4 = 0 , with a double root at 2. The general solution is
y ( t ) = c1t 2 + c2t 2 ln t

for t > 0 .
68.

9t2y + 3ty + y = 0 9m(m 1) + 3m + 1 = 0 9m 6m + 1 = 0


2

Euler-Cauchy method: y = t m, t > 0 (characteristic equation) 1 3

(3m 1)2 = 0
y (t ) = c1t1/ 3 + c2t1/ 3 ln t

m=

69.

4t 2 y + 8ty + y = 0 4m(m 1) + 8m + 1 = 0 4m2 + 4m + 1 = 0 (2m + 1)2 = 0

Euler-Cauchy method: y = t m, t > 0

m=

1 2

y (t ) = c1t 1/ 2 + c2t 1/ 2 ln t

Computer: Phase-Plane Trajectories 70.

(a)

y ( t ) = 2e t + e 3t
The roots of the characteristic equation are 1 and 3, so the characteristic equation is

( r + 1)( r + 3) = r 2 + 4r + 3 = 0 .
y ( t ) satisfies the differential equation y + 4 y + 3 y = 0 .

SECTION 4.2

Real Characteristic Roots

339

(b)

To find the IC for the trajectory of y ( t ) in yy space we differentiate getting


y ( t ) = 2e t 3e 3t y (t ) ,

The IC of the given trajectory of

( y ( t ) , y ( t ) )
in yy space is ( y ( 0 ) , y ( 0 ) ) = ( 3, 5 ) . (c) We plot the trajectory starting at ( 3, 5) along with a few other trajectories in yy space.
71.
y ( t ) = e t + e 8 t

DE trajectories in yy space

(a)

The roots of the characteristic equation are 1 and 8, so the characteristic equation is

( r + 1)( r + 8) = r 2 + 9r + 8 = 0 .
y ( t ) satisfies the differential equation y + 9 y + 8 y = 0 .

(b)

The derivative is

y ( t ) = e t 8e8t .
The IC for the given trajectory in yy space is

( y ( 0 ) , y ( 0 ) ) = ( 2,
(c)

9) .

We plot this trajectory in yy space.

DE trajectory in yy space
72.
y ( t ) = et + e t

(a)

The roots of the characteristic equation are 1 and 1, so the characteristic equation is

( r 1)( r + 1) = r 2 1 = 0 .
y ( t ) satisfies the differential equation

y y = 0 .

340

CHAPTER 4

Higher-Order Linear Differential Equations

(b)

The derivative is
y ( t ) = et e t .

4 2 (2, 0) 2 4 2 y

The IC for the given trajectory in yy space is

( y ( 0 ) , y ( 0 ) ) = ( 2, 0 ) .
(c) We plot this and a few other trajectories of this DE in yy space.

DE trajectories in yy space
73.
y ( t ) = e t + te t

(a)

The characteristic equation has a double root at 1, so the characteristic equation is

( r + 1)2 = r 2 + 2r + 1 = 0 .
y ( t ) satisfies the differential equation

y + 2 y + y = 0 .
(b) The derivative is
y ( t ) = te t .
1.5

y'

The IC for the given trajectory in yy space is


y ( 0 ) = 1, y ( 0 ) = 0 .
1.5

(1, 0) y 1
1.5

(c)

See the figure to the right.

1.5

DE trajectory in yy space

74.

y ( t ) = 3 + 2e 2 t
(a) The roots of the characteristic equation are 0 and 2, so the characteristic equation is
r ( r 2 ) = r 2 2r = 0 . y ( t ) satisfies the differential equation y 2 y = 0 .

SECTION 4.2

Real Characteristic Roots

341

(b)

The derivative is
y ( t ) = 4e 2 t .

The IC for the given trajectory in yy space is

( y ( 0 ) , y ( 0 ) ) = ( 5, 4 ) .
(c) See the figure to the right. DE trajectories in yy space
Reduction of Order 75.

(a)

Let y2 = vy1 and

y2 = vy1 + vy1 y2 = vy1 + 2vy1 + vy1 .

Then

y2 + p ( x ) y2 + q ( x ) y2 = vy1 + 2vy1 + pvy1 + ( vy1 + pvy1 + qvy1 ) = 0 .

Because y1 + py1 + qy1 = 0 , cancel the terms involving v, and arrive at the new equation y1v + ( 2 y1 + p ( x ) y1 ) v = 0

(b)

Setting v = w and using the fact that y1dx = dy1 , we obtain


y1w + ( 2 y1 + p ( x ) y1 ) w = 0 2 y + p ( x ) y1 w + 1 w = 0 y1 dw 2 y1 p ( x ) y1 = dx w y1 2 y1 p ( x ) dx ln w = y 2 ln w = dy1 p ( x ) dx y1

ln w = ln y12 p ( x ) dx

p ( x ) dx e = v w= y12 p ( x ) dx e v= y12

By convention, the positive sign is chosen.

342

CHAPTER 4

Higher-Order Linear Differential Equations

(c)

If v is a constant function on I, then v 0 and w 0 because v = w . The condition w 0 contradicts our work in part (b) as ln w where w = 0 is undefined. Because v is not constant on I, { y1 , y2 } is a linearly independent set of I.

Reduction of Order: Second Solution 76.


y 6 y + 9 y = 0 , y1 = e3t

We identify p ( t ) = 6 , so

p (t ) dt = 6t .
Substituting in the formula developed in Problem 75, we have
y2 = y1 y 4 y + 4 y = 0 , y1 = e 2t

p ( t ) dt e dt = e3t y12 ( t )

(e )
3t

e6t

dt = te3t .

77.

We wont use the formula this time. We simply redo the steps in Problem 75. We seek a second solution of the form y2 = vy1 = ve 2t . Differentiating, we have
y2 = ve 2t + 2ve 2t y2 = ve 2t + 4ve 2t + 4ve 2t .

Substituting into the equation we obtain


y2 4 y2 + 4 y2 = ve 2t = 0 .

Dividing by e 2t gives v = 0 or
v ( t ) = c1t + c2 .

Hence, we have found new solutions


y2 = ve2t = c1te 2t + c2 e2t .

Because y1 = e 2t , we let c1 = 1 , c2 = 0 , yielding a second independent solution


y2 = te2t .

SECTION 4.2

Real Characteristic Roots

343

78.

t 2 y ty + y = 0 , y1 = t

We wont use the formula this time. We simply redo the steps in Problem 75. We seek a second solution of the form y2 = vy1 = tv . Differentiating, we have
y2 = tv + v y2 = tv + 2v .

Substituting into the equation we obtain


t 2 y2 ty2 + y2 = t 3v + t 2 v = 0 .

Letting w = v and dividing by t 3 yields 1 w + w = 0 . t We can solve by integrating the factor method, getting w = c1t 1 . Integrating we find
v = c1 ln t + c2 ,

so
y2 = tv = c1t ln t + c2t .

Letting c1 = 1 , c2 = 0 , we get a second linearly independent solution


y2 = t ln t .
79.

(t

+ 1 y 2ty + 2 y = 0 , y1 = t

We wont use the formula this time. We simply redo the steps in Problem 75. We seek a second solution of the form y2 = vy1 = tv . Differentiating yields
y2 = tv + v y2 = tv + 2v .

Substituting into the equation we get

(t
factor method, getting

+ 1 y2 2ty2 + 2 y2 = t t 2 + 1 v + 2v = 0 .

Letting w = v and dividing by t t 2 + 1 , we can solve the new equation using the integrating

t (t

2
2

+1

dt = ln t 2 + 1 + 2ln t = ln

t2 . t2 +1

344

CHAPTER 4

Higher-Order Linear Differential Equations

We arrive at
w = c1 t2 +1 = c1 + c1t 2 . 2 t

Integrating this, we get


v = c1 t t 1 + c2 ,

so
y2 = tv = c1 t 2 1 + c2t .

Letting c1 = 1 , c2 = 0 we get a second linearly independent solution


y2 = t 2 1 .
Classical Equations 80.

y 2ty + 4 y = 0 , y1 ( t ) = 1 2t 2 (Hermites Equation)

Letting y2 = vy1 = v 1 2t 2 , we have

( ) y = (1 2t ) v 8tv 4v
y2 = 1 2t 2 v 4tv
2 2

and perform the long division, yielding the equation 8t v + 2t + 2 v = 0 . 2t 1 Letting w = v and solving the first-order equation in w, we get
w = c1et 2t 2 1
2

To find y2 we simply let c1 = 1 and integrate to get


v = et 2t 2 1

(
2

dt .

Multiplying by 1 2t 2 yields a final answer of


y2 ( t ) = 1 2t 2
81.

) e ( 2t
t2

dt .

(1 t ) y ty + y = 0 , y (t ) = t
2 1

(Chebyshevs Equation)

Letting y2 = vy1 = vt , we have


y2 = tv + v , y2 = tv + 2v ,

SECTION 4.2

Real Characteristic Roots

345

hence we have the equation

(1 t ) y ty + y
2 2 2

= t 1 t 2 v + 2 3t 2 v = 0 .

Dividing by t 1 t 2 , and letting w = v ,


w + 2 3t 2 t 1 t2

w=0.

Using partial fractions yields

t (1 t ) dt = 2ln t + 2 ln t 1 + 2 ln 1 + t ,
2

2 3t 2

so our integrating factor is t 2 1 t 2 and


w = c1

1
t
2

1 t2

Letting c1 = 1 and multiplying by t yields a final answer of


y2 ( t ) = tv = t

1
2

1 t2

dt .

This is a perfect example of a formula that does not tell us much about how the solutions behave. Check out the IDE tool Chebyshevs Equation to see the value of graphical solutions.
82.

ty + (1 t ) y + y = 0 , y1 ( t ) = t 1 (Laguerres Equation)

Letting y2 = vy1 = v ( t 1) , we have


y2 = v ( t 1) + v , y2 = v ( t 1) + 2v ,

hence we have the equation


ty2 + (1 t ) y2 + y = t ( t 1) v + t 2 + 4t 1 v = 0 .

Dividing by t ( t 1) and letting w = v yields


w + t 2 + 4t 1 w = 0. t ( t 1)

Hence by use of partial fractions, our integrating factor is u = e


w = C1 et . t (t 1) 2

1+ t + t 1 dt so that

Letting c1 = 1 and multiplying by t 1 yields a final answer of


y2 ( t ) = v ( t 1) = ( t 1)

t ( t 1)

et

dt .

346

CHAPTER 4

Higher-Order Linear Differential Equations

Lagranges Adjoint Equation 83.

(a)(b) Differentiating the right side of

(t ) [ y + y + y ] =
we obtain

d [ (t ) y + g (t ) y ] dt

y + y + y = y + y + g y + gy
Setting the coefficients of y, y , y equal, we find for y : = (no information) for y : = + g for y : = g The last equation yields g =

dt and substituting this into the second equation, and

differentiating, gives a differential equation for the integrating factor

+ = 0.
(c) We perform the differentiation on the right-hand-side of the given equation, yielding

(t ) [ y + p (t ) y + q(t ) y ] = y + y + g (t ) y + g (t ) y.
Multiplying out the left-hand side and subtracting yields

[ p(t ) g (t )] y + [ q(t ) g (t )] y = 0.
Setting the first set of coefficients equal to 0 yields = p g , hence = p + p g , so that g = + p + p. The second set of coefficients yields q g = 0 so that g = q. Setting these two equations for g equal to each other yields p + (q p) = 0 which was to be shown.
Suggested Journal Entry 84.

Student Project

SECTION 4.3

Complex Characteristic Roots

347

4.3

Complex Characteristic Roots

Solutions in General 1.

y + 9 y = 0 The characteristic equation is r 2 + 9 = 0 , which has roots 3i, 3i. The general solution is
y ( t ) = c1 cos3t + c2 sin 3t .

2.

y + y + y = 0

1 3 i . The general solution is The characteristic equation is r 2 + r + 1 = 0 , which has roots 2 2


3 3 y ( t ) = e t 2 c1 cos t + c2 sin t. 2 2
3.

y 4 y + 5 y = 0 The characteristic equation is r 2 4r + 5 = 0 , which has roots 2 i . The general solution is

y ( t ) = e 2t ( c1 cos t + c2 sin t ) .
4.

y + 2 y + 8 y = 0 The characteristic equation is r 2 + 2r + 8 = 0 , which has roots 1 i 7 . The general solution is


y ( t ) = e t c1 cos 7t + c2 sin 7t .

5.

y + 2 y + 4 y = 0 The characteristic equation is r 2 + 2r + 4 = 0 , which has roots 1 i 3 . The general solution is


y ( t ) = e t c1 cos 3t + c2 sin 3t .

6.

y 4 y + 7 y = 0
The characteristic equation is r 2 4r + 7 = 0 , which has roots 2 i 3 . The general solution is
y ( t ) = e 2t c1 cos 3t + c2 sin 3t .

7.

y 10 y + 26 y = 0 The characteristic equation is r 2 10r + 26 = 0 , which has roots 5 + i . The general solution is

y ( t ) = e5t ( c1 cos t + c2 sin t ) .

348

CHAPTER 4

Higher-Order Linear Differential Equations

8.

3 y + 4 y + 9 y = 0

2 23 . The general solution The characteristic equation is 3r 2 + 4r + 9 = 0 , which has roots i 3 3 is

23 23 y ( t ) = e 2t 3 c1 cos t + c2 sin t. 3 3
9.

y y + y = 0
The characteristic equation is r 2 r + 1 = 0 , which has roots 1 3 i . The general solution is 2 2

3 3 y ( t ) = et 2 c1 cos t + c2 sin t . 2 2
10.

y + y + 2 y = 0
1 7 The characteristic equation is r 2 + r + 2 = 0 , which has roots i . The general solution is 2 2 7 7 y ( t ) = e t 2 c1 cos t + c2 sin t. 2 2
Initial-Value Problems

11.

y + 4 y = 0 , y ( 0 ) = 1 , y ( 0 ) = 1
The characteristic equation is r 2 + 4 = 0 , which has roots 2i . The general solution is y ( t ) = c1 cos 2t + c2 sin 2t . Substituting this into the initial conditions gives y ( 0 ) = c1 = 1 , y ( 0 ) = 2c2 = 1 . Hence, the solution of the initial-value problem is
1 y ( t ) = cos 2t sin 2t . 2

12.

y 4 y + 13 y = 0 , y ( 0 ) = 1 , y ( 0 ) = 0 The characteristic equation is r 2 4r + 13 = 0 , which has roots 2 3i . The general solution is y ( t ) = e 2t ( c1 cos3t + c2 sin 3t ) . Substituting this into the initial conditions yields y ( 0 ) = c1 = 1 , y ( 0 ) = 2c1 + 3c2 = 0 , resulting in
2 c1 = 1 , c2 = . Hence, the solution of the initial-value problem is 3

2 y ( t ) = e 2t cos3t sin 3t . 3

SECTION 4.3

Complex Characteristic Roots

349

13.

y + 2 y + 2 y = 0 , y ( 0 ) = 1 , y ( 0 ) = 0
The characteristic equation is r 2 + 2r + 2 = 0 , which has roots 1 i . Hence, the general solution is y ( t ) = e t ( c1 cos t + c2 sin t ) . Substituting this into the initial conditions yields y ( 0 ) = c1 = 1 , y ( 0 ) = c1 c2 = 0 , resulting in c1 = 1 , c2 = 1 . Hence, the solution of the initial-value problem is

y ( t ) = e t ( cos t + sin t ) .
14.

y y + y = 0 , y ( 0 ) = 0 , y ( 0 ) = 1 From Problem 6, 3 3 y ( t ) = et 2 c1 cos ( t ) + c2 sin ( t ) . 2 2 Substituting this into the initial conditions yields y ( 0 ) = 0 , y ( 0 ) = 1 , resulting in c1 = 0 ,
c2 = 2 3 . Hence, the solution of the initial-value problem is 3

y (t ) =
15.

3 2 t . 3e t 2 sin 2 3

y 4 y + 7 y = 0 , y ( 0 ) = 0 , y ( 0 ) = 1 From Problem 6,
y ( t ) = e 2t c1 cos

( 3t ) + c sin ( 3t )} .
2

Subsituting this into the initial conditions yields y ( 0 ) = 0 , y ( 0 ) = 1 , resulting in

c1 = 0 , c2 =
Hence, the solution of the initial-value problem is
y (t ) =

1 3. 3

1 3e 2t sin 3

( 3t ) .

16.

y + 2 y + 5 y = 0 , y ( 0 ) = 1 , y ( 0 ) = 1
The characteristic equation is r 2 + 2r + 5 = 0 , which has roots 1 2i . Hence, the general solution is

y ( t ) = e t ( c1 cos 2t + c2 sin 2t ) .

350

CHAPTER 4

Higher-Order Linear Differential Equations

Subsituting this into the initial conditions yields y ( 0 ) = 1 , y ( 0 ) = 1 , resulting in c1 = 1 , c2 = 0 . Hence, the solution of the initial-value problem is y ( t ) = e t cos 2t .
Working Backwards 17.

(r 1)3 = r 3 3r 2 + 3r 1 y 3 y + 3 y y = 0

18.

(r 4)( r (1 i ))(r (1 + i )) = r 3 6r 2 + 10r 8 = 0 y 6 y + 10 y 8 y = 0

19.

(r 2)( r (2 + i ))(r (2 i )) = r 3 6r 2 + 13r 10

y 6 y + 13 y 10 y = 0
20.

(r 4)(r 2 (2 + i ))(r (2 i )) = r 4 4r 3 + r 2 + 16r 20 = 0 y (4) 6 y + y + 16 y 20 y = 0


Matching Problems

21.

y y = 0 r = 0, 1 y(t) = c1 + c2et Graph D

22.

y + y = 0 r = 0, 1 y(t) = c1 + c2et Graph B

23.

y + 3 y + 2 y = 0 r = 2, 1 y(t) = c1e2t + c2et Graph A

24.

y 5 y + 6 y = 0 r = 2, 3 y(t) = c1e2t + c2e3t Graph C


1 3i 2

25.

y + y + y = 0 r =
1 t 2

y(t) = e
26.

3t 3t + c2 sin c1 cos 2 2

Graph G

y + y = 0 r = i y(t) = c1 cos t + c2 sin t Graph F

27.

y + 4 y + 4 y = 0 r = 2, 2 y (t ) = (c1 + c2t )e 2t Graph E

SECTION 4.3

Complex Characteristic Roots

351

28.

y y + y = 0 r =
1

1 3i 2 Graph H

t 3 3 = e 2 c1 cos y (t ) t + c2 sin t 2 2

Eulers Formula 29.

(a)

The Maclaurin series for e x is ex = 1 + x + 1 2 1 3 x + x + 2! 3! 1 ( i )n + n!


+

1 n x + n!

(b) (c)

ei = 1 + i +

1 1 ( i )2 + ( i )3 + 2! 3!

Using the given identities for i, we can write

ei = 1 + i +

1 1 1 ( i )2 + ( i )3 + + ( i )n + n! 2! 3! 1 1 1 1 = 1 2 + 4 + + i 3 + 5 4! 3! 5! 2! (e)

= cos + i sin

(d)

Done in part (c).

Done in part (c).

Long-Term Behavior of Solutions 30.

r1 < 0 , r2 < 0 . When r1 r2 , the solution is

y ( t ) = c1e r1t + c2 er2t


and goes to 0 as t . When r = r1 = r2 < 0 , the solution has the form y ( t ) = c1e rt + c2te rt . In this case using lHpitals rule we prove the second term te rt goes to zero as t when r<0.
31.

r1 < 0 , r2 = 0 . The solution y ( t ) = c1e r1t + c2 approaches the constant c2 as t because r1 < 0 .

32.

r = i ,

y ( t ) = e t ( c1 cos t + c2 sin t ) . For

the solution y (t ) oscillates with

decreasing amplitude when < 0 ; oscillates with increasing amplitude when > 0 ; oscillates with constant amplitude when = 0 .

352

CHAPTER 4

Higher-Order Linear Differential Equations

33.

r1 = 0 , r2 = 0 . The solution y ( t ) = c1 + c2t approaches as t when c2 > 0 and when c2 < 0 .

34.

r1 > 0 , r2 < 0 . The solution y ( t ) = c1e r1t + c2 e r2t approaches as t when c1 > 0 and when c1 < 0 .

35.

r = i , y ( t ) = c1 cos t + c2 sin t is a periodic function of period


2 c12 + c2

, and amplitude

Linear Independence 36.

Suppose c1e t cos t + c2 e t sin t = 0 on an arbitrary interval. Dividing both sides by e t , then differentiating the new equation and dividing by , yields c1 cos t + c2 sin t = 0 c2 cos t c1 sin t = 0. Hence, c1 = 0, c2 = 0 and we have proven linear independence of the given functions.
Real Coefficients

37.

Solution of the differential equation is y ( t ) = k1e t ( cos t + i sin t ) + k2 e t ( cos t i sin t ) = e t ( k1 + k2 ) cos t + ie t ( k1 k2 ) sin t . For the solution to be real, there must exist real numbers r and s such that k1 + k2 = r k1 k2 = si Solving for k1 and k2 , we get
1 1 r + si 2 2 1 1 k2 = r si . 2 2 k1 =

SECTION 4.3

Complex Characteristic Roots

353

Solving

dn y =0 dt n

38.

(a)

d4y dt 4 d3y dt 3 d2y dt 2 dy dt

=0 = k3 = k 3t + k 2 =

(b)

4 y ( ) = 0 . The characteristic equation is

r 4 = 0 , which has a fourth-order root at 0. Hence, the solution is

y ( t ) = c0 + c1t + c2t 2 + c3t 3 ,


which is the same as in part (a).

1 2 k3t + k2t + k1 2 1 1 y = k3t 3 + k2t 2 + k1t + k0 3! 2

(c)

In general we have y ( t ) = kn 1 1 1 t n 1 + kn 2 t n2 + ( n 1)! ( n 2 )! + c1t + c0 + k1t + k0

= cn 1t n 1 + cn 2t n 2 + because all of the constants are arbitrary.


Higher-Order DEs 39.
d 5 y 4 d 4 y 4d 3 y + =0 dt 5 dt 4 dt 3

The characteristic equation is

r 5 4r 4 + 4r 3 = r 3 r 2 4r + 4 = r 3 ( r 2 ) = 0 ,
2

which has roots, 0, 0, 0, 2, 2. Hence, y ( t ) = c1 + c2t + c3t 2 + c4 e 2t + c5te 2t .


40.

d 3 y 4d 2 y 7 dy + 10 y = 0 dt dt 3 dt 2 The characteristic equation is

r 3 + 4r 2 7 r 10 = 0 ,
which has roots, 1, 2, 5. Hence, y ( t ) = c1et + c2 e 2t + c3e 5t .

354

CHAPTER 4

Higher-Order Linear Differential Equations

41.

d 5 y dy =0 dt 5 dt The characteristic equation is

r 5 r = r r 4 1 = r r 2 1 r 2 + 1 = r ( r 1)( r + 1) r 2 + 1 = 0 , which has roots, 0, 1, i. Hence y ( t ) = c1 + c2 et + c3et + c4 cos t + c5 sin t .


42.

) (

)(

y 4 y + 5 y 2 y = 0 r 3 4 r 2 + 5r 2 = 0 (characteristic equation) f (1) = 1 4 + 5 2 = 0 By long division, we obtain r = 1 is a root

(r 1)(r 2 3r + 2) = 0 (r 1)(r 2)(r 1) = 0 y (t ) = c1et + c2tet + c3e2t r = 1, 1, 2

43.

y + 6 y + 12 y + 8 y = 0 r 3 + 6r 2 + 12r + 8 = 0 (characteristic equation) f ( 2) = 8 + 24 24 + 8 = 0 By long division, we obtain r = 2 is a root


(r + 2)(r 2 + 4r + 4) = 0 (r + 2)3 = 0 y (t ) = c1e
2 t

r = 2, 2, 2
2 2 t

+ c2te

2 t

+ c3t e

44.

(4)

y=0

r 1 = 0 (characteristic equation) 2 2 (r + 1)(r 1) = 0 r = i , 1


4

y (t ) = c1 cos t + c2 sin t + c3et + c4 e t


Linking Graphs 45.
5 1 3 2 5 5 5 t=0 t y 3 5 y' 3 1 1 2 t 5 1 2 3 5 y 5 y'

SECTION 4.3

Complex Characteristic Roots

355

46.
5

y 3 2 3 1 5 5 t 5

y' 5 1 t 2 3

y' 3 2 y

5 1 5 t=0

Changing the Damping 47.

The curves below show the solution of

x + bx + x = 0 , x ( 0 ) = 4 , x ( 0 ) = 0
for damping b = 0 , 0.5, 1, 2, 4. The larger the damping the faster the curves approach 0. The curve that oscillates has no damping ( b = 0 ) .
x(t ) b = 4 4 2 t x b=0 b = 0.5 4 2 b = 0.5 4 b=0 8 12 16 4 2 2 4 b=1 4 2

b=1 b=2

b=2 2

b=4 4 x

In Figure 4.3.12 (b) in the text the larger the damping b the more directly the trajectory heads for the origin. The trajectory that forms a circle corresponds to zero damping. Note that every time a curve in (a) crosses the axis twice the corresponding trajectory in (b) circles the origin.

356

CHAPTER 4

Higher-Order Linear Differential Equations

Changing the Spring 48.

(a)

The solutions of x + x + kx = 0 , x ( 0 ) = 4 , x ( 0 ) = 0 are shown for k = 1 1 , , 1, 2, 4. For 4 2

x(t ) = 4 k 1 k = 0.5 2 k = 0.25 4 k =2 k=4 8 12 16 t

larger k we have more oscillations.


2 4

(b)

For larger k, since there are more oscillations, the phase-plane trajectory spirals further around the origin.
4 k =2 4 2 2 4 k=4 2

k = 0.25 2 x k = 0.5 k =1

Changing the Mass 49.

(a) (b) (c)

b = 0 and o =

k so that o is inversely proportional to m

m.

If m is doubled, o is decreased by a factor of

1 2

. 2.

If m is doubled, the damping required for critical damping is increased by a factor of x + 2 x + 3x = 0 , x(0) = 1, x(0) = 0 r2 + 2r + 3 = 0 (characteristic equation) r = 1 2i

Finding the Maximum 50.

(a)

x = e t c1 cos 2t + c2 sin 2t x = et 1 = c1 0= 2c2 1 so that c2 = 1 2

( (

2c1 sin 2t + 2c2 cos 2t e t c1 cos 2t + c2 sin 2t

1 sin 2t x = e t cos 2t + 2

SECTION 4.3

Complex Characteristic Roots

357

To find maximum displacement, set x = 0 . t 1 1 sin 2t = 0 x = e t 2 sin 2t + 2 cos 2t e cos 2t + 2 2 2 sin 2t = 0 when 2 + 2

2t = , so that t =

sec

Substituting for t:

xmax = e

1 sin 2 cos 2 = e + 2 2 2

Max. Amplitude xmax = e (b) m = 1, b = 2, k = 10

x(0) = 0, x(0) = 2

The DE is x + 2 x + 10 x = 0 for which the characteristic equation r2 + 2r + 10 = 0 gives r = 1 3i. x(t ) = e t (c1 cos3t + c2 sin 3t ) x(t ) = e t (3c1 sin 3t + 3c2 cos3t ) e t (c1 cos3t + c2 sin 3t ) 2 x(t ) = e t sin 3t is the solution 3 To find the maximum displacement x max , set x (t) =0 and solve for t:

2 0 = (3et cos 3t et sin 3t) 3 so that tan 3t = 3 and t = 0.416 radians which gives x max = 0.4172 .
x(0) = 0, x(0) = 2

(c)

m = 1, b = 4, k = 4

The DE is x + 4 x + 4 = 0 for which the characteristic equation r2 + 4r + 4 = 0 gives r = 2, 2 x(t ) = c1e 2t + c2te 2t x(t ) = 2c1e 2t + c2 (2te 2t The solution is x(t ) = 2te 2t . To find the maximum displacement xmax, we set x(t ) = 0 and solve for t: 0 = 2(2te 2t + e2t ) so that t = 1/2 which yields xmax = e1. c1 = 0, c2 = 2 2 t + e )

358

CHAPTER 4

Higher-Order Linear Differential Equations

Oscillating Euler-Cauchy 51.

We used the substitution y = t r and obtained for r1 = + i and r2 = i the solution


y ( t ) = k1t + i + k2t i = k1e(
+ i ) ln t

= e ln t ( c1 cos ( ln t ) + c2 sin ( ln t ) ) = t ( c1 cos ( ln t ) + c2 sin ( ln t ) ) .

+ k 2 e(

i ) ln t

= k1e ln t +i ln t + k2 e ln t i ln t

This is the same process as that used at the start of Case 3 in the text utilizing the Eulers Formula (4).
52.

1 3 t 2 y + 2ty + y = 0 , r ( r 1) + 2r + 1 = 0 , r 2 + r + 1 = 0 , r = i, 2 2 3 3 y ( t ) = t 1 2 c1 cos ln t + c2 sin ln t 2 2

53.

t 2 y + 3ty + 5 y = 0 Letting y = t r yields

t 2 r ( r 1) t r 2 + 3trt r 1 + 5t r = 0 t r {r ( r 1) + 3r + 5} = 0, and gives r 2 + 2r + 5 = 0 , which has roots 1 2i . Hence, the solution is


y ( t ) = t 1 ( c1 cos ( 2ln t ) + c2 sin ( 2ln t ) ) .
54.

t 2 y + 17ty + 16 y = 0

Euler-Cauchy: y = tm, t > 0 (characteristic equation)

m(m 1) + 17m + 16 = 0 m2 + 16m + 16 = 0 m=

16 (16) 2 4(16) = 8 4 3 2

y (t ) = t 8 c1 cos(4 3 ln t ) + c2 sin(4 3 ln t )

SECTION 4.3

Complex Characteristic Roots

359

Third-Order Euler-Cauchy 55.

The third-order Euler-Cauchy equation has the form at 3 y + bt 2 y + cty + dy = 0 . The derivatives of y = t r

(t > 0)

are y = rt r 1 y = r ( r 1) t r 2 y = r ( r 1)( r 2 ) t r 3

Substitute these equations into the third-order Euler-Cauchy equation above to obtain: at 3 r ( r 1)( r 2 ) t r 3 + bt 2 r ( r 1) t r 2 + ctrt r 1 + dt r = 0 at r r ( r 1)( r 2 ) + bt r r ( r 1) + ct r r + dt r = 0 Dividing by t r , we obtain the characteristic equation: ar ( r 1)( r 2 ) + br ( r 1) + cr + d = 0
Third-Order Euler-Cauchy Problems 56.

t 3 y + t 2 y 2ty + 2 y = 0 has Euler-Cauchy characteristic equation:

r ( r 1)( r 2 ) + r ( r 1) 2r + 2 = 0 r 3 3r 2 + 2r + r 2 r 2r + 2 = 0 r 3 2r 2 r + 2 = 0 Note: r = 1 is a zero of the polynomial f ( r ) = r 3 2r 2 r + 2 because f (1) = 1 2 1 + 2 = 0 . Therefore r 1 is a factor of r 3 2r 2 r + 2 , which enables us to find the other factors.

r 3 2r 2 r + 2 = ( r 1)( r + 1)( r 2 )
so r = 1 , 1, 2. Hence, the general solution to this Euler-Cauchy DE is y ( t ) = c1t + c2t 1 + c3t 2 , for t > 0 .
57.

t 3 y + 3t 2 y + 5ty = 0

Let y = tm, t > 0

m(m 1)(m 2) + 3m(m 1) + 5m = 0 m3 3m 2 + 2m + 3m 2 3m + 5 = 0 m3 + 4m = 0 y(t) = c1 + c2 cos (2 ln t) + c3 sin (2 ln t)

(characertistic equation) m = 0, 2i

360

CHAPTER 4

Higher-Order Linear Differential Equations

Inverted Pendulum 58.

The differential equation x x = 0 has the characteristic equation r 2 1 = 0 with roots 1 . Hence, the general solution is x ( t ) = c1et + c2 e t . (a) With initial conditions x ( 0 ) = 0 , x ( 0 ) = 1 , we find c1 = tion of the IVP is 1 1 x ( t ) = et et = sinh t . 2 2 As t , x ( t ) 0 if c1 = 0 , and then x ( t ) 0 also. This will happen whenever x (0) = x ( 0) .
Pendulum and Inverted Pendulum

1 1 and c2 = . Hence, the solu2 2

(b)

59.

(a)

The inverted pendulum equation has characteristic equation r 2 1 = 0 , which has roots 1 . Hence, the solution x ( t ) = c1et + c2 e t = c1 ( cosh t + sinh t ) + c2 ( cosh t sinh t ) = C1 sinh t + C2 cosh t , where C1 = c1 c2 , C2 = c1 + c2 .

(b)

The characteristic equation of the pendulum equation is r 2 + 1 = 0 , which has roots i . Hence, the solution x ( t ) = c1 cos t + c2 sin t .

(c)

The reader may think something strange about this because one form (a) appears real and (b) complex, but they are really the same; the difference is taken up by how one chooses the coefficients c1 , c2 in each case. The span of eit , e it is the same as the span of

{sin t ,

cos t} .

SECTION 4.3

Complex Characteristic Roots

361

Finding the Damped Oscillation 60.

The initial conditions x (0) = 1 , x (0) = 1 give the constants c1 = 1 , c2 = 2 . Hence, we have x ( t ) = e t ( cos t + 2sin t ) .

x 1.5 x(t ) = e t (cos t + 2 sin t ) 1 0.5 t

1 0.5

Extremes of Damped Oscillations 61.

The local maxima and minima of the curve x ( t ) = e t ( c1 cos t + c2 sin t ) have nothing to do with the exponential factor e t ; they depend only on c1 cos t + c2 sin t , which can be rewritten as A cos ( t ) having period T = 2

. Hence, consecutive maxima and

minima occur at equidistant values of t, the distance between them being one-half the period, or

. (You can note in Problem 32 that the time between the first local maxima and the first local minima is = .)
1

Underdamped Mass-Spring System 62.

We are given parameters and initial conditions

m = 0.25 , b = 1 , k = 4 , x ( 0 ) = 1 , x ( 0 ) = 0 .
Hence, the IVP is

0.25 x + x + 4 x = 0 , x ( 0 ) = 1 , x ( 0 ) = 0 ,
which has the solution 3 x ( t ) = e 2t cos 2 3t + sin 2 3t . 3

362

CHAPTER 4

Higher-Order Linear Differential Equations

Damped Mass-Spring System 63.

The IVP is x + bx + 64 x = 0 , x ( 0 ) = 1 , x ( 0 ) = 0 . (a) (b) (c) 5 b = 10 : (underdamped), x ( t ) = e 5t cos 39t + sin 39t 39

b = 16 : (critically damped), x ( t ) = (1 + 8t ) e8t b = 20 : (overdamped), x ( t ) =


1 4e 4t e 16t 3

LRC-Circuit I 64.

(a) (b) (c) (d)

The IVP is LQ + RQ +

1 Q = Q + 8Q + 25Q = 0 , Q ( 0 ) = 1 , Q ( 0 ) = 0 C

4 4 5 Q ( t ) = e 4t cos3t + sin 3t = e 4t cos ( 3t ) where = tan 1 3 3 3


I ( t ) = Q ( t ) = 5e 4t sin ( 3t ) 20 4t 4 e cos ( 3t ) where = tan 1 3 3

Charge on the capacitor and current in the circuit approach zero as t + .

LRC-Circuit II 65.

(a)

The IVP is LQ + RQ +

1 1 Q = Q + 1Q + 4Q = 0 , Q ( 0 ) = 1 , Q ( 0 ) = 0 C 4

(b)

2 3 2t 3 3 Q ( t ) = e 2t cos 2 3t + e cos 2 3t , tan = sin 2 3t = 3 3 3

(c) (d)

I (t ) = Q (t ) =

4 3 2t 3 e cos 2 3t 4e 2t sin 2 3t , tan = 3 3

As t , both Q ( t ) 0 and I ( t ) 0

Computer Lab: Damped Free Vibrations 66.

IDE Lab

SECTION 4.3

Complex Characteristic Roots

363

Effects of Nonconstant Coefficients 67.

1 x+ x=0 t (a) This ODE describes (among other things) an undamped vibrating spring in which the restoring force is initially very large (when t is near zero), but eventually decays to zero, causing the frequency of vibration to decrease and the solution period to increase as t increases. (b) We plotted the solution with IC x ( 0.1) = 2 , x ( 0.1) = 0 in the tx and xx planes.
x

3 2 1 20 10 1 2 3 4 10 20 x

(c)

As we expected, the tx graph shows that the period of the oscillation increases with t. We see also that the amplitude increases in the absence of friction. The xx phase portrait shows that as time and amplitude increase, velocity decreases, which is consistent with the previous observations. A good question for further exploration would be whether amplitude increases indefinitely or levels off.

68.

1 x+ x+x=0 t (a) This ODE describes a damped vibrating spring in which the damping starts very large when t is near zero, but decays to zero. We suspect that initially the amplitude of a solution will rapidly decay, but as time increases the motion could become almost like simple harmonic oscillation, as there will be almost no friction. (b) We plotted the solution with IC x ( 0.1) = 2 , x ( 0.1) = 0 in the tx as well as the xx planes.

364

CHAPTER 4

Higher-Order Linear Differential Equations

1 0.5 1 0.5 1 1.5 2 1 2 x

(c)

As first expected, the tx graph shows that the solution is rapidly decaying. However the xx phase portrait, constructed with a longer time interval, shows that our second expectation is not confirmed. As time increases the oscillations do not become harmonicthe amplitude of the oscillations continues to decrease, gradually and indefinitely.

69.

tx + x = 0 (a) If you divide by t, you will see that this equation is the same as the equation in Problem 67.

70.

x + x2 1 x + x = 0 (a) This ODE shows negative friction for x < 1 and positive damping for x > 1 . For a small initial condition near x = 0 , we might expect the solution to grow and then oscillate around x = 1 . (b) We plotted the solutions in the tx and xx planes at initial velocity x ( 0 ) = 0 for three different initial displacements: x ( 0 ) = 0.5 , x ( 0 ) = 2.0 , x ( 0 ) = 4.0 .
. x 4

4 x

SECTION 4.3

Complex Characteristic Roots

365

(c)

As expected, the tx graph shows that initially the solution is growing for x ( 0 ) = 0.5 and decaying for x ( 0 ) = 4 . We also see that all the solutions seem to become periodic with the same amplitude and period, but we note that the motion is not exactly sinusoidal and that the amplitude is about 2 rather than 1 as we suspected. The xx phase portrait confirms that the long term trajectories are not circular as in simple harmonic motion, but distorted as we see in the tx graph. This equation is called van der Pols equation and describes oscillations (mostly electrical) where internal friction depends on the value of the dependent variable x; further details will be explored in Chapter 7.

71.

x + ( sin t ) x + x = 0 (a) In this ODE damping changes periodically from negative to positive, so we can predict oscillation in amplitude as well as periodic vibratory motion. (b) We plotted the solution with IC x ( 0 ) = 2, x ( 0 ) = 0 in the tx and xx planes.

(c)

The tx graph looks like a superposition of two periodic oscillations. The xx phase portrait for a longer time interval shows that continued oscillations almost repeat, but never exactly. This is called quasi-periodic motion.

72.

1 x + x + tx = 0 t (a) For this ODE damping is initially large, but vanishes as time increases; the restoring force on the other hand is initially small but increases with time. How will these effects combine? (b) We plotted the solution with IC x ( 0.1) = 2, x ( 0.1) = 0 in the tx and xx planes.

366

CHAPTER 4

Higher-Order Linear Differential Equations

2 1

1 1 2

(c)

As we expected, the tx graph shows initially large damping, which rapidly decreases the amplitude of the solution, and increasing frequency, due to the effect of the increasing spring constant, which shortens the period. The center of the xx graph will continue to fill in, very slowly, if you give it a much longer time interval.

73.

x + ( sin 2t ) x = 0 (a) In this ODE the restoring force changes periodically from positive to negative with a frequency that is different from the natural frequency of the spring. We expect some complicated but periodic motion. (b) We plotted the solution with IC x ( 0 ) = 2 , x ( 0 ) = 0 in the tx and xx planes.
x 4 2 100 t

20 2 4

40

60

80

(c)

The tx graph to t = 100 indeed looks almost periodic, with period 50. However the xx phase portrait over a longer time interval shows that continued motion almost repeats, but never exactly. This is another example of quasi-periodic motion, as in Problem 71. Extending the tx graph will be another good way to see that the long term motion is indeed not perfectly repeating.

SECTION 4.3

Complex Characteristic Roots

367

Boundary-Value Problems 74.

y + y = 0 , y(0) = 0, y = 0 2 y(t) = c1 cos t + c2 sin t y(0) = 0 = c1


y = 0 = c2, so y (t ) = 0 is the solution. 2

75.

y + y = 0 , y(0) = 0, y = 1 2 y(t) = c1 cos t + c2 sin t y(0) = 0 = c1


y = 1 = c2, so y (t ) = sin t is the solution. 2

76.

y + y = 0 , y(0) = 1, y ( ) = 1 y(t) = c1 cos t + c2 sin t y(0) = 1 = c1 y ( ) = 1 = c1 *No solutions

77.

y + y = 0 , y = 1, y = 2 4 2 y(t) = c1 cos t + c2 sin t 1 = c1 2 = c2 y (t ) = 1 1 + c2 2 2 c1 + c2 = c1 = 2

2 2

2 2 cos t + 2sin t is the solution.

368

CHAPTER 4

Higher-Order Linear Differential Equations

Exact Second-Order Differential Equations 78.

1 1 1 y + y 2 y = 0 is the same as y + y = 0 . t t t 1 Integrating we obtain the linear equation y + y = c1 , t for which = e Thus,

t dt = eln t = t so we have ty + y = c t. 1

c c c d (ty ) = c1t , so ty = 1 t 2 + c2 and y (t ) = 1 t + 2 . 2 dt 2 t


c is the general solution. t

Substituting back into the original equation we find c1 = 0, so y (t ) =


79.

2 2 y 2 = 0 t t 2 y + y = 0 t y + Integrating and setting c1 = 0, we obtain 2 dt 2 y + y = 0 = e t = e2ln t = t 2 t 2 t y + 2ty = 0 = c. d 2 c (t y ) = 0 , t2y = c, y (t ) = 2 dt t

80.

(t 2 2t ) y + 4(t 1) y + 2 y = 0 where t 2 2t 0 Find ( gy ) : ( gy ) = gy + yg ( gy ) = ( gy + yg ) = gy + y g + yg + g y = gy + 2 g y + yg Let g = t2 2t. Then g = 2t 2, g = 2


Then ( gy ) = (t 2 2t ) y + 4(t 1) y + 2 y ( gy ) = 0 ( gy ) = c1 gy = c1t + c2 so y (t ) = c1t + c2 . t 2 2t

Suggested Journal Entry 81.

Student Project

SECTION 4.4

Undetermined Coefficients

369

4.4

Undetermined Coefficients

Inspection First 1. 3. 5. 7.

y y = t y p ( t ) = t
y = 2 y p ( t ) = t 2

2. 4. 6. 8.

y + y = 2 y p ( t ) = 2t
ty + y = 4t y p ( t ) = t 2

y 2 y + 2 y = 4 y p ( t ) = 2
y y + y = et y p ( t ) = et

y y = 2cos t y p ( t ) = cos t
y + y + y = 2t + 2 y p ( t ) = 2t

Educated Prediction

The homogeneous equation y + 2 y + 5 y = 0 has characteristic equation r 2 + 2r + 5 = 0 , which has complex roots 1 2i . Hence,

yh ( t ) = c1et sin 2t + c2 e t cos 2t ,


so for the right-hand sides f ( t ) , we try the following:
9. 10. 12.
f ( t ) = 2t 3 3t y p ( t ) = At 3 + Bt 2 + Ct + D

f ( t ) = tet y p ( t ) = ( At + B ) et

11.

f ( t ) = 2sin t y p ( t ) = A sin t + B cos t

f ( t ) = 2e t sin t y p ( t ) = e t ( A cos t + B sin t )


Guess Again

The homogeneous equation y 6 y + 9 y = 0 has characteristic equation r 2 6r + 9 = 0 , which has a double root 3, 3. Hence, yh ( t ) = c1e3t + c2te3t . We try particular solutions of the form:
13. 14.

f ( t ) = t cos 2t y p ( t ) = ( At + B ) sin t + ( Ct + D ) cos t

f ( t ) = te3t y p ( t ) = At 3 + Bt 2 e3t
(We cant have any terms here dependent on terms in the homogeneous solution.)

15. 16.

f ( t ) = e t + sin t y p ( t ) = Ae t + B sin t + C cos t

f ( t ) = t 4 t 2 + 1 y p ( t ) = At 4 + Bt 3 + Ct 2 + Dt + E

370

CHAPTER 4

Higher-Order Linear Differential Equations

Determining the Undetermined 17.

observe that y p ( t ) = t is a solution of the nonhomogeneous equation. Hence, the general solution is y (t ) = t + c .
18.

y = 1 . The homogeneous solution is yh ( t ) = c , where c is any constant. By simple inspection we

tion we observe that y p ( t ) = 1 is a solution of the nonhomogeneous equation. Hence, the general solution is

y + y = 1 . The homogeneous solution is yh ( t ) = ce t where c is any constant. By simple inspec-

y ( t ) = ce t + 1 .
19.

y + y = t . yh ( t ) = ce t , y p = At + B , y p = A . Substituting into the DE gives A + ( At + B ) = t .


Coefficient of t: A = 1 . Coefficient of 1: A + B = 0 . Hence, A = 1 , B = 1 . y p = t 1 , y = ce t + t 1 .

20.

y = 1 . The homogeneous solution of the equation is yh ( t ) = c1t + c2 , 1 where c1 , c2 are arbitrary constants. By inspection, we note that y p = t 2 is a particular 2 solution. Hence, the solution of the homogeneous equation is 1 y ( t ) = t 2 + c1t + c2 . 2 If you could not find a particular solution by inspection, you could try a solution of the form y p ( t ) = At 2 + Bt + C .

21.

y + 4 y = 1 . The characteristic equation is r 2 + 4r = 0 , which has roots 0, 4. Hence, the homogeneous solution is

yh ( t ) = c1 + c2 e 4t .
The constant on the right-hand side of the differential equation indicates we seek a particular solution of the form y p ( t ) = A , except that the homogeneous solution has a constant solution; thus we seek a solution of the form y p ( t ) = At . Substituting this expression into the differential equation yields 4 A = 1 , or A = 1 . Hence, we have a particular solution 4 1 y p (t ) = t , 4

SECTION 4.4

Undetermined Coefficients

371

so the general solution is 1 y ( t ) = c1 + c2 e 4t + t . 4


22.

y + 4 y = 1 . The characteristic equation is r 2 + 4 = 0 , which has roots 2i . Hence, the homogeneous solution is yh ( t ) = c1 cos 2t + c2 sin 2t . The constant on the right-hand side of the differential equation indicates we seek a particular solution of the form y p ( t ) = A . Substituting this expression into the differential equation yields 4 A = 1 , or A = 1 1 . We have a particular solution y p ( t ) = , so the general solution 4 4
y ( t ) = c1 cos 2t + c2 sin 2t + 1 . 4

23.

y + 4 y = t . The characteristic equation is r 2 + 4r = 0 , which has roots 0, 4. Hence, the homogeneous solution is

yh ( t ) = c1 + c2 e 4t .
The term on the right-hand side of the differential equation indicates we seek a particular solution of the form y p ( t ) = At + B . However, the homogeneous solution has a constant term so we seek a solution of the form y p ( t ) = At 2 + Bt . Substituting this expression into the differential equation yields

y + 4 y = 2 A + 8 At + 4 B = t .
Setting the coefficient of t, 1 equal to each other yields A =
1 1 , B = . Thus, the solution 8 16

1 1 y ( t ) = c1 + c2 e 4t + t 2 t . 8 16
24.

y + y 2 y = 3 6t . The characteristic equation is r 2 + r 2 = 0 , which has roots 2 and 1.


Hence, the homogeneous solution yh ( t ) = c1e2t + c2 et . The linear polynomial on the right-hand side of the equation indicates we seek a particular solution of the form

372

CHAPTER 4

Higher-Order Linear Differential Equations

y p ( t ) = At + B . (Note that we dont have any matches with the homogeneous solution.) Substituting this expression into the differential equation yields the equation y + y 2 y = A 2 At 2 B = 3 6t so A = 3 , B = 0 . Hence, we have the general solution

y ( t ) = c1e2t + c2 et + 3t .
25.

y + y = et + 3 . The characteristic equation is r 2 + 1 = 0 , which has roots i . Hence, the homogeneous solution is yh ( t ) = c1 cos t + c2 sin t . The terms on the right-hand side of the differential equation indicates we seek a particular solution of the form
y p ( t ) = Aet + B .

Substituting this expression into the differential equation yields y + y = 2 Aet + B = et + 3 . Setting coefficients of et , 1 equal to each other, we get equations for A, B, which yield A = B = 3 . Hence, we have the general solution 1 y ( t ) = c1 cos t + c2 sin t + et + 3 . 2
26.

1 , 2

y y 2 y = 6et . The characteristic equation is r 2 r 2 = 0 , which has roots 1 and 2. Hence, the homogeneous solution is yh ( t ) = c1e2t + c2 e t . The exponential term on the right-hand side of the differential equation indicates we seek a particular solution of the form y p ( t ) = Aet . (Note this is not linearly dependent on any of the exponential terms in the homogeneous solution.) Substituting this expression into the differential equation we get y y 2 y = 2 Aet = 6et .

SECTION 4.4

Undetermined Coefficients

373

Hence, A = 3 , and we have a particular solution


y p ( t ) = 3et ,

and hence

y ( t ) = c1e2t + c2 e t 3et .
27.

y + y = 6sin 2t . The characteristic equation is r 2 + r = 0 , which has roots 0 and 1. Hence, the homogeneous solution is yh ( t ) = c1 + c2 e t . The sine term on the right-hand side of the differential equation indicates we seek a particular solution of the form y p ( t ) = A cos 2t + B sin 2t . Substituting into the differential equation yields y + y = ( 4 A + 2 B ) cos 2t + ( 4 B 2 A ) sin 2t = 6sin 2t . Comparing coefficients yields the equations 4 A + 2 B = 0 4 B 2 A = 6,
3 6 which has the solution A = , B = . Hence, we have 5 5 3 6 y p ( t ) = cos 2t sin 2t , 5 5

and the general solution is


3 6 y ( t ) = c1 + c2 e t cos 2t sin 2t . 5 5

28.

y + 4 y + 5 y = 2et . The characteristic equation of the differential equation is r 2 + 4r + 5 = 0 , which has roots 2 i . Hence, the homogeneous solution is yh ( t ) = e 2t ( c1 cos t + c2 sin t ) . The exponential on the right-hand side of the differential equation indicates we seek a particular solution of the form y p ( t ) = Aet . Substituting this expression into the differential equation yields y + 4 y + 5 y = 10 Aet = 2et ,

374

CHAPTER 4

Higher-Order Linear Differential Equations

which yields A = is given by

1 1 . Hence, we have a particular solution y p ( t ) = et , and the general solution 5 5

1 y ( t ) = e 2t ( c1 cos t + c2 sin t ) + et . 5

29.

y + 4 y + 4 y = te t . The characteristic equation is given by r 2 + 4r + 4 = 0 , which has a double root of 2, so the homogeneous solution is

yh ( t ) = c1e2t + c2te2t .
The term on the right-hand side of the differential equation indicates we seek a particular solution of the form
y p ( t ) = Ate t + Be t .

Substituting this expression into the differential equation yields

y + 4 y + 4 y = Atet + ( 2 A + B ) e t = te t .
Comparing coefficients, yields equations, which we solve, getting A = 1 , B = 2 . Hence, the general solution is y ( t ) = c1e 2t + c2te2t + te t 2e t .
30.

y y = t sin t . The characteristic equation is r 2 1 = 0 , which has roots 1 . Hence, the homogeneous solution is yh ( t ) = c1et + c2 e t . The term on the right-hand side of the differential equation indicates we seek a particular solution

y p ( t ) = ( At + B ) cos t + ( Ct + D ) sin t .
Differentiating this expression two times and substituting it into the differential equation yields the algebraic equation y y = 2Ct sin t 2 At cos t + ( 2 A 2 D ) sin t + ( 2C 2 B ) cos t = t sin t . Comparing terms in sin t , cost , t sin t , t cos t , we get equations that yield 1 1 A = 0, B = , C = , D = 0. 2 2 Hence,
y ( t ) = c1et + c2 e t 1 ( t sin t + cos t ) . 2

SECTION 4.4

Undetermined Coefficients

375

31.

y + y = 12cos 2 t . The characteristic equation is r 2 + 1 = 0 , which has roots i . Hence, the homogeneous solution is yh ( t ) = c1 cos t + c2 sin t . Using the trigonometric identity cos t = 1 (1 + cos 2t ) 2

the term on the right-hand side of the differential equation yields 12cos 2 t = 6 (1 + cos 2t ) . Hence, we seek a particular solution of the form y p ( t ) = A cos 2t + B sin 2t + C . Substituting this into the differential equation yields y + y = 3 A cos 2t 3B sin 2t + C = 6 + 6cos 2t . Comparing coefficients, we get A = 2 , B = 0 , C = 6 , so the general solution is y ( t ) = c1 cos t + c2 sin t 2cos 2t + 6 .
32.

y y = 8tet . The characteristic equation is r 2 1 = 0 , which has roots 1 . Hence, the homogeneous solution is

yh ( t ) = c1et + c2 e t .
The term on the right-hand side of the differential equation indicates we seek a particular solution
y p ( t ) = Atet + Bet ,

but one term in the homogeneous solution is linearly dependent on this term, so we seek

y p ( t ) = et At 2 + Bt .
Substituting this expression into the differential equation yields y y = 4 Atet + ( 2 A + 2 B ) et = 8tet , which gives the two equations 4 A = 8 , 2 A + 2 B = 0 , which gives A = 2 , B = 2 . Hence, the general solution is

y ( t ) = c1et + c2 e t + 2tet ( t 1) .

376

CHAPTER 4

Higher-Order Linear Differential Equations

33.

y 4 y + 4 y = te 2t . The characteristic equation of the differential equation is r 2 4r + 4 = 0 , which has a double root of 2. Hence, the homogeneous solution is yh ( t ) = c1e2t + c2te2t . The term on the right-hand side of the differential equation indicates we seek a particular solution of the form y p ( t ) = Ate2t + Be2t , but both terms are linearly dependent with terms in the homogeneous solution, so we choose y p ( t ) = At 3e2t + Bt 2 e2t . Differentiating and substituting this expression into the differential equation yields the algebraic equation y 4 y + 4 y = e 2t ( 0 + 0 + 6 At + 2 B ) = te 2t . Comparing coefficients, we get A = 1 , B = 0 . Hence, the general solution is 6

1 y ( t ) = c1e 2t + c2te 2t + t 3e 2t . 6

34.

y 4 y + 3 y = 20cos t . The characteristic equation of the differential equation is r 2 4r + 3 = 0 , which has roots 1, 3. Hence,

yh ( t ) = c1et + c2 e3t .
The term on the right-hand side of the differential equation indicates we seek y p ( t ) = A cos t + B sin t . Substituting this expression into the differential equation yields y 4 y + 3 y = ( 4 A + 2 B ) sin t + ( 2 A 4 B ) cos t = 20cos t . Comparing coefficients yields A = 2 , B = 4 . Hence, y ( t ) = c1et + c2 e3t + 2cos t 4sin t .
35.

y 3 y + 2 y = et sin t . The characteristic equation of the differential equation is r 2 3r + 2 = 0 , which has roots 1, 2. Hence,

yh ( t ) = c1et + c2 e2t .
The term on the right-hand side of the equation indicates we seek a particular solution
y p ( t ) = Aet cos t + Bet sin t .

SECTION 4.4

Undetermined Coefficients

377

Differentiating and substituting this expression into the equation yields y 3 y + 2 y = ( A B ) et sin t + ( A B ) et cos t = et sin t . Comparing coefficients, we find A = 1 1 , B = , yielding the general solution 2 2

1 y ( t ) = c1et + c2 e2t + et ( cos t sin t ) . 2


36.

y + 3 y = sin t + 2cos t . The characteristic equation is r 2 + 3r = 0 , which has roots 0, 3. Hence,


the homogeneous solution is yh ( t ) = c1 + c2 e 3t . The sine and cosine terms on the right-hand side of the equation indicate we seek a particular solution of the form y p ( t ) = A cos t + B sin t . Substituting this into the equation yields y + 3 y = ( A + 3B ) cos t + ( B 3 A ) sin t = sin t + 2cos t . 1 1 Comparing terms, we arrive at ( A + 3B ) = 2 , ( B 3 A ) = 1 , yielding A = , B = . From 2 2 this, that the general solution is y ( t ) = c1 + c2 e 3t + 1 ( sin t cos t ) . 2

37.

y 4 y = 6t
(1) Find yh: r3 4r2 = 0 r2(r 4) = 0 r = 0, 0, 4 yh = c1 + c2t + c3e4t (2) Find yp: yP = t2(At + B) = At3 + Bt2 y p = 3At2 + 2Bt y p = 6At + 2B

y p = 6A
y p 4 y p = 6 A 4(6 At + 2 B ) = 24 At + 6 A 8B = 6t coefficient of t: 24A = 6, coefficient of 1: 6A 8B = 0

378

CHAPTER 4

Higher-Order Linear Differential Equations

1 so that A = , and 8B = 4
1 3 Hence yp = t 3 t 2 4 16

3 3 1 6 = so that B = . 2 16 4

(3)
38.

1 3 y (t ) = yh + y p = c1 + c2t + c3e 4t t 3 t 2 4 16

y (3) 3 y + 3 y y = et (1) Find yh: r3 3r2 + 3r 1 = 0 (characteristic equation) f(r) = r3 3r2 + 3r 1 = 0 f(1) = 1 3 + 3 1 = 0 so r = 1 is a root. By long division, we obtain r3 3r2 + 3r 1 = (r 1)(r2 2r + 1) = (r 1)3 yh = c1et + c2tet + c3t2et (2) Find yp: yp = t3(Aet) = At3et y p = At 3et + 3 At 2 et Triple root r = 1, 1, 1

y p = At 3et + 3 At 2 et + 3 At 2 et + 6 Atet = At 3et + 6 At 2 et + 6 Atet


y (3) = At 3et + 3 At 2 et + 3 At 2 et + 6 Atet + 3 At 2 et + 6 Atet + 6 Atet + 6 Aet p

y (3) 3 y + 3 y p y p = p p

At 3et + 9 At 2 et + 18 Atet + 6 Aet 3 At 3et 18 At 2 et 18 Atet + 0et + 3 At 3et + 9 At 2 et + At 3et + 0t 2 et + 0tet + 0et 0tet + 0et 6 Aet = et so that A = 1 6

Thus yp = (3)

1 3 t t e 6

1 y (t ) = yh + y p = c1et + c2tet + c3t 2 et + t 3et 6

SECTION 4.4

Undetermined Coefficients

379

39.

y (4) y = 10 (1) Find yh: r4 1 = 0 (r 2 + 1)( r 2 1) = (r 2 + 1)(r 1)( r + 1) r = i, 1

yh = c1 cos t + c2 sin t + c3et + c4et (2) Find yp: yp = A, so that y p = y = y = y (4) = 0 p p p y (4) y p = 0 A = 10 A = 10 y p = 10 p (3)
40.

y (t ) = yh + y p = c1 cos t + c2 sin t + c3et + c4 e t 10

y = y y y = 0
(1) (2) (3) Find yh: r3 r2 = 0 r2(r 1) = 0 r = 0, 0, 1

There is no yp because the DE is homogeneous. y(t) = c1 + c2t + c3et

Initial-Value Problems 41.

y + y 2 y = 3 6t , y (0) = 1, y(0) = 0
(1) Find yh: r2 + r 2 = 0 (r 1)(r + 2) = 0 r = 1, 2 yh = c1et + c2e2t (2) Find yp: yp = At + B, y p = A, y = 0 y + y p 2 y p = A 2( At + B) = 3 6t p p coefficient of t: 2A = 6, coefficient of 1: A 2B = 3 A = 3, B = 0 yp = 3t (3) y(t) = yh + yp = c1et + c2e2t + 3t; y = c1et 2c2 e2t + 3 y(0) = 1 c1 + c2 = 1; y (0) = 0 c1 2c2 + 3 = 0 c1 + c2 = 1 c1 2c2 = 3 5 2 y = et + e 2t + 3t 3 3 c2 = c1 = 2 3 5 3

380

CHAPTER 4

Higher-Order Linear Differential Equations

42.

y + 4 y + 4 y = te t , y (0) = 1, y (0) = 1 (1) Find yh: r2 + 4r + 4 = 0 (r + 2)2 = 0 r = 2, 2 yh = c1e2t + c2te2t = (c1 + c2t)e2t (2) Find yp: yp = et(At + B) y p = e t ( At + B ) + Ae t y = e t ( At + B) Ae t Ae t = e t ( At + B) 2 Ae t p So y + 4 y p + 4 y p = e t ( At + B) 2 Ae t + 4(e t ( At + B ) + Ae t ) + 4e t ( At + B) p
= e t ( At + 2 A + B ) This gives A = 1, 2A + B = 0 and so A = 1 and B = 2. Therefore yp = et(t 2). (3) y = yh + yp = c1e2t + c2te2t + et(t 2) y = 2c1e 2t + c2 e 2t 2c2te 2t et (t 2) + e t y (0) = 1 c1 2 = 1 y (0) = 1 2c1 + c2 + 2 + 1 = 1 y(t) = e2t + et(t 2) c1 = 1 c2 = 0

43.

y + 4 y = t , y (0) = 1, y(0) = 1
(1) (2) Find yh: r2 + 4 = 0 r = 2i yh = c1 cos 2t + c2 sin 2t Find yp: yp = At + B, y p = A, y = 0 p y + 4 y p = 4( At + b) = 4 At + 4 B = t p coefficient of t: 4A = 1, coefficient of 1: 4B = 0 A = yp = (3)
1 t 4

1 ,B = 0 4

y = yh + yp = c1 cos 2t + c2 sin 2t +

1 1 t , y = 2c1 sin 2t + 2c2 cos 2t + 4 4


1 5 5 = 1 2c2 = c2 = 4 4 8

y(0) = 1 c1 = 1; y (0) = 1 2c2 + 5 1 y (t ) = cos 2t sin 2t + t 8 4

SECTION 4.4

Undetermined Coefficients

381

44.

y + 2 y + y = 6cos t , y (0) = 1, y (0) = 1 (1) Find yh: r2 + 2r + 1 = 0 (r + 1)2 = 0 r = 1, 1 yh = c1et + c2tet (2) Find yp: yp = A cos t + B sin t , y p = A sin t + B cos t , y = A cos t B sin t p y + 2 y p + y p = p A cos t B sin t + 2( B cos t A sin t ) + A cos t + B sin t 2B cos t 2A sin t = 6 cost coefficient of cos t: 2B = 6, coefficient of sin t: 2A = 0 A = 0, B = 3 yp = 3 sin t (3) y = yh + yp = c1e t + c2te t + 3sin t , y = c1e t c2te t + c2 e t + 3cos t y(0) = 1 c1 = 1; y (0) = 1 c1 + c2 + 3 = 1 c2 = 3 y (t ) = e t 3te t + 3sin t

45.

4 y + y = cos 2t , y (0) = 1, y (0) = 0 (1) (2) Find yh: 4r2 + 1 = 0 r2 = 1 1 1 1 r = i yh = c1 cos t + c2 sin t 4 2 2 2

Find yp: yp = A cos 2t + B sin 2t, y p = 2 A sin 2t + 2 B cos 2t , y = 4 A cos 2t 4 B sin 2t p 4 y + y p = 16 A cos 2t 16 B sin 2t + A cos 2t + B sin 2t p

= 15 A cos 2t 15 B sin 2t = cos 2t


coefficient of cos 2t: 15A = 1, coefficient of sin 2t: 15 B = 0 A= yp =
1 ,B=0 15

1 cos 2t 15

382

CHAPTER 4

Higher-Order Linear Differential Equations

(3)

1 1 1 y = yh + yp = c1 cos t + c2 sin t cos 2t 2 2 15


1 1 1 1 2 y = c1 sin t + c2 cos t + sin 2t 2 2 2 2 15

y(0) = 1 c1

1 16 = 1 c1 = 15 15

y (0) = 0 c2 = 0
y (t ) =
16 1 1 cos t cos 2t 15 2 15

46.

y + 9 y = cos3t , y (0) = 1, y (0) = 1 (1) Find yh: r2 + 9 = 0 yh = c1 cos 3t + c2 sin 3t (2) Find yp: yp = t(A cos 3t + B sin 3t) y p = t (3 A sin 3t + 3B cos3t ) + ( A cos3t + B sin 3t ) y = t (9 A cos3t 9 B sin 3t ) + (3 A sin 3t + 3B cos3t ) 3 A sin 3t + 3B cos3t p y + 9 y p = (9 A 9 A)t cos3t + (9 B 9 B )t sin 3t + (6 B )cos3t + (6 A sin 3t ) = cos 3t p coefficient of cos 3t: 6B = 1, coefficient of sin 3t: 6A = 0 A = 0, B = (3) 1 1 so that yp = t sin 3t 6 6 1 t sin 3t , 6 r = 3i

y = yh + yp = c1 cos 3t + c2 sin 3t + y = 3c1 sin 3t + 3c2 cos 3t +

1 1 t cos3t + sin 3t 2 6

y(0) = 1 c1 = 1; y (0) = 1 3c2 = 1 c2 =


1 1 y (t ) = cos3t sin 3t + t sin 3t 3 6

1 3

SECTION 4.4

Undetermined Coefficients

383

47.

y 3 y + 2 y = 4e t , y(0) = 1, y (0) = 0 (1) Find yh: r2 3r + 2 = 0 (r 1)(r 2) = 0 r = 1, r = 2, so that yh = c1et + c2e2t (2) Find yp: yp = Aet, y p = Ae t , y = Ae t p
y 3 y p + 2 y p = Ae t 3( Ae t ) + 2 Ae t = 6 Ae t = 4e t so that A = p 2 2 yp = e t 3 3

(3)

y = yh + yp = c1et + c2e2t + y(0) = 1 c1 + c2 +

2 t 2 e , y = c1et + 2c2 e 2t e t 3 3 c1 + c2 =
1 3

2 =1 3

c1 = 0 c2 = 1 3

2 y (0) = 0 c1 + 2c2 = 0 3 1 2 Thus y (t ) = e 2t + e t . 3 3


48.

2 c1 + 2c2 = 3

y 4 y + 3 y = e t + t , y(0) = 0, y (0) = 0 (1) Find yh: r2 4r + 3 = 0 (r 1)(r 3) = 0 so that yh = c1et + c2e3t (2) Find yp: yp = Aet + Bt + C, y p = Ae t + B, y = Ae t p Thus y 4 y p + 3 y p = Ae t 4( Ae t + B ) + 3( Ae t + Bt + C ) p = Ae t 4( Ae t + B) + 3( Ae t + Bt + C ) = 8Aet + 3Bt 4B + 3C = et + t 8A = 1, 3B = 1, 4B + 3C = 0 A = Thus, yp = 1 t 1 4 e + t+ . 8 3 9
1 1 4 4 ,B= , C= B= . 8 3 3 9

384

CHAPTER 4

Higher-Order Linear Differential Equations

(3)

Find y: y = yh + yp = c1et + c2e3t + 1 t 1 4 e + t+ . 8 3 9

1 1 y = c1et + 3c2 e3t e t + . 8 3

y(0) = 0 c1 + c2 +

1 4 + =0 8 9

c1 + c2 =

41 72

c1 = c2 =

3 4

1 1 y (0) = 0 c1 + 3c2 + = 0 8 3

5 c1 + 3c2 = 24

13 72

3 13 1 1 4 Therefore, y(t) = et + e3t + e t + t + . 4 72 8 3 9

49.

y y 2 y = 4 cos 2t, y(0) = 0, y (0) = 0 (1) Find yh: r2 r 2 = 0 (r + 1)(r 2) = 0 so r = 2, 1 yh = c1et + c2e2t (2) Find yp: yp = A cos 2t + B sin 2t, y p = 2A sin 2t + 2B cos 2t y = 4A cos 2t 4B sin 2t p y y p 2 y p = 4A cos 2t 4B sin 2t + 2A sin 2t 2B cos 2t 2A cos 2t 2B sin 2t p = (6A 2B) cos 2t + (2A 6B) sin 2t = 4 cos 2t coefficient of cos 2t: 6A 2B = 4, coefficient of sin 2t: 2A 6B = 0 so A = 3B and B =
1 3 A= 5 5

3 1 Thus yp = cos 2t sin 2t . 5 5

SECTION 4.4

Undetermined Coefficients

385

(3)

y = yh + yp = c1et + c2e2t

3 1 cos 2t sin 2t 5 5

y = c1et + 2c2e2t +
y(0) = 0 c1 + c2

6 2 sin 2t cos 2t 5 5 c1 + c2 = 3 5 c1 = c2 = 4 15 1 3

3 =0 5

2 y (0) = 0 c1 + 2c2 = 0 5 y (t ) =
50.

2 c1 + 2c2 = 5

4 t 1 2t 3 1 e + e cos 2t sin 2t 15 3 5 5

y 4 y + 3 y = t 2 , y(0) = 1, y (0) = 0, y (0) = 0


(1) Find yh: r3 4r2 + 3r = 0 r(r2 4r + 3) = 0 r(r 3)(r 1) = 0, so r = 0, 1, 3 yh = c1 + c2et + c3e3t (2) Find yp: yp = t(At2 + Bt + C) = At3 + Bt2 + Ct y p = 3At2 + 2Bt + C, y = 6At + 2B, y = 6A p p y 4 y + 3 y p = 6A 24At 8B + 9At2 + 6Bt + 3C = t2 p p coefficient of t2: 9A = 1, coefficient of t: 24A + 6 B = 0, coefficient of 1: 6A 8B + 3C = 0 A= 1 4 26 ,B= , C= 9 9 27 1 3 4 2 26 t + t + t 9 9 27
1 3 4 2 26 t + t + t 9 9 27

yp = (3)

y = yh + yp = c1 + c2et + c3e3t +

Using this general solution and the initial conditions we obtain: y (t ) = 161 2 t 8 3t 1 3 4 2 26 e e + t + t + t 81 3 81 9 9 27

386

CHAPTER 4

Higher-Order Linear Differential Equations

51.

y(4) y = e2t, y(0) = y (0) = y (0) = y (0) = 0 (a) Find yh: r4 1 = 0 (r2 + 1)(r2 1) r = i, 1 yh = c1 cos t + c2 sin t + c3et + c4et (2) Find yp: yp = Ae2t, y p = 2Ae2t, y = 4Ae2t, y = 8Ae2t, y (4) = 16Ae2t p p p Thus y (4) y p = 16Ae2t Ae2t = e2t 15Ae2t = e2t A = p (3) y = yh + yp = c1 cos t + c2 sin t + c3et + c4et + y = c1 sin t + c2 cos t + c3 et c4et + y = c1 cos t c2 sin t + c3 et + c4et + 1 2t e 15
1 1 2t so that yp = e . 15 15

2 2t e 15

4 2t e 15

y = c1 sin t c2 cos t + c3 et c4et +


y(0) = 0 c1 + c3 + c4 +
y (0) = 0 c2 + c3 c4 + 1 =0 15 2 =0 15 4 =0 15 8 =0 15

8 2t e 15

y (0) = 0 c1 + c3 + c4 +

y (0) = 0 c2 + c3 c4 +

From these 4 equations in 4 unknowns, we obtain (by the methods of Chapter 3), c1 = 1 1 1 1 , c2 = , c3 = and c4 = 10 5 4 12 1 1 1 1 1 cos t + sin t et + e t + e2t 10 5 4 12 15

y (t ) =

52.

y(4) = et, y(0) = 1, y (0) = 0, y (0) = 0 , y (0) = 0 (1) Find yh: r4 = 0 r = 0 (multiplicity 4) yh = c1 + c2t + c3t2 + c4t3

SECTION 4.4

Undetermined Coefficients

387

(2)

Find yp: yp = Aet, y p = Aet, y = Aet, y = Aet, y (4) = Aet p p p


y (4) = Aet = et A = 1 so that yp = et p

(3)

y = yh + yp = c1 + c2t + c3t2 + c4t3 + et y = c2 + 2c3t + 3c4t2 + et y = 2c3 + 6c4t + et y = 6c4 + et

y(0) = 1 c1 + 1 = 1 c1 = 0 y (0) = 0 c2 + 1 = 0 c2 = 1 y (0) = 0 2c3 + 1 = 0 c3 =


y (0) = 0 6c4 + 1 = 0 c4 =

1 2
1 6

1 1 y (t ) = t t 2 t 3 + e t 2 6 t 4 y + y = t cos 2
1 1 Find yh: yh = c1 cos t + c2 sin t 2 2

53.

Find yp: y p1 = At + B t t y p2 = t c cos + D sin 2 2


t t y p (t ) = y p1 + y p2 = At + B + Ct cos + Dt sin 2 2

54.

y y = t 2 + et Find yh: r3 r2 = 0 r2(r 1) = 0 r = 0, 0, 1 yh = c1 + c2t + c3et

388

CHAPTER 4

Higher-Order Linear Differential Equations

Find yp: y p1 = t 2 ( At 2 + Bt + C ), y p2 = t ( Det ) y p (t ) = y p1 + y p2 = At 4 + Bt 3 + Ct 2 + Dtet


55.

y 5 y + 6 y = cos t tet Find yh: r2 5r + 6 = 0 (r 2)(r 3) = 0 r = 2, 3 yh = c1e2t + c2e3t Find yp: y p1 = A cos t + B sin t , y p2 = et (Ct + D) y p (t ) = y p1 + y p2 = A cos t + B sin t + et (Ct + D)

56.

y(4) y = tet + sin t r4 1 = 0 (r2 + 1)(r2 1) = 0 r = i, 1 yh = c1 cos t + c2 sin t + c3et + c4et y p1 = tet ( At + B ) , y p2 = t (C cos t + D sin t ) y p (t ) = et ( At 2 + Bt ) + Ct cos t + Dt sin t
Judicious Superposition

57.

(a)

The characteristic equation is r 2 r 6 = 0 has roots r = 3 , 2, so the general solution is yh ( t ) = c1e3t + c2 e2t .

(b)

(i)

Substituting y p ( t ) = Aet yields

Aet Aet 6 Aet = et ,


1 1 which yields A = . Hence, y p ( t ) = et . 6 6

(ii)

Substituting y p ( t ) = Ae t yields

Ae t + Ae t 6 Ae t = e t
1 1 or A = . Hence, y p ( t ) = e t . 4 4

SECTION 4.4

Undetermined Coefficients

389

(c)

Calling L ( y ) = y y 6 y we found in part (b) that 1 1 L et = et , and L e t = e t . 6 4 Multiplying each equation by


1 and using basic properties of derivatives yields 2

1 1 1 1 L et = et , and L et = e t 8 2 12 2 and
1 1 1 L et e t = et + e t = cosh t . 8 2 12

Hence, a solution of y y 6 y = cosh t is y p (t ) =


Wholesale Superposition 58.

1 t 1 t e e . 12 8

We first solve the equation y + y = first getting the homogeneous tn n!

yh ( t ) = ce t .
To find a particular solution, we try y (p ) ( t ) = An t n + An 1t n 1 + + A1t + A0 .
n

Substituting this into the equation yields tn nAn t n 1 + ( n 1) An 1t n 2 + + A1 + An t n + An 1t n 1 + + A1t + A0 = . n! Comparing coefficients, we have

390

CHAPTER 4

Higher-Order Linear Differential Equations

An = An 1 = An 2 = An 3 = and so on. Hence, we have


y (p ) =
n

1 n! 1 n 1)! ( 1 ( n 2 )! 1 , ( n 3 )!

tn t n 1 t n2 + . n ! ( n 1)! ( n 2 )!

Further, we have y (p ) ( t ) = 1
0

y (p ) ( t ) = t 1
1

t2 t +1 2! t3 t2 3 y (p ) ( t ) = + t 1 3! 2! t 4 t3 t 2 4 y (p ) ( t ) = + t + 1 4! 3! 2! y (p ) ( t ) =
2

y (p ) ( t ) =
n

t t t + n! ( n 1)! ( n 2 )!

n 1

n2

By superposition, the sum of these solutions is a solution of y + y = et . (We agree our discussion is formal in the sense that we have proven superposition for finite sums.) There is a slight problem in adding the preceding functions because the sum changes form depending on whether we add an even or odd number of terms. We have

t2 t4 t 2n 0 1 2n S 2 n = y (p ) ( t ) + y (p ) ( t ) + y (p ) ( t ) = 1 + + + + 2! 4! ( 2n )! t3 t5 t 2 n +1 0 1 2 n +1 S 2 n +1 = y (p ) ( t ) + y (p ) ( t ) + y (p ) ( t ) = t + + + + 3! 5! ( 2n + 1)!

SECTION 4.4

Undetermined Coefficients

391

However, because the sequence S n converges, it converges to the average of the nth and

( n + 1) st terms. That is,

1 1 1 t2 t3 ( S2 n + S2 n+1 ) = 1 + t + + + = et 2 2 2! 3! 2 1 Hence, we found y p ( t ) = et . 2


Discontinuous Forcing Functions 59.

2 y + y = 1

0t <4 t4

y (0) = y (0) = 0 y1(0) = y1 (0) = 0

Part 1: y1 + y1 = 2

Find (y1)h: r2 + r = 0 r(r + 1) = 0 r = 0, 1 (y1)h = c1 + c2et

Find (y)p: (y1)p = 2t by inspection, so y1 = c1 + c2et + 2t y1 = c2 e t + 2 y1 = 2 + 2et + 2t Part 2: y2 + y2 = 1 y2 = c1 + c2et + t y2 = c2et + 1 Thus, when t = 4, 6 + 2e 4 = c1 + c2 e4 + 4 2e 4 + 2 = c2 e 4 + 1 8 = c1 + 5 2e + 1 = c2e
4 4

0 = c1 + c2 0 = c2 + 2

c2 = 2, c1 = 2

y2(4) = y1(4) = 2 + 2e4 + 8 = 6 + 2e4 y2 (4) = y1 (4) = 2e 4 + 2

c1 = 3 2 + e4 = c2 c2 = 2 e4

y2 = 3 + (2e e4)et + t 2 + 2e t + 2t and y(t) = 4 t 3 + (2 e )e + t 0t <4 t4

392

CHAPTER 4

Higher-Order Linear Differential Equations

60.

cos t y + 16 y = 0

0t t >

y (0) = 1, y (0) = 0 0t y1(0) = 1, y1 (0) = 0

Part 1: y1 + 16 y1 = cos t

Find (y1)h: (y1)h = c1 cos 4t + c2 sin 4t Find (y1)p : (y1)p = A cos t + B sin t ( y1 ) = A cos t B sin t p Therefore, y1 = c1 cos 4t + c2 sin 4t +
B = 0, A =
1 15

1 cos t 15 1 sin t 15 c1 =

14 15

y1 = 4c1 sin 4t + 4c2 cos 4t y1(0) = 1 = c1 + y1 (0) = 0 = 4c2 y1(t) = 14 1 cos 4t + cos t 15 15 t>
1 15

c2 = 0

Part 2: y2 + 16 y2 = 0

y2 = c1 cos 4t + c2 sin 4t

y2 = 4c1 sin 4t + 4c2 cos 4t


y1() =
14 1 13 + = = y2 ( ) = c1 15 15 15

c1 = c2 = 1

13 15

y1 ( ) = 4 = y2 ( ) = 4c2 Thus y2(t) = 13 cos 4t + sin 4t 15


0t t >

1 14 15 cos 4t + 15 cos t and y(t) = 13 cos 4t + sin 4t 15

SECTION 4.4

Undetermined Coefficients

393

Solutions of Differential Equations Using Complex Functions 61.

y 2 y + y = 2sin t The homogeneous solution is yh = c1et + c2tet. For the particular solution we use y 2 y + y = 2eit and seek the imaginary part of the particular solution. We let yp = Aeit. Then y p = iAeit and y = Aeit. p By substitution, we obtain Aeit 2iAeit + Aeit = 2eit 2iA = 2 A=
1 =i i

yp = ieit = i(cos t + i sin t) = i cos t sin t Im(yp) = cos t y(t) = yh + Im yp = c1et + c2tet + cos t
62.

y + 25 y = 6sin t

We will use y + 25 y = 6eit

The homogeneous solution is yh = c1 cos 5t + c2 sin 5t For the particular solution we want Im(yp) where yp = Aeit; y p = iAeit and y = Aeit p By substitution, we obtain Aeit + 25Aeit = 6eit 24A = 6 so that A =
1 4

1 1 Im(yp) = Im eit = sin t 4 4 y(t) = c1 cos 5t + c2 sin 5t +


63.

1 sin t 4

y + 25 y = 20sin 5t

We will use y + 25 y = 20ei 5t

The homogeneous solution is yh = c1 cos 5t + c2 sin 5t For the particular solution we note that e5it is included in yh, so we must use an extra factor of t in yp. We want Im(yp) where yp = Ate5it, so y p = A(t 5ie5it + e5it ) , and y = A5i (t 5ie5it + e5it ) + Ai5e5it = A(10ie5it 25te5it ) . p

394

CHAPTER 4

Higher-Order Linear Differential Equations

By substitution, we obtain A(10ie 5i 25te5it ) + 25 Ate5it = 20e5it so that 10Ai = 20. Thus A = 2i and yp = 2ite5it. Im(yp) = Im ( 2it (cos5t + i sin 5t ) ) = 2t cos 5t y = c1 cos 5t + c2 sin 5t 2t cos 5t
Complex Exponents 64.

y 3 y + 2 y = 3e2it The homogeneous solution is yh = c1et + c2e2t. We seek a particular solution of the form yp = Ae2it. Then y = 2iAe 2it and y = 4 Ae 2it . By substitution, we obtain

4 Ae 2it 3(2iAe 2it ) + 2( Ae 2it ) = 3e 2it 2 Ae 2it 6 Aie 2it = 3e2it A(2 + 6i ) = 3 A= 3 2 6i 3 9 = + i 2 + 6i 2 6i 20 20

3 9 2it 3 9 yp = + ie = + i (cos 2t + i sin 2t 20 20 20 20 9 3 3 9 = cos 2t sin 2t + i cos 2t sin 2t 20 20 20 20 It can be verified directly by substitution that Re(yp) = 3 9 cos 2t sin 2t satisfies 20 20

y 3 y + 2 y = 3cos 2t
and that Im(yp) =
9 3 cos 2t sin 2t satisfies 20 20

y 3 y + 2 y = 3sin 2t
Suggested Journal Entry 65.

Student Project

SECTION 4.5

Variation of Parameters

395

4.5

Variation of Parameters

Straight Stuff 1.

y + y = 4t The homogeneous solutions to the equation are y1(t ) = 1 and y2 (t ) = et . To find a particular solution of the form y p ( t ) = v1 + v2 e t , we solve the equations v1 + e t v2 = 0 e t v = 0. for v1 , v2 . This gives Integrating yields v1 = 4t v1 ( t ) = 2t 2 and and v2 = 4tet . v2 ( t ) = 4et (1 t ) .

Hence, we have a particular solution y p ( t ) = y1v1 + y2 v2 = 1 2t 2 + e t 4et (1 t ) = 2t 2 4t + 4 . Combining the constant term with the homogeneous solution, we write the general solution as y(t ) = c1 + c2et + 2t 2 4t .
2.

( )

y y = et The homogeneous solutions to the equation are y1(t ) = 1 and y2 (t ) = et . To find a particular solution of the form y p ( t ) = v1 y1 + v2 y2 = v1 + v2 et , we solve
v1 + et v2 = 0 et v2 = e t .

This gives Integrating yields

v1 = e t
v1 ( t ) = e t

and and

v2 = e2t .
1 v2 ( t ) = e 2t . 2

Hence, we have a particular solution 1 1 1 y p ( t ) = y1v1 + y2 v2 = 1 e t + et e 2t = e t e t = e t . 2 2 2

( )

The general solution is 1 y ( t ) = c1 + c2 et + e t . 2

396

CHAPTER 4

Higher-Order Linear Differential Equations

3.

1 y 2 y + y = et , (t > 0) t The two linear independent solutions y1 and y2 of the homogeneous equation are y1(t ) = et and y2 (t ) = tet . Using the method of variation of parameters, we seek the particular solution y p ( t ) = v1 ( t ) et + v2 ( t ) tet . In order for y p (t ) to satisfy the differential equation, 1 and 2 must satisfy

y1v1 + y2 v2 = et v1tet v2 = 0 1 . y1v1 + y2 v2 = et v1et (t + 1)v = et 2 t Solving algebraically for v1 and v2 we obtain Integrating gives the values v1 ( t ) = 1 v1 ( t ) = t and and
1 v2 = . t

v2 = ln t .

Substituting these values into y p yields the particular solution


y p (t ) = tet + tet ln t .

Hence, the general solution is y(t ) = c1et + c2tet + tet ln t .


4.

y + y = csc t The two linearly independent solutions y1 and y2 of the homogeneous equation are
y1(t ) = cos t

and

y2 (t ) = sin t .

Using the method of variation of parameters, we seek the particular solution

y p ( t ) = v1 ( t ) cos t + v2 ( t ) sin t .
In order for y p (t ) to satisfy the differential equation, v1 and v2 must satisfy y1v1 + y2 v2 = (cos t )v1 + (sin t )v = 0 2 y1v1 + y2 v2 = ( sin t )v1 + (cos t )v = csc t. 2 Solving algebraically for v1 and v2 we obtain Integrating gives the values v1 ( t ) = 1 v1 ( t ) = t and and v2 = cot t . v2 = ln ( sin t ) .

Substituting these values into y p yields the particular solution


y p ( t ) = t cos t + sin t ln ( sin t ) .

Hence, the general solution is

y ( t ) = c1 cos t + c2 sin t t cos t + sin t ln ( sin t ) .

SECTION 4.5

Variation of Parameters

397

5.

y + y = sec t tan t
The homogeneous solutions are y1(t ) = cos t and y2 (t ) = sin t . We seek the solution We form the system

y p ( y ) = ( cos t ) v1 + ( sin t ) v2 .
(cos t )v1 (sin t )v2 = 0 (sin t )v1 + (cos t )v2 = sec t tan t. . v2 = tan t . v2 = ln sec t .

Solving algebraically for v1 and v2 yields Integrating gives the values The particular solution is Thus, the general solution is
6.

v1 ( t ) = tan 2 t and

v1 = tan t t

and

y p = ( tan t t ) cos t + sin t ln sec t = sin t t cos t + sin t ln sec t .


y(t ) = c1 cos t + c2 sin t t cos t + sin t ln sec t .

y 2 y + 2 y = et sin t The homogeneous solutions are y1(t ) = et cos t and y2 (t ) = et sin t . To find a particular solution of the form y p ( t ) = v2 et cos t + v2 et sin t , we solve the equations

( e cos t e sin t ) v + ( e sin t + e cos t ) v = e sin t


t t t t t 1 2

et cos tv1 + et sin tv2 = 0

for v1 and v2 . This yields

v1 = sin 2 t 1 ( t + cos t sin t ) 2

and and

v2 = sin t cos t . 1 v2 ( t ) = sin 2 t . 2

Integrating yields the functions v1 ( t ) = Hence, a particular solution

1 1 1 y p ( t ) = y1v1 + y2 v2 = et cos t ( t + cos t sin t ) + et sin t sin 2 t = et ( sin t t cos t ) 2 2 2 and the general solution is 1 y(t ) = c1et cos t + c2et sin t tet cos t . 2

398

CHAPTER 4

Higher-Order Linear Differential Equations

7.

y 3y + 2 y =

1 1 + et y p ( y ) = et v1 + e 2t v2 to form the system


et v1 + e 2t v2 = 0 et v1 + 2e2t v2 = 1 . 1 + e t e t 1 + et and v2 = e 2t . 1 + et

The homogeneous solutions are y1(t ) = et and y2 (t ) = e2t . Hence we seek the particular solution

Solving algebraically for v1 and v2 yields The first integral is trivial; v1 = ln(1 + e t ).

v1 =

The second one is more difficult. However, if we perform some algebra, we can write v2 = et 1 + et et e 2t et = = et , which integrates to give v2 = e t + ln 1 + e t . 1 + et 1 + et 1 + et

With 1 and 2 we have the particular solution

y p = et ln 1 + et et + e2t ln 1 + et and the general solution is

g
)

y ( t ) = c1et + c2 e2t + et + e 2t ln 1 + e t . (The term et in yp was absorbed in the homogeneous solution, giving a better form for the solution.)
8.

) (

y + 2 y + y = e t ln t , (t > 0) The homogeneous solutions are y1(t ) = et and y2 (t ) = tet . We seek a particular solution y p ( y ) = e t v1 + te t v2 to form the system

et v1 + te t v2 = 0 e t v1 + (e t tet )v = et ln t. 2 Solving algebraically for v1 and v2 , yields Integrating yields Hence, we have a particular solution 1 1 3 1 y p = t 2 e t ln t + t 2 e t + t 2 e t ln t t 2 e t = t 2 et ln t . 2 4 4 2 Thus the general solution is 1 y(t ) = c1et + c2tet + t 2et (2 ln t 3) . 4 v1 = t ln t and and v2 = ln t . v2 = t ln t t .

1 1 v1 = t 2 ln t + t 2 2 4

SECTION 4.5

Variation of Parameters

399

9.

y + 4 y = tan 2t
yh = c1 cos 2t + c2 sin 2t y1 y2

v1 =

( tan 2t )(sin 2t ) 1 sin 2 2t 1 1 cos 2 2t 1 dt = dt = dt = sec 2t cos 2t dt 2 2 cos 2t 2 cos 2t 2 1 = ( ln sec 2t + tan 2t sin 2t ) 4 tan 2t cos 2t 1 dt = cos 2t 2 4

v2 =

1 1 So yp = y1v1 + y2v2 = cos 2t ( ln sec 2t + tan 2t sin 2t ) sin 2t cos 2t . 4 4

General solution:
10.

y(t) = c1 cos 2t + c2 sin 2t + yp.

y + 5 y + 6 y = cos(et ) yh = c1 e 2t + c2 e3t y1 y2 v1 = v2 =

( cos(e ) ) e
t

3t

e5t

dt = e 2t cos(et )dt = et sin(et ) + cos(et )

e 2t cos(et ) dt = e3t cos(et )dt = 2sin(et ) e2t sin(et ) = 2et cos(et ) 5t e

So yp = y1v1 + y2v2 = e2t et sin(et ) + cos(et ) + e3t 2sin(et ) e2t sin(et ) 2et cos(et ) = 2e 3t sin(et ) e2t cos(et ) General solution:
11.

y(t) = c1e2t + c2e3t + yp.

y + y = sec 2 t yh = c1 cos t + c2 sin t y1 y2 v1 = ( sec2 t )sin dt = ( sec t tan t )dt = sec t v2 = sec2 t cos t dt = sec t dt = ln sec t + tan t So, yp = y1v1 + y2v2 = cos t sec t + sin t ln sec t + tan t = 1 + sin t ln sec t + tan t General solution: y (t ) = cos1 cos t + c2 sin t 1 + sin t ln sec t + tan t

400

CHAPTER 4

Higher-Order Linear Differential Equations

12.

y y =

et t

yh = c1 et + c2 e t y1 y2 v1 = 1 et t 1 1 1 t (e )dt = 2 t dt = 2 ln t 2 1 et t 1 e 2t 1 t e2 s dt = ds e dt = 2 t 2 t 2 t0 s
2s t e 1 t 1 e ln t e t ds t0 s 2 2

v2 =

So yp = v1y1 + v2y2 = General solution:

y(t) = c1et + c2et + yp

Variable Coefficients 13.

t 2 y 2ty + 2 y = t 3 sin t , y1(t ) = t , y2 (t ) = t 2 We begin by dividing the equation by t 2 , to get the proper form for using variation of parameters. y 2 2 y + 2 y = t sin t . t t

Substitution verifies that y1 and y2 for a fundamental set of solution to the associated homogeneous equation, so yh = c1t + c2t 2 , we seek a particular solution

y p ( y ) = v1t + v2t 2 ,
tv1 + t 2 v2 = 0 v1 + 2t 2 v2 = t sin t

where v1 and v2 satisfy the conditions

Solving algebraically for v1 and v2 , yields


Integrating yields Thus, y p (t ) = t 2 cos t t sin t t 2 cos t = t sin t .

v1 = t sin t v1 = t cos t sin t

and and

v2 = sin t . v2 = cos t .

Hence, the general solution of this equation is


14.

t 2 y + ty 4 y = t 2 1 + t 2 , y1(t ) = t 2 , y2 (t ) = t 2 We begin by dividing the equation by t 2 , to get the proper form for using variation of parameters: 1 4 y + y 2 y = 1 + t 2 . t t

y(t ) = c1t + c2t 2 t sin t .

SECTION 4.5

Variation of Parameters

401

Substitution verifies that y1 and y2 form a fundamental set of solutions to the associated homogeneous equation, so yh = c1t 2 + c2t 2 . We seek a particular solution y p ( y ) = v1t 2 + v2 t 2 , t 2 v1 + t 2 v2 = 0 2tv1 2t 3v2 = 1 + t 2 . Solving algebraically for v1 and v2 yields Integrating yields Thus,
y p (t ) = 1 2 1 t2 t4 t ln t + t 4 . 4 8 16 24 1 1 y t = c1t 2 + c2t 2 + t ln t + t 4 . 4 12

where 1 and 2 satisfy the conditions

1+ t2 v1 = 4t

and and

v2 =

t 3 1 + t 2 4

).

1 1 v1 = ln t + t 2 4 8

v2 =

1 4 1 6 t t . 16 24

Hence, the general solution of this equation is

bg

(Notice that the term t 2 in yp can be absorbed in the homogeneous solution.)


15.
(1 t ) y + ty y = 2(t 1)2 et , y1(t ) = t , y2 t = et

bg

We begin by dividing the equation by (1 t ) , to get the proper form for variation of parameters
y + 1 t y y = 2 t 1 e t 1 t 1 t

b g

Susbtitution verifies that y1 and y2 form a fundamental set of solutions to the associated homogeneous equation, so yh = c1t + c2 et We seek a particular solution y p ( y ) = v1t + v2 et , t v1 + et v2 = 0 v1 + et v2 = 2(t 1)et

where v1 and v2 satisfy the conditions

Solving algebraically for v1 and v2 yields


yields v1 = 2e t

v1 = 2e t and

and

v2 = 2te 2t . Integrating

1 v2 = e 2t t + . 2

Thus,

1 1 y p ( t ) = 2tet + te t + e t = e t t . 2 2 y(t ) = c1t + c2et + et

Hence, the general solution of this equation is

F 1 tI . H2 K

402

CHAPTER 4

Higher-Order Linear Differential Equations

16.

1 1 y + y + 1 2 y = t 1 2 , t 4t

F H

I K

y1 ( t ) = t 1/ 2 sin t , y2 ( t ) = t 1/ 2 cos t

Substitution verifies that y1 and y2 form a fundamental set of solutions to the associated homogeneous equation, so yh = c1t 1/ 2 sin t + c2t 1/ 2 cos t We seek a particular solution y p ( y ) = v1t 1/ 2 sin t + v2t 1/ 2 cos t , t 1/ 2 sin tv1 + t 1/ 2 cos v2 = 0

where v1 and v2 satisfy the conditions

1 3/ 2 1 sin t + t 1/ 2 cos t v1 + t 3/ 2 cos t t 1/ 2 sin t v2 = t 1/ 2 . 2t 2

Multiplying through by t1/ 2 then solving for v1 and v2 : v1 = cos t v1 = sin t


Thus,

and and

v2 = sin t. v2 = cos t.

y p ( t ) = t 1/ 2 sin 2 t + cos 2 t = t 1/ 2 , and the general solution of this equation is y ( t ) = yh + y p = c1t 1/ 2 sin t + c2t 1/ 2 cos t + t 1/ 2 .

Third-Order Theory 17.

L( y ) = y + p(t ) y + q(t ) y + r (t ) y = f (t ) Given we seek Differentiating yields

yh (t ) = c1 y1 + c2 y2 + c3 y3 ,
y p (t ) = v1 y1 + v2 y2 + v2 y3 . y p = v1 y1 + v1 y1 + v2 y2 + v2 y2 + v3 y3 + v3 y3 = = v1 y1 + v2 y2 + v3 y3 (if we set y1v1 + y2 v2 + y3v3 = 0 ).

Differentiating, again

y = v1 y1 + v1 y1 + v2 y2 + v2 y2 + v3 y3 + v3 y3 p = v1 y1 + v2 y2 + v3 y3 (if now we set y1v1 + y2 v2 + y3v3 = 0 ).

Differentiating yet again:

y = v1 y1 + v1 y1 + v2 y2 + v2 y2 + v3 y3 + v3 y3 . p

Substituting y p , y , y and y into the L( y) = f , then regrouping all terms in v1 and v2, we see p p p that the coefficient of each is 0 because each yi is a solution of L(yi) = 0. Thus we are left with y = y1v1 + y2 v2 + y3v3 = f . p This last equation, together with the two assumptions (in parentheses) that we made while differentiating, gives a system to solve for v1 , v2 , v3 : y1v1 + y2 v2 + y3v3 = 0 y1v1 + y2 v2 + y3v3 = 0 y1v1 + y2 v2 + y3v3 = f .

SECTION 4.5

Variation of Parameters

403

We use Cramers Rule to solve the system, then integrate to find v1 , v2 , v3 and hence, obtain a particular solution

y p ( t ) = v1 y1 + v2 y2 + v3 y3 .
Third-Order DEs 18.

y 2 y y + 2 y = et The characteristic equation ( 1)( + 1)( 2 ) = 0 and has roots 1, 1, and 2. The fundamental set is y1 = et , y2 = et , and y3 = e2t . Hence, yh = c1et + c2et + c3e2t . By variation of parameters, we seek y p (t ) = v1et + v2 e t + v3e2t , as in Problem 17. Hence the system to solve is et v1 + e t v2 + e 2t v3 = 0 et v1 e t v2 + 2e 2t v3 = 0 et v1 + e t v2 + 4e 2t v3 = et . Using Cramers rule and computing the determinants yields:
0 e t t 0 e t e t e v1 = W e 2t 2e 2 t 4e 2t 3e 2t 1 = = 2t 2 6e

et W = et t e

et e t et

e 2t 2e2t = 6e2t ; 4e 2 t

v2 =

et t e t e

et t e t e v3 = Hence we obtain Hence,


1 2 t v1 = 2 v1 =

e 2t 0 2e 2t et 4e 2t e 4t 1 = = e 2t 2t W 6 6e t e 0 t 0 e e t et 2et 1 t = = e W 6e 2t 3 0

We get a particular solution of

1 1 v2 = e 2t v3 = e t . 6 3 1 1 v2 = e 2t v3 = e t . 12 3 1 1 1 y p (t ) = tet + et et and the general solution is 2 12 3 1 1 y(t ) = c1et + c2et + c3e2t tet et . 2 4

404

CHAPTER 4

Higher-Order Linear Differential Equations

19.

y + y = sec t Find yh: yh = r3 + r = 0 r(r2 + 1) = 0 r = 0, i

yh = c1 + c2 cos t + c3 sin t yp = v1 + v2 cos t + v3 sin t 1 cos t W = 0 sin t 0 cos t


0 0 sec t

sin t cos t = 1 sin t


sin t cos t sin t

v1 =

cos t sin t cos t 1

= sec t

cos t sin t = sec t sin t cos t

v = ln sec t + tan t

v2 =

1 0 0 0 0 sec t 1

sin t cos t sin t

=1

0 sec t

cos t cos t = 1 = 1 sin t sec t

v2 = t 1 cos t 0 0 sin t 0 0 cos t sec t 1

v3 =

=1

sin t 0 sin t = sin t sec t = cos t sec t cos t

v3 = ln cos t y(t) = c1 + c2 cos t + c3 sin t + ln sec t + tan t t cos t + sin t ln cos t


20.

y + 9 y = tan 3t Find yh: r3 + 9r = 0 r(r2 + 9) = 0 r = 0 3i

yh = c1 + c2 cos 3t + c3 sin 3t yp = v1 + v2 cos 3t + v3 sin 3t 1 cos3t W = 0 3sin 3t 0 9cos3t sin 3t 3sin 3t 3cos3t = 1 9cos3t 9sin 3t 3cos3t = 27 9sin 3t

SECTION 4.5

Variation of Parameters

405

v1 =

0 cos3t sin 3t 3sin 3t 3cos3t 0 tan 3t 9cos3t 9sin 3t 27 ln cos3t 27


1 0 0 0 0 tan 3t 27 1 cos3t 27 1 cos3t 0 3sin 3t 0 9cos3t 27 0 0 tan 3t sin 3t 3cos3t 9sin 3t

sin 3t tan 3t cos3t tan 3t tan 3t = (3cos 2 3t + 3sin 2 3t ) = 27 3sin 3t 3cos3t 27 9

v1 =

v2 = v2 =

3cos3t tan 3t 1 = sin 3t 27 9

v3 =

3sin 3t tan 3t 1 sin 2 3t = 27 9 cos3t

v3 =

1 1 cos 2 3t 1 cos3t dt = 27 ( ln sec3t + tan 3t sin 3t ) 9


1 1 sin 3t ln cos3t + cos 2 3t ( ln sec3t + tan 3t sin 3t ) 27 27 27

y = c1 + c2 cos3t + c3 sin 3t
Method Choice 21.

y y = f (t ) We first find the homogeneous solution. The characteristic equation ( 2 1) = 0 has roots 0, 1, so the homogeneous solution is yh = c1 + c2 et + c3e t . (a) y y = 2e t . Because et is in yh, we must try yp = a tet. The method of undetermined coefficients is straightforward and gives a = 1, so yp = tet and the general solution can be written y (t ) = c1 + c2 et + c3e t + te t . (b) y y = sin 2 t . We cannot use undertermined coefficients on sin2 t, so we use variation of parameters to seek a particular solution of the form yp(t) = v1 + v2et + v3et, with the derivatives of v1, v2, and v3 determined from the equations 1v1 + et v2 + e t v3 = 0 0v1 + et v2 e t v3 = 0 0v1 + et v2 + e t v3 = sin 2 t . Discussion continues on next page.

406

CHAPTER 4

Higher-Order Linear Differential Equations

Using Cramers rule (as outlined in Problem 18), we obtain v1 = sin 2 t v2 = 1 t 2 e sin t 2 1 v3 = et sin 2 t . 2

The antiderivative of 1 is easy to find; the other two must be left as integrals
v1 = 1 ( sin t cos t t ) 2 1 v2 = e t sin 2 t dt 2 1 v3 = et sin 2 t dt . 2

Hence, the general solution is y ( t ) = c1 + c2 et + c3e t + (c) 1 1 1 ( sin t cos t t ) + et et sin 2 t dt + et et sin 2 t dt . 2 2 2

y y = tan t . As in Part (b) , we must use variation of parameters to find yp, with
1v1 + et v2 + e t v3 = 0 0v1 + et v2 e t v3 = 0 0v1 + et v2 + e t v3 = tan t . Using Cramers rule (as outlined in Problem 18), to solve these equations we find v1 = tan t 1 v2 = e t tan t 2 1 v3 = et tan t . 2

The antiderivative of 1 is easy to find; the other two must be left as integrals v1 = ln cos t Hence, the general solution is
1 1 1 y ( t ) = c1 + c2 et + c3et + ln cos t + et et tan t dt + e t et tan t dt. 2 2 2

1 v2 = e t tan t dt 2

1 v3 = et tan t dt . 2

Parts (b) and (c) demonstrate the power of graphical methods because the algebraic expressions for y(t) are pretty meaningless. It is easier and more informative to use DE software to approximate solutions of this equation in ty space than it is to pursue the analytical formula for the solution. The figures show curves for several initial conditions to show the variety that can occur. For any IVP there would be only one solution. Note: We used a 3D graphc DE solver with the following equations for y y = f (t ) :

y = x y = x = z y = z = x + f (t )
relisted as

x = y y = x z = x + f (t )

SECTION 4.5

Variation of Parameters

407

(b)

f(t) = sin2 t. The expression for y(t) on the previous page can be further evaluated using the identity sin2 t = 1 (1 cos 2t ), 2

but solution behavior is more easily seen on a graph of y(t). (c) f(t) = tan t The expression for y(t) on the previous page is even more complicated than that for part (b); again, solution behavior is more readily understood with a graph of y(t).
Greens Function Representation 22.

y + y = f (t )
We know that y1 = cos t and y2 = sin t are the solutions of the corresponding homogeneous equation. Their Wronksian is W y1, y2 (t ) =

cos t sin t = 1. sin t cos t


y1 ( t ) f ( t )

which is makes it easy to use the suggested variation of parameters formulas


v1 = W ( y1 , y2 )( t ) y2 ( t ) f ( t ) = sin(t ) f (t ), v2 = W ( y1 , y2 )( t ) = cos(t ) f (t ).

Integrating yields
v1 = sin ( s ) f ( s ) ds
t 0

v2 = cos ( s ) f ( s ) ds .
t 0

Hence, y p ( t ) = y1v1 + y2 v2

= cos(t ) sin( s ) f ( s )ds + sin(t ) cos( s ) f ( s ) ds


0 0

= [ cos(t )sin( s ) + sin(t ) cos( s ) ] f ( s )ds


t 0 t

= sin(t s ) f ( s )ds .
0

Green Variation 23.

The homogeneous solutions are y1 = et and y2 = et . We seek a particular solution of the form y p = v1 y1 + v1 y2 , where v1 and v2 satisfy et v1 + e t v2 = 0 et v1 e t v2 = f ( t ) .

408

CHAPTER 4

Higher-Order Linear Differential Equations

Adding and subtracting the equations and solving yields 1 v1 = e t f ( t ) 2 1 t v1 = e s f ( s ) ds 2 0 y p = v1 y1 + v2 y2 1 v2 = et f ( t ) 2 1 t v2 = e s f ( s ) ds . 2 0

Integrating gives Hence,

t t 1 1 = et e s f ( s ) e t e s f ( s ) ds 0 2 0 2 t et s e t + s = f (s) ds 0 2

= sinh ( t s ) f ( s ) ds .
t 0

Greens Follow-Up 24.

From the Leibniz Rule in multivariable calculus we have the following result: For a continuous function g(t,s),
r g d t 0 g ( t , s ) ds = limt g (r , r ) + 0 t (t , s)ds . r dt

In Problem 22, the solution of the equation y + y = f (t ) is


y (t ) =

z
t

sin(t s) f (s)ds

Differentiating yields
y = sin ( t t ) f ( t ) + cos ( t s ) f ( s ) ds = cos ( t s ) f ( s ) ds
t t 0 0

and
y = cos ( t t ) f ( t ) sin ( t s ) f ( s ) ds = f ( t ) sin ( t s ) f ( s ) ds .
t 0 0

Hence,
y + y = f (t )

sin(t s) f ( s)ds +

sin(t s) f (s)ds = f (t ) .

Suggested Journal Entry I 25.

Student Project
Suggested Journal Entry II

26.

Student Project

SECTION 4.6

Forced Oscillations

409

4.6

Forced Oscillations

Mass-Spring Problems 1.
x + 2 x + x = 6cos t

Find xh:

r2 + 2n + 1 = 0 (r + 1)2 = 0 r = 1 xh = c1et + c2tet.

Find xp:

xp = A cos t + B sin t, xp = A sin t + B cos t , x = A cos t B sin t p

x + 2 xp + x p = A cos t B sin t p + 2( B cos t A sin t ) + A cos t + B sin t 2 B cos t 2 A sin t = 6cos t 2B = 6, 2A = 0 A = 0, B = 3 xp = 3 sin t x(t) = xh + xp = c1et + c2tet + 3 sin t
xss = 3 sin t = 3cos t 2 Amplitude C = 3; phase shift

= radians 2

2.

x + 2 x + 3 x = cos3t

Find xh:

r2 + 2r + 3 = 0 r = 1

2i

xh = e t c1 cos 2t + c2 sin 2t
Find xp:

xp = A cos 3t + B sin 3t, xp = 3 A sin 3t + 3B cos3t , x = 9 A cos3t 9 B sin 3t p x + 2 xp + 3 x p = p

9A cos3t 9 B sin 3t ) + 2(3B cos3t 3 A sin 3t ) + 3( A cos3t + B sin 3t ) (6 A + 6 B )cos3t + ( 6 A 6 B)sin 3t = cos3t

6A + 6B = 1, 6A 6B = 0 A=
1 1 ,B = , so 12 12

410

CHAPTER 4

Higher-Order Linear Differential Equations

xp =

1 1 cos3t + sin 3t 12 12

x(t) = xh + xp = e t c1 cos 2t + c2 sin 2t xss =

1 1 cos3t + sin 3t 12 12

1 1 2 3 cos3t + sin 3t = cos 3t 12 12 12 4

Amplitude C =
3.

2 ; phase shift = radians 12 4

2 x + 3 x = 4cos8t

Find xh:

2r2 + 3 = 0 r2
xh = c1 cos

3 3 r= i 2 2

3 3 t + c2 sin t 2 2

Find xp:

xp = A cos 8t, xp = 8 A sin 8t , x = 64 A cos8t p


2 x + 3 x p = 2(64 A cos8t ) p +3( A cos8t ) 125 A cos8t = 4cos8t 125A = 4 A= 4 125
4 cos8t 125

xp = x(t) = xh + xp = c1 cos xss =

3 3 4 t + c2 sin t cos8t 2 2 125

4 4 cos8t = cos(8t ) 125 125


4 ; phase shift = radians 8 125

Amplitude C =

SECTION 4.6

Forced Oscillations

411

4.

2 x + 2 x + Find xh:

1 5 x = cos t 2 2 2r2 + 2r +
1 1 =0r= 2 2

xh = c1e (1/ 2)t + c2te (1/ 2)t Find xp: xp = A cos t + B sin t, xp = A sin t + B cos t , x = A cos t B sin t p

2 x + 2 xp + p

1 x p = 2( A cos t B sin t ) 2 + 2( B cos t A sin t ) 1 + ( A cos t + B sin t ) 2 3 5 3 2 A + 2 B cos t + 2 A 2 B sin t = 2 cos t

3 5 3 A + 2B = , 2 A B = 0 2 2 2 3 4 A= , B= 5 5 3 4 xp = cos t + sin t 5 5 3 4 x(t) = xh + xp = c1e (1/ 2)t + c2te (1/ 2)t cos t + sin t 5 5 3 4 xss = cos t + sin t = cos(t 2.2) 5 5 Amplitude C = 1; phase shift

2.2 radians

5.

x + 2 x + 2 x = 2cos t

Find xh:

r2 + 2r + 2 = 0 r = 1 i xh = et ( c1 cos t + c2 sin t )

Find xp:

xp = A cos t + B sin t, xp = A sin t + B cos t x = A cos t B sin t p x + xp + 2 x p = A cos t B sin t p +2( B cos t A sin t ) +2( A cos t + B sin t ) ( A +2 B) cos t + (2 A + B )sin t = 2cos t A + 2B = 2, 2A + B = 0

412

CHAPTER 4

Higher-Order Linear Differential Equations

A=

2 4 , B= 5 5 2 4 xp = cos t + sin t 5 5

2 4 x(t) = xh + xp = e t (c1 cos t + c2 sin t ) + cos t + sin t 5 5 xss =


2 4 2 cos t + sin t = cos(t 1.1) 5 5 5

Amplitude C =
6.

2 5

; phase shift

1.1 radians

x + 4 x + 5 x = 2cos 2t
Find xh:

r2 + 4r + 5 = 0 r = 2 i xh = e 2t (c1 cos t + c2 sin t )

Find xp:

xp = A cos 2t + B sin 2t, xp = 2 A sin 2t + 2 B cos 2t , x = 4 A cos 2t 4 B sin 2t p x + 4 xp + 5 x p = p


4 A cos 2t 4 B sin 2t ) + 4(2 B cos 2t 2 A sin 2t ) + 5( A cos 2t + B sin 2t ) ( A + 8 B )cos 2t + (8 A + B)sin 2t = 2cos 2t A + 8B = 2, 8A + B = 0 A=
2 16 , B= 65 65

xp =

2 16 cos 2t + sin 2t 65 65
2 16 cos 2t + sin 2t 65 65

x(t) = xh + xp = e 2t (c1 cos t + c2 sin t ) + xss =


2 16 cos 2t + sin 2t = 65 65 2 65 2 65

cos(2t 1.4)

Amplitude C =

; phase shift

0.73 radians

SECTION 4.6

Forced Oscillations

413

Pushing Up 7.

m=

1 , b = 2.5, k = 6 4

1 x + 2.5 x + 6 x = 2cos 2t 4

The IVP is x + 10 x + 24 x = 8cos 2t , x(0) = 2, x(0) = 0. Find xh:

r2 + 10r + 24 = 0
(r + 4)(r + 6) = 0

r = 4, 6

xh = c1e4t + c2e6t
Find xp:

xp = A cos 2t + B sin 2t x p = 2A sin 2t + 2B cos 2t x p = 4 A cos 2t 4 B sin 2t

x p + 10 x p + 24 x p = 4 A cos 2t 4 B sin 2t + 10(2 A sin 2t + 2 B cos 2t ) + 24( A cos 2t + B sin 2t )


coeff. of cos 2t: 4A + 24A + 20B = 8 20A + 20 B = 8 20A + 20B = 0

coeff. of sin 2t: 4B + 24B 20A = 0 A=B= 1 5

xp =

1 1 cos 2t + sin 2t 5 5

Therefore 1 1 x(t) = xh + xp = c1e 4t + c2 e 6t + cos 2t + sin 2t 5 5


2 2 x = 4c1e 4t 6c2 e 6t sin 2t + cos 2t 5 5

Substituting initial conditions gives

1 34 5 c1 = , 2 5 x0 (0) = 0 = 4c 6c2 + 5 x(0) = 2 = c1 + c2 + Thus x(t) =


34 4t 23 6t 1 1 e + e + cos 2t + sin 2t . 5 5 5 5

c2 =

23 5

414

CHAPTER 4

Higher-Order Linear Differential Equations

Pulling Down 8.

m=

16 1 = , b = 6, k = 16 32 2

1 x + 6 x + 16 x = 4cos 4t 2

The IVP is x + 12 x + 32 x = 8cos 4t , x (0) = 1, x (0) = 0 . Find xh: r2 + 12r + 32 = 0 (r + 4)(r + 8) = 0 xh = c1e4t + c2e8t Find xp: xp = A cos 4t + B sin 4t x p = 4A sin 4t + 4B cos 4t x p = 16 A cos 4t 16 B sin 4t x p + 12 x p + 32 x = 16 A cos 4t 16 B sin 4t + 12(4 A sin 4t + 4 B cos 4t ) + 32( A cos 4t + B sin 4t ) = 8cos 4t Coeff. of cos 4t: 16A + 32A + 48B = 8 Coeff. of sin 4t: 16B + 32B 48A = 0 A= xp =
1 3 cos 4t + sin 4t 20 20

r = 4, 8

16A + 48B = 8 48A + 16B = 0

1 3 B= 20 20

Therefore x(t) = xh + xp = c1e 4t + c2 e 8t +


1 3 cos 4t + sin 4t 20 20

1 3 x = 4c1e 4t 8c2 e 8t sin 4t + cos 4t 5 5 Substituting intial conditions,

1 = c1 + c2 +

1 20 3 5

0 = 4c1 8c2 +

19 7 4 20 c1 = , c2 = 3 4 5 c1 + 2c2 = 20 c1 + c2 =

7 4 1 3 Thus x(t) = e 4t e 8t + cos 4t + sin 4t . 4 5 20 20

SECTION 4.6

Forced Oscillations

415

Mass-Spring Again 9.

(a)

The mass is m = 100 kg ; gravitational force (weight) acting on the spring is mg = 100 ( 9.8 ) = 980 newtons. Because the weight stretches the spring by

20 cm = 0.2 m , we have
k=

980 = 4900 nt m . 0.20

(b)

The initial-value problem for this mass is

x + 49 x = 0 , x ( 0 ) = 0.40 , x ( 0 ) = 0 .
Solving we write the transient solution in polar form
x ( t ) = C cos (0t ) = C cos ( 7t )

where the circular frequency is 0 = 7 radians per second. Using the initial conditions gives

x ( 0 ) = C cos = 0.4 x ( 0 ) = 7C sin = 0 or = 0 , C = 0.4 . Hence,


x ( t ) = 0.4cos ( 7t ) .

(c) (d)

Amplitude: C = 0.4 meter; period: T = If b = 500 , then

2 seconds . 7

b 2 4mk = 250,000 4 (100 )( 4900 ) < 0 .


The system is underdamped. (e)

100 x + 500 x + 4900 x = 0 has characteristic equation


r 2 + 5r + 49 = 0 ,

5 1 which has roots x1,2 = i 171 . Hence, the general solution is 2 2 171 171 x ( t ) = e5t 2 c1 cos t + c2 sin 2 2 t . Using the initial conditions x ( 0 ) = 0.4 , x ( 0 ) = 0 gives
x ( 0 ) = c1 = 0.4 x ( 0 ) = c2 171 5 c1 = 0 2 2

416

CHAPTER 4

Higher-Order Linear Differential Equations

which implies c1 = 0.4 , c2 =

2 171 . Hence, the solution is 171

171 2 171 171 sin x ( t ) = e 5t 2 0.4cos t+ t . 2 171 2


Adding Forcing 10.

We change the unforced equation in Problem 9 to the forced equation 100 x + 500 x + 4900 x = 100cos f t . (a)

b 2 4mk = 5002 4 (100 )( 4900 ) < 0 , so the system is underdamped. From Equation (21) in the text, the amplitude is a maximum when the forcing frequency is

f =
(b)

k b2 4900 5002 = = 6.04 rad sec . m 2m 2 100 2 (100 )2

Given f = 7 , we have seen that 0 = 7 , m = 100 , b = 500 , and hence tan becomes infinite, so that =
xss ( t ) =

2
2

radians. Hence, by Equation (17)


F0 cos f t

m
2 0

2 f

) + (b )
2 f

100

(100 )2 ( 49 49 )2 + ( 500 7 )2

cos 7t 0.029cos 7t . 2 2

See the graph for the solution of the IVP problem. You can see that the steady state appears to be

xss ( t ) 0.029cos 7t . 2

(c)

The undamped equation is 100 x + 4900 x = 100cos 7t . The particular solution now has the form xss ( t ) = Ct cos ( 7t ) or xss ( t ) = t ( A cos 7t + B sin 7t ) .

SECTION 4.6

Forced Oscillations

417

Electric Analog 11.

From Problem 10 we found the differential equation for the mechanical system to be 100 x + 500 x + 4900 x = 100cos f t or x + 5 x + 49 x = cos f t . So if R = 4 ohms , then the equivalent electrical equation (making one equation a constant multiple of the other equation) LQ + RQ + would be:
4 4 4 4 Q + ( 5 ) Q + ( 49 ) Q = cos f t . 5 5 5 5

1 Q = V0 cos f t C

This means we have


L = 0.80 henries 1 1 = 39.2 C = 0.025 farads 39.2 C V ( t ) = 0.80cos f t .
Damped Forced Motion I 12.

x + 8 x + 36 x = 72cos 6t
The given characteristic equation has roots 4 2i 5 , hence in the long run the homogeneous
equation solution always decays to zero. We are only interested in a particular solution, and in this case that solution is

x = A cos 6t + B sin 6t .
Differentiating and substituting into the differential equation gives
3 . 2
x 2 1 t

A= 0, B =

Hence, the steady-state solution is given by 3 xss ( t ) = sin 6t . 2 The graph of the steady-state solution is shown.
1 2

418

CHAPTER 4

Higher-Order Linear Differential Equations

Damped Forced Motion II 13.

The initial-value problem x + 4 x + 20 x = 20cos 2t , x ( 0 ) = x ( 0 ) = 0 , has xh ( t ) = e 2t ( c1 cos 4t + c2 sin 4t ) , and

x p ( t ) = A cos 2t + B sin 2t .
Substituting x p into the differential equation we find A = 1 , B = 1 x p = cos 2t + sin 2t . 2 Substituting the general solution into the initial conditions x ( 0 ) = x ( 0 ) = 0 , we find 3 1 x ( t ) = e 2t cos 4t + sin 4t + cos 2t + sin 2t . 4 2 The steady-state portion of the solution is shown. (See figure.)
x(t ) 1.5 0.5 0.5 1.5 1 2 3 4 5 6 t

1 , so 2

Calculating Charge 14.

4Q + 100Q = 10cos 4t , Q (0) = 0, Q(0) = 0 Find Qh : Find Qp : 4Q + 100Q = 0 Q + 25Q = 0 Qh = c1 cos5t + c2 sin 5t Qp has the form A cos 4t + B sin 4t.
5 5 cos 4t. , B = 0 and so Qp = 18 18

Substitution in 4Q p + 100Q p = 10cos 4t leads to A = Thus Q = Qh + Qp = c1 cos 5t + c2 sin 5t + 5 cos 4t . 18

The initial conditions Q(0) = 0 and Q(0) = 0 give us c1 = and the solution of the IVP is Q(t) =
5 5 cos5t + cos 4t . 18 18

5 , c2 = 0 18

SECTION 4.6

Forced Oscillations

419

Charge and Current 15.

Q + 12Q + 100Q = 12cos10t , Q (0) = 0, Q(0) = 0 (a) Find Qh: Find Qp: r 2 + 12r + 100 = 0 r = 6 8i Qh = e 6t (c1 cos8t + c2 sin 8t ). Qp has the form A cos 10t + B sin 10t. 1 10

Substitution in 4Q p + 12Q p + 100Q p = 12cos10t leads to A = 0, B = and so Qp =


1 sin10t. 10

Thus Q = Qh + Qp = e 6t (c1 cos8t + c2 sin 8t ) +

1 sin10t. 10
1 8

The initial conditions Q(0) = 0 and Q(0) = 0 give us c1 = 0 c2 = 1 1 and the solution of the IVP is Q(t) = e 6t sin 8t + sin10t . 8 10 (b) 3 I(t) = Q(t ) = e 6t sin 8t cos8t + cos10t. 4

True/False Questions 16. True

The steady-state solution, being a particular solution, has the form A cos f t + B sin f t, where the forcing function is Fo cos f t. The steady-state solution can be written in the form xss = C cos(f t ). Hence, the frequency of the steady-state is the same as that of the forcing function.
17. False

The amplitude of the steady-state is a function of the frequency of the forcing function. In fact, A( f ) =

F0 m ( 2 ) 2 + (b f ) 2 f
2 2 0

We can see that A(f) 0 as f and that F0 as f 0. k

420

CHAPTER 4

Higher-Order Linear Differential Equations

Beats 18.

The identity cos ( A + B ) cos ( A B ) = 2sin A sin B may be used here. In this case, if A = 2t , B = t , and we have A + B = 3t , A B = t . Hence, cos3t cos t = 2sin 2t sin t .

The Beat Goes On 19.

The trigonometric identity sin ( A + B ) sin ( A B ) = 2sin B cos A may be used here. Let A = 3t and B = t . From this we get A + B = 3t and A B = 2 . Hence,

3t t 3t + t sin 3t sin t = 2sin cos 2 2 = 2sin t cos 2t.


Steady State Note: We must be careful in finding the phase angle using the formula

= tan 1

B A
B . The value of you get A

because we dont know in which quadrant lies using = tan 1

might be units different from the correct value. Unless you know by some other means in which quadrant lies, it is best to use the two equations C cos = A , C sin = B . A good rule of thumb is to think of the AB plane; when both A, B are positive will be in the first quadrant (i.e., between 0 and quadrant, and so on.

), but when both A and B are negative will be in the third

SECTION 4.6

Forced Oscillations

421

20.

x + 4 x + 4 x = cos t The homogeneous solution to the equation is xh ( t ) = c1e 2t + c2te 2t . We use the method of undetermined coefficients to find x p ( t ) = A cos t + B sin t . Differentiating, we get
xp ( t ) = A sin t + B cos t x ( t ) = A cos t B sin t . p Substituting into the equation gives the equation x + 4 xp + 4 x p = ( A + 4 B + 4 A ) cos t + ( B 4 A + 4 B ) sin t = cos t . p Hence A = 3 4 , B= with the particular solution 25 25 xp (t ) = 3 4 cos t + sin t . 25 25

Nothing in xp dies off with time; this is our steady-state solution. Putting this in polar form

5 1 3 4 = C = + = 25 5 25 25 4 = tan 1 0.93 radians. 3 Hence, the steady-state solution is

x p (t ) = xss ( t ) = 0.20cos ( t 0.93) .

21.

x + 2 x + 2 x = 2cos t The roots of the characteristic equation are 1 i . We use for the particular solution: x p ( t ) = A cos t + B sin t = C cos ( t ) .

422

CHAPTER 4

Higher-Order Linear Differential Equations

Another approach to x p is to note that F0 = 2 ,

0 = 2 , f = 1 , m = 1 , b = 2 and simply
substitute these numbers into the text solution to find xss ( t ) = 2 5 cos ( t )

where the phase angle is = tan 1 ( 2 ) 1.1 radians.


22.

x + x + x = 4cos3t 1 i 3 . Notice that, as in previous problems, there The roots of the characteristic equation are 2 2 will be an e t 2 involved in all of the terms in the homogeneous equation, so xh is transient, and none of these terms will be involved in x p . We move on to find a particular solution, using the method of undetermined coefficients. Let

x p ( t ) = A cos3t + B sin 3t .
Then we have xp = 3 A sin 3t + 3B cos3t , x ( t ) = 9 A cos3t 9 B sin 3t . p Hence,

x + xp + x p = ( 8 A + 3B ) cos3t + ( 3 A 8B ) sin 3t = 4cos3t . p


Solving we get
A= 32 12 , B= . 73 73

In polar coordinates we have C = A2 + B 2 = 1168 4 = 73 73

= arctan

12 2.78 radians. 32

Hence, the steady-state solution is x p (t ) = xss ( t ) = 4 73 cos ( 3t 2.78 ) .

SECTION 4.6

Forced Oscillations

423

Resonance 23.

The differential equation is given by x + 12 x = 16cos t . The circular frequency is

0 =
the frequency is

k = 12 = 2 3 radians per second, m

f0 =

3 oscillations per second,

and the period of oscillations is T=


24.

seconds.

If resonance exists, the input frequency f is the same as the natural frequency 0 = 2 3 (see Problem 23). Hence, we have the initial-value problem x + 12 x = 16cos 2 3t , x ( 0 ) = x ( 0 ) = 0 . This equation has homogeneous solution xh ( t ) = c1 cos 2 3t + c2 sin 2 3t . To find a particular solution we seek a function of the form
x p = At cos 2 3t + Bt sin 2 3t .

Differentiating and substituting into the differential equation yields

A= 0, B =
so the general solution is

16 4 3

=4

3 , 3

x ( t ) = c1 cos 2 3t + c2 sin 2 3t +

4 3 t sin 2 3t . 3

Substituting this into x ( 0 ) = x ( 0 ) = 0 yields c1 = 0 , c2 = 0 . Hence, the solution to the IVP is x (t ) = 4 3 t sin 2 3t . 3

424

CHAPTER 4

Higher-Order Linear Differential Equations

Eds Buoy 25.

(a)

Simple harmonic motion with m= 2000 125 = = 62.5 slugs 32 2 2 = 5 seconds, T=

hence

0 =
or
2 k = m0 =

2 = 5

k m

125 4 2 = 10 2 . 2 25
4+2 = 3 feet being above water. 2

We measure the displacement of the buoy x ( t ) from the water level with x ( t ) = 0 corresponding to the position of the buoy with

Because the forced equation in rough seas has an amplitude of 3 feet and a period 2 . We therefore get the equation of 7 seconds, the frequency of the forced response is 7 2 t . 62.5 x + 10 2 x = 3cos 7 We are interested in the steady-state solution of this equation, hence we use the method of undetermined coefficients to find a particular solution. In this case we let
x p = A cos 2 t 2 t + B sin . 7 7

We now differentiate and substitute into the equation yielding A = we have x p (t ) = xss ( t ) = 49 2 t 2 t 0.06cos . cos 2 7 7 80

49 , B = 0 . Hence, 80 2

[Although no friction term has been included in the preceding DE, there will in reality be such a term, so the homogeneous solution would go to zero leaving only the oscillation xss ( t ) .]

SECTION 4.6

Forced Oscillations

425

(b)

The steady-state solution never varies more than

49 0.06 feet from its equilibrium 80 2 position 3 feet above the level water line. The steady-state solution has the buoy moving in phase with the waves so when a 3-foot wave crest hits, the buoys height above sea level is approximately 3.06 feet. Thus the buoy is always at least 0.06 feet above the water and is never submerged.

General Solution of the Damped Forced System 26.

(a)

We know the form of the particular solution is xss ( t ) = A cos f t + B sin f t . Substituting this into the equation mx + bx + kx = F0 cos f t and simplifying, we find k m 2 A + b f B cos f t + k m 2 B b f A sin f t = F0 cos f t . f f Setting the coefficients of the sine and cosine terms equal, yields the two equations

( k m ) A + b ( k m ) B b
2 f 2 f

f f

B = F0 A = 0.

Solving, we obtain

A=

F0 k m 2 f

( k m )
2 f

)
.

+ b 2 2 f + b 2 2 f

B=

F0b f k m 2 f

(b)

From part (a) we have xss ( t ) =

F0 k m
2 f

+b
2

2 f

k m 2 cos f t + b f sin f t . f

Rewriting this in polar form, yields xss ( t ) =


b f

F0

( k m )
2 f

+b
2

cos f t ,
2 f

with tan =

2 m 0 2 f

. From this equation it can be seen that the long-term response

of the system is oscillatory with the same frequency f as the forcing term, but with a phase lag.

426

CHAPTER 4

Higher-Order Linear Differential Equations

Phase Portrait Recognition 27.

x + 0.3 x + x = cos t (C) We have damping but we also have a sinusoidal forcing term. Hence, the homogeneous solution goes to zero and particular solutions consist of sines and cosines, which give rise to circles in the phase plane. Therefore, starting from the origin x ( 0 ) = x ( 0 ) = 0 we get a curve that approaches a circle from the inside.

28.

x+x=0 (A) The equation models the undamped harmonic oscillator, which has circular trajectories.

29.

x + x = cos t (D) This equation has resonance so the trajectories in phase space spiral to infinity.

30.

x + 0.3x + x = 0 (B) The system is unforced but damped, and hence trajectories must approach x ( 0 ) = x ( 0 ) = 0 .
Matching 3D Graphs

31.

(a) (e)

E B, D, E

(b) (f)

B C, D

(c) (g)

C, D A

(d)

A, C

Mass-Spring Analysis I 32.

(a) (c) (d) (e) (f)

xh = 4cos 4t 3sin 4t

(b)

The amplitude of xh = 5 .

The amplitude (time-varying) of x p is 5t . x p = 5t sin 4t ; x p will be unchanged. Because f = 0 , 0 = 4 = k = k , k = 16 . m

The system is in a state of pure resonance because 0 = f . The mass will oscillate with increasing amplitude.

Electrical Version 33.

(a) (b) (c) (e) (f)

Qh = 4cos 4t 5sin 4t
The amplitude of the transient solution is A = 42 + 52 = 41 Qs = Q p = 6t cos 4t 4= 1 1 1 = , C= 16 LC C (d) Q p = 6t cos 4t

The charge on the capacitor will oscillate with ever-increasing amplitude due to pure resonance.

SECTION 4.6

Forced Oscillations

427

Mass-Spring Analysis II 34.

(a) (b) (c) (d) (e)


(f)

xh = 3e 2t cos t 2e2t sin t From the exponential function e2t we see that Underdamped The amplitude (time-varying) of xh = 13e 2t .
xss = x p = 2 cos ( 5t ) 4k 16 so k = 5 Nt m , and 0 = 5 rad sec . From For2 F0 F0 2= = . Therefore F0 = 40 Nt . 2 2 800 2 5 5 + ( 4 5)
b = 2. Hence if m = 1, b = 4 . 2m

f = 5 rad sec , = 1 =
mula (19), we obtain

Perfect Aim 35.

(a)

The dart is fired straight at the target with initial velocity vo. Let yD denote the vertical position of the dart at time t. yD = g yD = gt + c yD (0) = vo sin so yD = gt + v0 sin Integrating: 1 yD = gt 2 + (v0 sin )t + c 2
2 2 d = x0 + y0

1 yD (0) = 0 so yD = gt 2 + (v0 sin )t 2 Now consider the target. Let yT denote the vertical position of the target at time T. The 1 initial conditions are yT(0) = y0 and yT (0) = 0. By similar calculations, yT = y0 gt2. 2 (b) To find the time t1 when the heights of dart and target are equal, set yT(t) = yD(t). Then yo y0 y0 1 2 1 gt = v0 sin t gt 2 so that T1 = and x1 = (v0 cos ) 2 2 v0 sin t v0 sin y0 y0 = so that x1 = x0 (i.e., the dart hits the target). x0 x1 y0 = tan

However, tan = (c)

Substituting t1 into either equation for the height of the dart or the target at impact yields yT. 2 gy0 yT = y0 2(v0 sin ) 2 y Simplifying by using the diagram so that sin = 0 , we obtain d 1 d yT = y0 g . 2 v0
2

428

CHAPTER 4

Higher-Order Linear Differential Equations

Extrema of the Amplitude Response 36.

We write A f =

( )

F0

( k m )
2 f

= +b
2 2 f

F0 / m k b 2 2 m f + m f
2 2

Differentiating A with respect to f , we find

A f

( )

2 f 2 f m = 2 k 2 + f m

k ( m 2bm ) F b ) ( m )
2 2 2 32 2 f

from which it follows that A f = 0 if and only if f = 0 or

( )

f =

k b2 . m 2m 2

When b 2 > 2mk , f is not real. Hence A f = 0 only when f = 0 . In this case A f

( )

( )

damps to zero as f goes from 0 to . It is clear then that the maximum of A f

( )

occurs when f = 0 and has the value


A(0) = 1 . k

When b 2 < 2mk , then f is real and positive. It is easy using the sign of the derivative to see that the maximum of A f

( ) occurs at

f =

k b2 . m 2m 2

Evaluating the amplitude response at this value of f yields the expression

Amax =

F0
2 b k b 2 m 4m

Suggested Journal Entry 37.

Student Project

SECTION 4.7

Conservation and Conversion

429

4.7

Conservation and Conversion

Total Energy of a Mass-Spring 1.

x + x = 0 , x ( 0 ) = 1 , x ( 0 ) = 4 The total energy of the system is E = 1 2 1 2 mx + kx . Here m = 1 , k = 1 , so 2 2


E= 1 2 1 2 x + x . 2 2

Because the system is conservative it does not change over time. Initially we have x ( 0 ) = 1 , x ( 0 ) = 4 , so the initial energy of this system is E= which remains constant in time.
Nonconservative Mass-Spring System 2.

1 1 17 ( 4 )2 + (1) = , 2 2 2

x + 2 x + 26 x = 0 , x ( 0 ) = 1 , x ( 0 ) = 4 (a) The solution of the IVP is x ( t ) = e t sin ( 5t ) + cos ( 5t ) . At time t =

we have x = e 5 . 5

Also
x ( t ) = e t sin ( 5t ) + cos ( 5t ) + e t 5cos ( 5t ) 5sin ( 5t ) ,

so x = e 5 5e 5 = 4e 5 . 5 (b) Because m = 1 , k = 26 , we have 1 1 1 E = m x + k x = 16e2 2 5 2 5 2 5


2 2

) + 1 ( 26e 2

2 5

) = 21e

2 5

430

CHAPTER 4

Higher-Order Linear Differential Equations

(c)

Because the initial energy of the system was E ( 0) =


2 2 1 1 1 1 m x ( 0 ) + k x ( 0 ) = 1 42 + 26 12 = 21 ( joules or ergs ) 2 2 2 2

) (

the energy loss = 21 21e 2 5 = 21 1 e 2

).

General Formula for Total Energy in an LC-Circuit 3.


LQ + 1 Q = 0 , Q ( 0 ) = Q0 , I ( 0 ) = I 0 C

The total energy of this LC system is the constant value E (t ) =


Energy in an LC-Circuit 4.

1 1 2 1 2 1 2 LQ 2 + Q = LI 0 + Q0 . 2 2C 2 2C

LQ +

1 Q = 0 , Q ( 0 ) = Q0 , I ( 0 ) = I 0 C

The total energy of this LC system is the constant value E (t ) = 1 1 2 1 2 1 2 1 16 2 LQ 2 + Q = LI 0 + Q0 = 4 12 + 4 = 130 . 2 2C 2 2C 2 2

( )

( )

Energy Loss in LRC-Circuit 5.

LQ + RQ + CQ = 0 , Q ( 0 ) = Q0 , I ( 0 ) = I 0 We are given L = 1 henry, R = 1 ohm, C = 4 farads, Q0 = 0 coulomb, I 0 = 2 amps. Hence, the IVP is Q + Q + 0.25Q = 0 , Q ( 0 ) = 0 , I ( 0 ) = 2 , whose solution is given by Q ( t ) = 2te t 2 I ( t ) = Q ( t ) = 2e t 2 te t 2 = e t 2 ( t 2 ) . Hence, the initial energy is E (0) = At time t the energy is E (t ) = 1 2 1 2 1 1 1 2 2 LI ( t ) + Q ( t ) = e t ( t 2 ) + 4t 2 e t = e t ( t 2 ) + t 2 = e t t 2 2t + 2 . 2 2C 2 8 2 1 2 1 2 1 1 LI 0 + Q0 = (1) 22 + ( 0 ) = 2 joules. 2 2C 2 8

( ) )

Hence, after time t the energy loss is 2 e t t 2 2t + 2 joules.

SECTION 4.7

Conservation and Conversion

431

Questions of Energy 6.

x x + x3 = 0 (a) KE = 1 2 1 1 x , V = x + x 3 dx = x 2 + x 4 2 2 4
1 2 1 2 1 4 x x + x 2 2 4

E ( x, x ) = KE + V =

(b)

To find the equilibrium points, we seek the solutions of E E = x + x3 = 0 , = x =0. x x Solving these equations, we find three equilibrium points at ( 1, 0 ) , (0, 0) and (1, 0). Because x = 0 for all these points, we determine which points are stable (local maxima) by simply drawing the graph of V ( x ) (shown in part (c)).

(c)

Graph of the potential energy V ( x ) is shown. Note that V ( x ) has local minima at x = 1 and a local maxima at x = 0 . Hence,

V ( x) 3 2

( 1, 0 )

and (1, 0 )

are stable points, and

( 0, 0 )
is an unstable point.

1 0.5

Potential energy of x x + x3 = 0
7.

x x x3 = 0 (a) KE = 1 2 1 1 x , V = x x3 dx = x 2 x 4 2 2 4
1 2 1 2 1 4 x x x 2 2 4

E ( x, x ) = KE + V =

(b)

To find the equilibrium points, we seek the solutions of the two equations E E = x x3 = 0 , = x =0. x x Solving these equations, we find one equilibrium point at ( 0, 0 ) . Because x = 0 we determine if it is a stable point (local minima) or unstable point (local maxima) by simply drawing the graph of V ( x ) shown in part (c).

432

CHAPTER 4

Higher-Order Linear Differential Equations

The graph of potential energy V ( x ) is shown. Note that V ( x ) has local maxima at x = 0 , and hence ( 0, 0 ) is an unstable equilibrium point. (c) See the figure to the right.
2 1

V ( x) 1

1 1 2

Potential energy of x x x3 = 0
8.

x x + x2 = 0 (a) KE = 1 2 1 1 x , V = x + x 2 dx = x 2 + x3 2 2 3 1 2 1 2 1 3 x x + x 2 2 3

E ( x, x ) = KE + V = (b)

To find the equilibrium points, we seek the solutions of the two equations
E E = x + x2 = 0 , = x =0. x x

Solving these equations, we find two equilibrium points at ( 0, 0 ) , (1, 0 ) . Because x = 0 for both these points, we determine which points are stable (local minima) and which are unstable (local maxima) by simply drawing the graph of V ( x ) shown in part (c). The graph of potential energy V ( x ) is shown. Note that V ( x ) has local minima at x = +1 and a local maxima at x = 0 . Hence, ( 0, 0 ) is an unstable point and
V ( x) 1

(1, 0 )
(c)

1 1

is a stable point.

See figure.
2

Potential energy of x x + x 2 = 0
9.

x + x2 = 0 (a) KE = 1 2 1 x , V = x 2 dx = x3 2 3

SECTION 4.7

Conservation and Conversion

433

E ( x, x ) = KE + V = (b)

1 2 1 3 x + x 2 3

To find the equilibrium points, we seek the solutions of the equations


E E = x2 = 0 , = x =0. x x

Solving these equations, we find one equilibrium point at

( 0, 0 ) .
To determine if the point is stable, we note that the potential energy V ( x) = 1 3 x 3
2 1 1 2 V ( x) 3 2 x

does not have a local maxima or minima at ( 0, 0 ) , so ( 0, 0 ) is an unstable (or semistable) equilibrium point. (c) The graph of V ( x ) is a simple cubic.

Potential energy of x + x 2 = 0
10.

x ex 1 = 0
(a)
KE = 1 2 x , V = e x 1 dx = e x x 2

E ( x, x ) = KE + V = (b)

1 2 x ex x 2

To find the equilibrium points, we seek the solutions of the two equations E E = e x 1 = 0 , = x =0. x x It is clear that the first of these equations has no root, so the equation has no equilibrium points.
V ( x) 2 1 x

(c)

Note that the graph of the potential energy V ( x ) does not have any local maxima or minima points, which corresponds to the lack of equilibrium points found in part (b).

1 1 2

Potential energy of x e x 1 = 0

434

CHAPTER 4

Higher-Order Linear Differential Equations


2

11.

x + ( x 1) = 0 (a)
KE = 1 2 1 2 x , V = ( x 1) dx = x 3 x 2 + x 2 3

E ( x, x ) = KE + V = (b)

1 2 1 3 x + x x2 + x 2 3

To find the equilibrium points, we seek the solutions of the two equations
E E = x2 2x + 1 = 0 , = x =0. x x

Solving these equations, yields only one real equilibrium point (1, 0 ) . Because x = 0 , we determine if it is stable (local minima) or unstable (local maxima) by simply drawing the graph of V ( x ) show in part (c). Thus, we find that (1, 0 ) is an unstable (or semistable) equilibrium point. (c) The graph of the potential energy V ( x ) is shown. Note that V ( x ) has neither a maximum nor a minimum at x = 1 , and hence (1, 0 ) is an unstable (or semistable) equilibrium point.
2 1 1 2 V ( x) 2 1 x

Potential energy of x + ( x 1) = 0
2

12.

x= (a)

1 x2
KE = 1 2 1 1 x , V = 2 dx = 2 x x 1 2 1 x + 2 x

E ( x, x ) = KE + V =

(b)

To find the equilibrium points, we seek the solutions of the two equations E 1 E = 2 =0, = x =0. x x x Because the first equation does not have a solution, there is no equilibrium point.

SECTION 4.7

Conservation and Conversion

435

(c)

The graph of the potential energy V ( x ) is shown. Note that V ( x ) does not have any local maxima or minima, which corresponds to the absence of equilibrium points noted in part (b).
2 1

V ( x) 4 2 x

1 2 4

Potential energy of x = 1 x 2
13.

x = ( x 1)( x 2 ) (a) KE = 1 2 1 3 x , V = ( x 1)( x 2 ) dx = x3 + x 2 2 x 2 3 2 1 2 1 3 3 2 x x + x 2x 2 3 2

E ( x, x ) = KE + V = (b)

To find the equilibrium points, we seek the solutions of the two equations
E E = x 2 + 3x 2 = 0 , = x =0. x x

Solving these equations, we find two equilibrium points at (1, 0 ) and ( 2, 0 ) . Because x = 0 , we determine which points are stable (local minima) and which are unstable (local maxima) by simply drawing the graph of V ( x ) show in part (c). (c) The graph of the potential energy V ( x ) is shown. Note that V ( x ) has local minima at x = 1 and a local maxima at x = 2 . Hence, (1, 0 ) is a stable point and
V ( x) 2 1 x

( 2, 0 )

is an unstable point.

2 1 2

Potential energy of x = ( x 1)( x 2 )

436

CHAPTER 4

Higher-Order Linear Differential Equations

Conservative or Nonconservative? 14.

x + x2 = 0 Conservative because it is of the form mx + F ( x ) = 0 . The total energy of this conservative system is E ( x, x ) = 1 1 1 mx + F ( x ) dx = x 2 + x3 . 2 2 3
4 2

x 4

2 2 2 4 4 x

We draw contour curves for this surface over the xx -plane to view the trajectories of the differential equation in the xx plane.
15.

x + kx = 0 Conservative because it has the form mx + F ( x ) = 0 . The total energy of this conservative system is 1 1 1 E ( x, x ) = mx + F ( x ) dx = x 2 + kx 2 . 2 2 2
4 2

4 2 2 2 4 4 x

We draw contour curves for this surface over the xx -plane to view the trajectories of the differential equation in the xx plane. The trajectories of x + kx = 0 are ellipses each with height times its width.
16.

x + x + x2 = 1
Not conservative due to the x term. The spiral trajectories in its phase plane cannot be level curves of any surface.
5

x 5

SECTION 4.7

Conservation and Conversion

437

17.

+ sin = 0
Conservative because it is of the form m + F ( ) = 0 . The total energy of this nonconservative system is E , =

4 2

1 1 m + F ( ) d = 2 cos . 2 2

We can draw contour curves for this surface over the -plane to view the trajectories of the differential equation in the plane.
18.

+ sin = 1
Conservative because it can be written in the form

2 1

m + F ( ) = 0 ,
where F ( ) = sin 1 . The total energy is
1 E , = 2 cos . 2

8 4 1 2 4 8

We can draw contour curves for this surface over the -plane to view the trajectories of the differential equation in the plane.
19.

+ + sin = 1
Not conservative due to the term. The

4 2
2

following phase plane portrait shows equilibria along the axis. Trajectory cannot be level curves for any surface.

2 4

Trajectories of a nonconservative system

438

CHAPTER 4

Higher-Order Linear Differential Equations

Time-Reversible Systems 20.

(a)

mx = F ( x ) . If we introduce backwards time = t , then taking the derivatives, yields dx dx d dx = = dt d dt d 2 d x d d dx d d 2x d 2x = ( x) = = 2 ( 1) = 2 d d dt dt 2 dt d d The conservative system x + F ( x ) = 0 is transformed into exactly the same equation d 2x + F ( x) = 0 d 2 in backwards time .

(b)

The solution of the IVP

x + x = 0 , x (0) = 1 , x ( 0) = 0
is x ( t ) = cos t . If we replace t by t, it yields the solution x ( t ) = cos ( t ) = cos t . Hence, running the system backwards looks exactly like running the system forward. (c) The solution of the IVP x = mg , x ( 0 ) = 0 , x ( 0 ) = 100 is
1 x ( t ) = mgt 2 + 100t . 2

1 If we replace t by t, we get x ( t ) = mgt 2 100t . Hence, the solution is not the same, 2 and the system is not time reversible. (d) If we think of a time-reversible system as a system where equations of motion are the same when we replace t by t, we might make the following conclusions. (i) (iv) yes no (ii) (v) no yes (iii) no

Computer Lab: Undamped Spring 21.

IDE Lab
Computer Lab: Damped Spring

22.

IDE Lab

SECTION 4.7

Conservation and Conversion

439

Conversion of Equations 23.


2 x + 0 x = f ( t )

Letting x1 = x , x2 = x , we have x1 = x2
2 x2 = 0 x1 + f ( t ) .

In matrix form, this becomes x1 0 1 x1 0 + . x = 2 2 0 0 x2 f ( t )


24.

g sin = 0 L

Letting x1 = , x2 = , we have x1 = x2 x2 = g sin x1 . L

This system is not linear, so there is no matrix form.


25.

ay + by + cy = 0
Letting x1 = y , x2 = y , we have x1 = x2 c b x2 = cx1 x2 . a a In matrix form, this becomes 1 0 x1 x1 b . x = c x2 2 a a

26.

LQ + RQ +

1 Q=0 C
dQ , we have dt

Letting x1 = Q , x2 =

x1 = x2 x2 = 1 R x1 x2 . LC L

440

CHAPTER 4

Higher-Order Linear Differential Equations

In matrix form, this becomes 0 1 x1 x1 = R . x 1 x2 2 L LC


27.

t 2 x + tx + t 2 n 2 x = 0 Letting x1 = x , x2 = x , we have x1 = x2 x2 In matrix form, this becomes 0 1 x1 2 x1 2 1 . x = t n x2 2 2 t t

(t =

n2 t
2

) x 1x .
1

28.

x + (1 + sin t ) x = 0 Letting x1 = x , x2 = x , we have x2 = (1 + sin t ) x1 . In matrix form, this becomes 0 1 x1 x1 x = 1 + sin t 0 x . ) 2 2 ( x1 = x2

29.

(1 t ) y 2ty + n ( n + 1) y = 0
2

Letting x1 = y , x2 = y , we have x1 = x2 x2 = In matrix form, this becomes 0 x1 x = n +1 2 n 1 t2 1 x1 2t . x2 1 t2 n ( n + 1) 1 t


2

x1 +

2t x2 . 1 t2

SECTION 4.7

Conservation and Conversion

441

30.

d4y d3y d 2 y dy +3 3 +2 2 + + 4y =1 dt dt 4 dt dt If we introduce x1 = y dy dt d2y x3 = 2 dt d3y x4 = 3 dt x2 = we have the differential equations x1 = x2 x2 = x3 x3 = x4 x4 = 4 x1 x2 2 x3 3 x4 + 1 or in matrix form x1 0 1 0 0 x1 0 x 0 0 1 0 x2 0 2 = + . x3 0 0 0 1 x3 0 x4 4 1 2 3 x4 1
Conversion of IVPs

31.

y y + 2 y = sin t , y ( 0 ) = 1 , y ( 0 ) = 1
Letting x1 = y , x2 = y yields x1 = x2 x2 = 2 x1 + x2 + sin t In matrix form this becomes x1 0 1 x1 0 x = 2 1 x + sin t ; 2 2 x1 ( 0 ) 1 = . x2 ( 0 ) 1 x1 ( 0 ) = 1 . x2 ( 0 ) = 1

32.

y + ty + y = 1 , y ( 0 ) = 0 , y ( 0 ) = 1 , y ( 0 ) = 2
Letting x1 = y , x2 = y , x3 = y yields x1 = x2 x1 ( 0 ) = 1 x2 = x3 x2 ( 0 ) = 1 . x3 = x1 tx2 + 1 x3 ( 0 ) = 2

442

CHAPTER 4

Higher-Order Linear Differential Equations

In matrix form, this becomes x1 0 1 0 x1 0 x = 0 0 1 x + 0 , 2 2 x3 1 t 0 x3 1


33.

x1 ( 0 ) 0 x2 ( 0 ) = 1 . x3 ( 0 ) 2

y + 3 y + 2 z = e t , y ( 0 ) = 0 , y ( 0 ) = 1

z + y + 2 z = 1 , z ( 0 ) = 1 , z ( 0 ) = 0
Letting x1 = y , x2 = y , x3 = z , x4 = z yields x1 = x2 x2 = 3 x2 2 x3 + e t x3 = x4 x4 = x1 2 x3 + 1 In matrix form this becomes 1 0 x1 0 x 0 3 2 2 = x3 0 0 0 x4 1 0 2
34.

x3 ( 0 ) = 1 x4 ( 0 ) = 0

x2 ( 0 ) = 1

x1 ( 0 ) = 0 .

0 x1 0 0 x2 e t + , 1 x3 0 0 x4 1

x1 ( 0 ) 0 x2 ( 0 ) = 1 . x3 ( 0 ) 1 x4 ( 0 ) 0

y + y + 2 z = 1 , y ( 0 ) = 1 , y ( 0 ) = 0 , y ( 0 ) = 1 z + y + 2 z = sin t , z ( 0 ) = 1 Letting x1 = y , x2 = y , x3 = y , x4 = z yields x1 = x2 x2 = x3 x3 = x2 2 x4 + 1 x4 = x1 2 x4 + sin t In matrix form this becomes x1 0 1 x 0 0 2 = x3 0 1 x4 1 0 0 0 x1 0 x1 ( 0 ) 0 1 0 x2 0 x2 ( 0 ) 0 + = . 0 2 x3 1 x3 ( 0 ) 1 0 2 x4 sin t x4 ( 0 ) 1 x1 ( 0 ) = 0 x2 ( 0 ) = 0 . x3 ( 0 ) = 1 x4 ( 0 ) = 1

SECTION 4.7

Conservation and Conversion

443

Conversion of Systems 35.

x1 + x1 + 2 x2 = e t x2 + 2 x2 = 0 Letting z1 = x1 , z2 = x1 , z3 = x2 , z4 = x2 yields the system z1 = z2 z2 = z1 2 z3 + e t z3 = z4 z4 = 2 z3 . In matrix form this becomes z1 0 z 1 2 = z3 0 z4 0 1 0 0 2 0 0 0 2 0 z1 0 0 z2 e t + . 1 z3 0 0 z4 0

36.

y = f ( t , y, y , y , z , z ) z = f ( t , y, y, y , z , z ) Letting x1 = y , x2 = y , x3 = y , x4 = z , x5 = z yields
x1 = x2 x2 = x3 x4 = x5 x1 = a11 x1 + a12 x2 + a13 x3 x2 = a21 x1 + a22 x2 + a23 x3 x3 = a31 x1 + a32 x2 + a33 x3 If we let z1 = x1 , z2 = x1 , z3 = x2 , z4 = x2 , z5 = x3 , z6 = x3 , we get
z1 = z2 z2 = a11 z1 + a12 z3 + a13 z5 z3 = z4 z4 = a21 z1 + a22 z3 + a23 z5 z5 = z6 z6 = a31 z1 + a32 z3 + a33 z5 .

x3 = f ( t , x1 , x2 , x3 , x4 , x5 )

x5 = g ( t , x1 , x2 , x3 , x4 , x5 ) .

37.

444

CHAPTER 4

Higher-Order Linear Differential Equations

In matrix form z = Az , this becomes z1 0 z a 2 11 z3 0 = z4 a21 z5 0 z6 a31 1 0 0 a12 0 0 0 a22 0 0 0 a32 0 0 0 a13 1 0 0 a23 0 0 0 a33 0 z1 0 z2 0 z3 . 0 z4 1 z5 0 z6

Solving Linear Systems 38.


x1 = x2 x2 = 2 x1 3 x2

From first DE: x2 = x1 . Substituting in second DE gives

( x1 ) = 2 x1 3 ( x1 )
or the second order DE x1 + 3 x1 + 2 x1 = 0 . Solving the second order DE gives x1 = c1e 2t + c2 e t . Substituting this result in first DE gives x2 = x1 = 2c1e 2t c2 e t . x1 = 3x1 2 x2 x2 = 2 x1 2 x2 1 3 From first DE: x2 = x1 + x1 . Substituting in second DE yields a second order DE to solve for 2 2 x1 . 3 3 1 1 x1 + x1 = 2 x1 2 x1 + x1 2 2 2 2 1 3 x1 + x1 = 2 x1 + x1 3 x1 2 2 x1 x1 2 x1 = 0 x1 = c1e 2t + c2 e t To find x2 , substitute the solution for x1 back into the first DE.
1 3 1 3 1 x2 = x1 + x1 = 2c1e 2t c2 e t + c1e 2t + c2 e t = c1e 2t + 2c2 et . 2 2 2 2 2

39.

) (

SECTION 4.7

Conservation and Conversion

445

40.

x1 = x1 + x2 x2 = 4 x1 + x2 From first DE: x2 = x1 x1 . Substituting in second DE yields a second order DE to solve for x1 .

( x1 x1 ) = 4 x1 + ( x1 x1 )
x1 x1 = 4 x1 + x1 x1 x1 2 x1 3x1 = 0 x1 = c1e3t + c2 e t From first calculation, x2 = x1 x1 , so x2 = 3c1e3t c2 e t c1e3t + c2 e t = 2c1e3t 2c2 e t .

) (

41.

x1 = x2 + t x2 = 2 x1 + 3 x2 + 5 From first DE: x2 = x1 t . Substituting in second DE yields a second order DE to solve for x1 .

( x1 t ) = 2 x1 + 3 ( x1 t ) + 5
x1 1 = 2 x1 + 3 x1 3t + 5 x1 3x1 + 2 x1 = 3t + 6. x1h = c1e 2t + c2 et To find x1 p by the method of undetermined coefficients, substitute x1 p = at + b , x1 p = a , x1 p = 0 to obtain

0 3a + 2at + 2b = 3t + 6 .
Comparing like terms,
3 Coefficients of t: 2a = 3 so a = . 2

Constants: 3a + 2b = 6 so b =

3 3 3 . Hence x1 p = t + . Therefore, 4 2 4 3 3 x1 = c1e 2t + c2 et t + . 2 4

From first calculation x2 = x1 t , so


x2 = 2c1e 2t + c2 et 3 t . 2

446

CHAPTER 4

Higher-Order Linear Differential Equations

Solving IVPs for Systems 42.


x1 = 6 x1 3 x2 x2 = 2 x1 + x2 1 From first DE: x2 = 2 x1 x1 . Substituting in second DE yields a second order DE to solve for x1 . 3 1 1 2 x1 x1 = 2 x1 + 2 x1 x1 3 3 x1 7 x1 + 12 x1 = 0 x1 = c1e3t + c2 e 4t .
1 From first calculation, x2 = 2 x1 x1 , so 3

x2 = 2 c1e3t + c2 e 4t Applying initial conditions:

) 1 ( 3c e 3
1

3t

2 + 4c2 e 4t = c1e3t + c2 e 4t . 3

c1 + c2 = 2 2 x2 ( 0 ) = 3 c1 + c2 = 3 3 so c2 = 3 and c1 = 5 . The solution to the IVP is x1 = 5e3t 3e 4t , x2 = 5e3t 2e 4t .

x1 ( 0 ) = 2

43.

x1 = 3 x1 + 4 x2 x2 = 2 x1 + x2 From first DE: x2 = x1 . 3 3 1 1 x1 x1 = 2 x1 + x1 x1 4 4 4 4 x1 3 x1 = 8 x1 + x1 3 x1 x1 4 x1 5 x1 = 0 x1 = c1e5t + c2 e t From first calculation, x2 = x2 =


1 3 x1 x1 , so 4 4

1 3 x1 x1 . Substituting in second DE yields a second order DE to solve for 4 4

1 3 1 5c1e5t c2 e t c1e5t + c2 e t = c1e5t c2 e t 4 4 2

) (

SECTION 4.7

Conservation and Conversion

447

Applying initial conditions: x1 ( 0 ) = 1 c1 + c2 = 1 1 c1 c2 = 1 2 x2 ( 0 ) = 1 so c1 = 0 and c2 = 1 . The solution to the IVP is x1 = e t , x2 = e t .

Counterexample 44.
An example: The degenerate system x1 + x2 + x1 = 0 x1 + x2 + x1 = 0 where both equations are exactly the same clearly cannot be written as a second-order equation in either x1 or x2 . The reader might contemplate finding all the solutions of such an undetermined system. Another approach: Note that when we write an nth-order equation such as ay + by + cy = 0 as a system of first-order equations by letting x1 = y , x2 = y , the system has the form 1 0 x1 x1 b . x = c x2 2 a a This shows we cannot obtain a second-order equation in x1 with x2 = x1 unless the coefficient matrix has the preceding form in which the first row contains a 0 and 1. Hence, a system such as x1 1 1 x1 x = 4 1 x 2 2 cannot be transformed into a second-order equation in x1 with x2 = x1 .

Coupled Mass-Spring System 45.


Given the linear system mx1 = k1 x1 + k2 ( x2 x1 ) = ( k1 + k2 ) x1 + k2 x2 mx2 = k2 ( x2 x1 ) = k2 x1 k2 x2 ,

448

CHAPTER 4

Higher-Order Linear Differential Equations

we let z1 = x1 z2 = x1 We then have the first-order system z1 = z2 z2 = k1 + k2 k z1 + 2 z3 m m z3 = x2 z4 = x2 .

z3 = z4 k k z4 = 2 z1 2 z3 . m m In matrix form this becomes 0 z1 ( k + k ) z 1 2 m 2 = z3 0 k2 z4 m 1 0 0 0 z 1 0 z 2 . 1 z3 z4 0

k2 m 0 0 k 0 2 m

Satellite Problem 46.


r = r ( t ) 2 ( t ) r (t )
2

+ u1 ( t )

2 ( t ) r ( t ) r (t )

1 u2 ( t ) r (t )

Letting x1 = r x2 = r we have the system x3 = x4 = ,

x1 = x2
2 x2 = x1 x4

k + u1 ( t ) x12

x3 = x4 x4 = 2 x2 x4 1 + u2 ( t ) . x1 x1

SECTION 4.7

Conservation and Conversion

449

Two Inverted Pendulums 47.

1 = ( mg + 1)1 + mg 2 u ( t ) 2 = mg1 + ( mg + 1) 2 u ( t )
Letting x1 = 1 x2 = 1 x3 = 2 x4 = 2 we have first-order linear system
x2 = ( mg + 1) x1 + mgx3 u ( t ) x4 = mgx1 + ( mg + 1) x3 u ( t ) . In matrix form this becomes x1 0 x mg + 1 2 = x3 0 x4 mg 1 0 0 mg 0 0 0 mg + 1 0 x1 0 x u (t ) 0 2 . + 1 x3 0 0 x4 u (t ) x3 = x4 x1 = x2

Suggested Journal Entry 48.


Student Project

S-ar putea să vă placă și